Not for Sale Applications of 2 Differentiation

What You’ll Learn Why It’s Important 2.1 Using First to Classify Maximum In this chapter, we explore many applications and Minimum Values and Sketch Graphs of differentiation. We learn to find maximum 2.2 Using Second Derivatives to Classify and minimum values of functions, and that skill Maximum and Minimum Values and Sketch allows us to solve many kinds of problems in Graphs which we need to find the largest and/or smallest 2.3 Graph Sketching: and Rational value in a real-world situation. Our differentiation Functions skills will be applied to graphing, and we will use 2.4 Using Derivatives to Find Absolute differentials to approximate values. We Maximum and Minimum Values will also use differentiation to explore elasticity of 2.5 Maximum–Minimum Problems; Business, demand and related rates. Economics, and General Applications 2.6 Marginals and Differentials 2.7 Elasticity of Demand 2.8 Implicit Differentiation and Related Rates

Where It’s Used y C(r) Minimizing Cost: Minimizing cost is a common goal in 0.32 manufacturing. For example, cylindrical food cans come in a variety of sizes. Suppose a soup can is to have a volume of 250 cm3. 0.24 (4.21, 0.2673) 2 The cost of material for the two circular ends is $0.0008 cm , and 0.16 the cost of material for the side is $0.0015 cm2. What dimensions > Cost (in dollars) 0.08 minimize the cost of material for the soup can?(This problem > appears as Example 3 in Section 2.5.) 20 4 6 8 r Radius (in centimeters)

188 Copyright 2016 Pearson. All rights reserved.

M02_BITT9391_11_SE_C02.1.indd 188 29/10/14 4:39 PM Not for Sale

2.1 ● Using First Derivatives to Classify Maximum and Minimum Values and Sketch Graphs 189

Using First Derivatives to Classify Maximum and Minimum Values 2.1 and Sketch Graphs The graph below shows a typical life cycle of a retail product and is similar to graphs ● Find relative extrema of a using the we will consider in this chapter. Note that the number of items sold varies with re- First- Test. spect to time. Sales begin at a small level and increase to a point of maximum sales, after which they taper off to a low level, possibly because of new competitive prod- ● Sketch graphs of continuous ucts. The company then rejuvenates the product by making improvements. Think functions. about versions of certain products: televisions can be traditional, flat-screen, or high- definition; music recordings have been produced as phonograph (vinyl) records, au- diotapes, compact discs, and MP3 files. Where might each of these products be in a typical product life cycle? Is the curve below appropriate for each product?

S PRODUCT LIFE CYCLE Sales of product

t Introduction Growth Maturity Decline Rejuvenation Time

Finding the largest and smallest values of a function—that is, the maximum and minimum values—has extensive applications. The first and second derivatives of a function can provide information for graphing functions and finding maximum and minimum values.

Increasing and Decreasing Functions If the rises from left to right over an interval I, the function is said to be increasing on, or over, I. If the graph drops from left to right, the function is said to be decreasing on, or over, I.

y y f Decreasing f(b) g(a)

f(a) g(b) Increasing g

a b x a b x If the input a is less than the input b, If the input a is less than the input b, then the output for a is less than the then the output for a is greater than the output for b. output for b.

Copyright 2016 Pearson. All rights reserved.

M02_BITT9391_11_SE_C02.1.indd 189 29/10/14 4:39 PM Not for Sale

190 Chapter 2 ● Applications of Differentiation

Technology connecTIon We can define these concepts as follows. Exploratory Definitions Graph the function A function f is increasing over I if, for every a and b in I, 1 3 2 f x = - 3x + 6x - 11x - 50 if a 6 b, then f a 6 f b . and1 2 its derivative A function f is decreasing over I if, for every a and b in I, 2 1 2 1 2 f′ x = -x + 12x - 11 if a 6 b, then f a 7 f b . using1 2 the window -10, 25, -100, 150 , with Xscl = 5 and 1 2 1 2 3 Yscl = 25. Then trace from left The above definitions can be restated in terms of secant lines as shown below. to right along4 each graph. Mov- ing the cursor from left to right, f b - f a f b - f a Increasing: 7 0. Decreasing: 6 0. note that the x-coordinate always b a b a increases. If the function is 1 2 - 1 2 1 2 - 1 2 increasing, the y-coordinate will y y increase as well. If a function is decreasing, the y-coordinate will f f(b) f(a) decrease. of secant Slope of secant Over what intervals is f line is positive. line is negative. increasing? Over what intervals is f f(a) decreasing? Over what intervals is f′ f positive? Over what intervals is f′ f(b) negative? What rules might relate the sign of f′ to the behavior of f? a b x a b x The following theorem shows how we can use the derivative (the slope of a tan- gent line) to determine whether a function is increasing or decreasing.

theorem 1 Let f be differentiable over an open interval I. If f′ x 7 0 for all x in I, then f is increasing over I. If f′ x 6 0 for all x in I, then f is decreasing over I. 1 2 1 2 1 3 2 Theorem 1 is illustrated in the following graph of f x = 3x - x + 3. 1 2 4± ±1, 3 y 1 f(x)= ± x3 ±x + 2± 3 3 1

±2 ±1 (1, 0) x

Increasing Decreasing Increasing f9(x) > 0 f9(x) < 0 f9(x) > 0 f is increasing over the intervals (±, ∞ ±1) and (1, ∞); of lines are positive. f is decreasing over the interval (±1, 1); slopes of tangent lines are negative.

Note in the graph above that x = -1 and x = 1 are not included in any interval over which the function is increasing or decreasing. These values are examples of critical values.

Copyright 2016 Pearson. All rights reserved.

M02_BITT9391_11_SE_C02.1.indd 190 29/10/14 4:39 PM Not for Sale

2.1 ● Using First Derivatives to Classify Maximum and Minimum Values and Sketch Graphs 191

critical Values Consider the following graph of a continuous function f.

y f

x c1 c2 c3 c4 c5 c6 c7 c8

Note the following:

1. f′ x = 0 at x = c1, c2, c4, c7, and c8. That is, the tangent line to the graph is horizontal for these values. 1 2 2. f′ x does not exist at x = c3, c5, and c6. The tangent line is vertical at c3, and there are corners at both c5 and c6. (See also the discussion at the end of Section1 2 1.4.)

Definition A critical value of a function f is any number c in the domain of f for which the tangent line at c, f c is horizontal or for which the derivative does not exist. That is, c is a critical value if f c exists and 1 1 22 f′ c = 0 or f′ c does1 2 not exist. 1 2 1 2 Thus, in the graph of f above:

● c1, c2, c4, c7, and c8 are critical values because f′ c = 0 for each value. ● c , c , and c are critical values because f′ c does not exist for each value. 3 5 6 1 2 A continuous function can change from increasing1 2 to decreasing or from decreas- ing to increasing only at a critical value. In the above graph, c1, c2, c4, c5, c6, and c7 separate intervals over which the function f increases from those in which it decreases or intervals in which the function decreases from those in which it in- creases. Although c3 and c8 are critical values, they do not separate intervals over which the function changes from increasing to decreasing or from decreasing to increasing.

Copyright 2016 Pearson. All rights reserved.

M02_BITT9391_11_SE_C02.1.indd 191 29/10/14 4:39 PM Not for Sale

192 Chapter 2 ● Applications of Differentiation

Finding Relative Maximum and Minimum Values

Now consider a graph with “peaks” and “valleys” at x = c1, c2, and c3.

y

f(b) Relative Maxima maximum f f(c2)

f(c3) Relative Minima f(a) minimum

f(c1)

abc1 c2 c3 x

Here, f c2 is an example of a relative maximum (plural: maxima). Each of f c1 and f c3 is called a relative minimum (plural: minima). Collectively, maximum and minimum1 2values are called extrema (singular: extremum). 1 2 1 2

Definitions Let I be the domain of f.

f c is a relative minimum if there exists within I an open interval I1 containing c such that f c … f x , for all x in I1; 1 2 and 1 2 1 2

f c is a relative maximum if there exists within I an open interval I2 containing c such that f c Ú f x , for all x in I2. 1 2 1 2 1 2 Look again at the graph above. The x-values at which a continuous function has relative extrema are those values for which the derivative is 0 or for which the deriva- tive does not exist—the critical values.

theorem 2 If a function f has a relative extreme value f c on an open interval, then c is a critical value, so 1 2 f′ c = 0 or f′ c does not exist. 1 2 1 2 A relative extreme point, c, f c , is higher or lower than all other points over some open interval containing c. A relative minimum point will have a y-value that is lower than those of a neighborhood1 1 of22 points both to the left and to the right of it, and, similarly, a relative maximum point will have a y-value that is higher than those of a neighborhood of points to the left and right of it. Theorem 2 is useful and important to understand. It says that to find relative extrema, we need only consider inputs for which the derivative is 0 or for which the derivative does not exist. Each critical value is a candidate for a value where a relative extremum might occur. That is, Theorem 2 does not guarantee that every critical value will yield a relative maximum or minimum. Consider, for example, the graph of f x = x - 1 3 + 2,

Copyright1 2 1 2016 Pearson.2 All rights reserved.

M02_BITT9391_11_SE_C02.1.indd 192 29/10/14 4:39 PM Not for Sale

2.1 ● Using First Derivatives to Classify Maximum and Minimum Values and Sketch Graphs 193

y shown at the left. Note that 10 2 2 9 f′ x = 3 x - 1 , and f′ 1 = 3 1 - 1 = 0. 8 Thus, c = 1 is a critical value, but f has no relative maximum or minimum at that 7 1 2 1 2 1 2 1 2 3 6 f(x) = (x ± 1) + 2 value. 5 Theorem 2 does guarantee that if a relative maximum or minimum occurs, then 4 the first coordinate of that extremum is a critical value. How can we tell when the 3 (1, 2) existence of a critical value indicates a relative extremum? The following graph leads ' = 2 f (1) 0 us to a test. 1

±11234 x y ±1 Relative maximum ±2 ±3 Relative ±4 maximum

Relative minimum Relative minimum x Decreasing Increasing Decreasing Increasing Decreasing

Note that at a critical value where there is a relative minimum, the function is decreasing on the left of the critical value and increasing on the right. At a critical value where there is a relative maximum, the function is increasing on the left of the critical value and decreasing on the right. In both cases, the derivative changes signs on either side of the critical value.

Graph over the Sign of f′ x Sign of f′ x Increasing or interval a, b f c for x in a, c for x in c, b decreasing 1 2 1 2 - 1 2 Relative1 2 1 2 + 1 2 Decreasing on a, c ; minimum increasing on c, b 1 2 ± + 1 2

acb Relative + - Increasing on a, c ; + ± maximum decreasing on c, b 1 2 1 2

acb No relative - - Decreasing on a, b maxima or ± 1 2 ± minima

acb + No relative + + Increasing on a, b maxima or 1 2 minima + acb

Copyright 2016 Pearson. All rights reserved.

M02_BITT9391_11_SE_C02.1.indd 193 29/10/14 4:39 PM Not for Sale

194 Chapter 2 ● Applications of Differentiation

Derivatives tell us when a function is increasing or decreasing. This leads us to the First-Derivative Test.

theorem 3 The First-Derivative Test for Relative Extrema For any continuous function f that has exactly one critical value c in an open interval a, b :

F1. f has1 a relative2 minimum at c if f′ x 6 0 on a, c and f′ x 7 0 on c, b . That is, f is decreasing to the left of c and increasing to the right of c. F2. f has a relative maximum at c if f′1 x2 7 0 on 1 a, c2 and f′1 x2 6 0 on 1c, b2. That is, f is increasing to the left of c and decreasing to the right of c. F3. f has neither a relative maximum nor1 2 a relative1 minimum2 at1 2c if f′ x has the1 same2 sign on a, c as on c, b . 1 2 1 2 1 2 We can use the First-Derivative Test to find relative extrema and create accurate graphs.

eXAmPLe 1 Graph the function f given by f x = 4x3 - 9x2 - 30x + 25, and find 1any2 relative extrema. Solution We plot points and use them to create a rough sketch of the graph, shown as the dashed line in the figure below.

y x f x 70 60 -3 -1742 (±1, 42) -2 17 40 -1 42 (0, 25) 0 25 (±2, 17) 20 (4, 17) 1 -10 25324 23 22 21 45x 2 -39 (1, ±10) 2 3 -38 20 4 17 240 (2, ±39) (3, ±38) 260 (±3, ±74) 270

According to this sketch, it appears that the graph has a tangent line with slope 0 at x = -1 and between x = 2 and x = 3. To verify this, we use to support our observations. We begin by finding the derivative: f′ x = 12x2 - 18x - 30. We next determine1 2 where f′ x does not exist or where f′ x = 0. Since f′ x is defined for all real numbers, there is no value x for which f′ x does not exist. 1 2 1 2 1 2 1 2

Copyright 2016 Pearson. All rights reserved.

M02_BITT9391_11_SE_C02.1.indd 194 29/10/14 4:39 PM Not for Sale

2.1 ● Using First Derivatives to Classify Maximum and Minimum Values and Sketch Graphs 195

The only possibilities for critical values are those where f′ x = 0. To find such values, we solve f′ x = 0: 1 2 2 12x - 181x 2- 30 = 0 2 2x - 3x - 5 = 0 Dividing both sides by 6 x + 1 2x - 5 = 0 Factoring x 1 0 or 2x 5 0 Using the Principle of Zero Products 1 + =21 2 - = 5 x = -1 or x = 2. 5 The critical values are -1 and 2. Since it is at these values that a relative maximum or minimum might exist, we analyze the sign of the derivative on the intervals 5 5 - ∞, -1 , -1, 2 , and 2, ∞ . 1 2 1 2 1 2 21, 5 5 , ` (2`, 21) 2)( 2 )(

x ±1 5 2

To do so, we choose a test value in each interval and make a substitution. Let’s use the values -2, 0, and 4. 2 - ∞, -1 : Test -2, f′ -2 = 12 -2 - 18 -2 - 30 48 36 30 54 0; 1 2 1 2 = 1+ 2 - 1= 2 7 5 2 -1, 2 : Test 0, f′ 0 = 12 0 - 18 0 - 30 = -30 6 0; 5 2 , ∞ : Test 4, f′ 4 = 12 4 - 18 4 - 30 1 2 2 1 2 1 2 1 2 = 192 - 72 - 30 = 90 7 0. 1 2 1 2 1 2 1 2 Interval -1 5 x ++1)++1*(++1)++1*2(++1)++11 Test Value x = -2 x = 0 x = 4

Sign of f′ x f′ -2 7 0 f′ 0 6 0 f′ 4 7 0

Result 1 2 f is1 increasing2 f is decreasing1 2 f is increasing1 2 5 5 on - ∞, -1 on -1, 2 on 2, ∞ 1 2 Change 1 2 Change 1 2 indicates a indicates a relative relative maximum minimum 5 at x = -1. at x = 2.

Therefore, by the First-Derivative Test, f has a relative maximum at x = -1 given by 3 2 f -1 = 4 -1 - 9 -1 - 30 -1 + 25 Substituting -1 for x in the original function 1 2 1 2 1 2 1 2 = 42 This is a relative maximum.

Copyright 2016 Pearson. All rights reserved.

M02_BITT9391_11_SE_C02.1.indd 195 29/10/14 4:39 PM Not for Sale

196 Chapter 2 ● Applications of Differentiation

5 and f has a relative minimum at x = 2 given by This is a relative f 5 4 5 3 9 5 2 30 5 25 175. 2 = 2 - 2 - 2 + = - 4 minimum. 5 175 Thus, there1 2is a relative1 2 maximum1 2 at -1 1,2 42 and a relative minimum at 2, - 4 , as we suspected from the original sketch of the graph. The information obtained from the1 first derivative2 is very useful in graphing1 this2 function. We know that it is continuous and where it is increasing, where it is de- creasing, and where it has relative extrema. We complete the graph by calculating additional function values. The graph of the function, shown below in red, is scaled to clearly show its important features.

Relative maximum f(x) = 4x3 − 9x2 − 30x + 25 Technology connecTIon (±1, 42)y 40 f'(x) = 12x2 − 18x − 30 Exploratory 30

Consider the function f given by 20 3 f x = x - 3x + 2. 10 Graph both f and f′ using the 1 2 x same set of axes. Examine the 24123 22 21 2 3 4 210 graphs using the table and trace features. Where do you think 220 the relative extrema of f occur? Where is the derivative equal to 230 0? Where does f have critical 240 values? (2.5, ± 43.75) Relative minimum

Increasing Decreasing Increasing f'(x) > 0 f'(xf) < 0 '(x) > 0

For reference, the graph of the derivative is shown in blue. Note that f′ x = 0 where f x has relative extrema. We summarize the behavior of this function by noting where it is increasing or decreasing and by characterizing its critical points:1 2 1 2 ● f is increasing over the interval - ∞, -1 . ● f has a relative maximum at the point 1, 42 . 1 -2 Quick Check 1 5 ● f is decreasing over the interval 1, . - 21 2 Graph the function g given by 5 175 3 ● f has a relative minimum at the point , . g x = x - 27x - 6, and find 2 - 4 ● 15 2 any relative extrema. f is increasing over the interval 2, ∞ . 1 1 2 1 2 To use the first derivative for graphing1 a2 function f: 1. Find all critical values by determining where f′ x is 0 and where f′ x is unde- fined (but f x is defined). Find f x for each critical value. 1 2 1 2 2. Use the critical values to divide the x-axis into intervals and choose a test value 1 2 1 2 in each interval. 3. Find the sign of f′ x for each test value chosen in step 2, and use this infor- mation to determine where f x is increasing or decreasing and to classify any extrema as relative 1maxima2 or minima. 1 2 4. Plot some additional points and sketch the graph.

Copyright 2016 Pearson. All rights reserved.

M02_BITT9391_11_SE_C02.1.indd 196 29/10/14 4:39 PM Not for Sale

2.1 ● Using First Derivatives to Classify Maximum and Minimum Values and Sketch Graphs 197

The derivative f′ is used to find the critical values off. The test values are substituted into the derivative f′, and the function values are found using the original function f.

eXAmPLe 2 Find the relative extrema and sketch the graph of the function f given by 3 4 f x = 2x - x . Solution1 2First, we determine the critical values. To do so, we find f′ x : 2 3 f′ x = 6x - 4x . 1 2 Next, we find1 2 where f′ x does not exist or where f′ x = 0. Note that 2 3 f′ x = 6x - 4x is defined for all real numbersx , so the only candidates for critical values are where f′ x 1=20: 1 2 1 2 2 3 6x - 41x 2 = 0 Setting f′ x equal to 0 2 2x 3 - 2x = 0 Factoring1 2 2x2 0 or 3 2x 0 1= 2 - = x2 = 0 or 3 = 2x 3 x = 0 or x = 2. 3 The critical values are 0 and 2. We use these values to divide the x-axis into three 3 3 intervals: - ∞, 0 , 0, 2 , and 2, ∞ . 1 2 1 2 1 2 0, 3 3 , ` (2`, 0) 2)( 2 )(

x 0 3 2

We next find the sign of f′ x on each interval, using a test value. 2 3 - ∞, 0 : Test -1, 1f′2 -1 = 6 -1 - 4 -1 = 6 + 4 = 10 7 0; 0, 3 : Test 1, f′ 1 6 1 2 4 1 3 6 4 2 0; 1 2 2 1 = 2 1- 2 =1 -2 = 7 3 2 3 2, ∞ : Test 2, f′ 2 = 6 2 - 4 2 = 24 - 32 = -8 6 0. 1 2 1 2 1 2 1 2 1 2 1 2 1 2 1 Interval2 0 3 x 2 ++1)++1*(++1)++1*(++1)++1

Test Value x = -1 x = 1 x = 2

Sign of f′ x f′ -1 7 0 f′ 1 7 0 f′ 2 6 0

Result 1 2 f is1 increasing2 f is increasing1 2 f is decreasing1 2 3 3 on - ∞, 0 on 0, 2 on 2 ,∞

1 No2 change1 2 Change 1 2 indicates a relative maximum 3 at x = 2.

Copyright 2016 Pearson. All rights reserved.

M02_BITT9391_11_SE_C02.1.indd 197 29/10/14 4:39 PM Not for Sale

198 Chapter 2 ● Applications of Differentiation

Therefore, by the First-Derivative Test, f has no extremum at x = 0 (since f x is in- 3 3 27 creasing on both sides of 0) and has a relative maximum at x = 2. Thus, f 2 , or 16, is a relative maximum. 1 2 We use the information obtained to sketch the graph below. Other function1 2 val- ues are listed in the table.

Relative x f x , approximately y maximum 3 27 -1 -3 ±, ±Ð 1 2 2 2 16 -0.5 -0.31 m 5 0 0 0 1 (0, 0) 0.5 0.19 1 1 ±1 1 2 x ±1 Not a 1.25 1.46 3 4 relative f(x) = 2x ±x 2 0 ±2 extremum

±3

Increasing Increasing Decreasing f'(x) > 0 f'(x) > 0 f'(x) < 0

● f is increasing over the interval - ∞, 0 . ● f has a critical point at 0, 0 , which is neither a minimum nor a maximum. 1 2 3 ● f is increasing over the interval 0, . 1 2 2 ● 3 27 f has a relative maximum at the point 2, 16 . ● 1 3 2 Quick Check 2 f is decreasing over the interval 2, ∞ . 1 2 3 Find the relative extrema Since f is increasing over the intervals - ∞, 0 and 0, 2 , we can say that f is increas- 3 1 2 of the function h given by ing over - ∞, 2 despite the fact that f′ 0 = 0 within this interval. In this case, we 4 8 3 1 2 h x = x - 3x . Then sketch observe that a secant line connecting any two points1 within2 this interval has a positive the graph. slope. 1 2 1 2 2 1 2 Technology connecTIon eXAmPLe 3 Find the relative extrema and sketch the graph of the function f given by f x x 2 2 3 1. exeRcIses = - + > In Exercises 1 and 2, consider the Solution1 2First,1 we determine2 the critical values. To do so, we find f′ x : function f given by 2 1 3 1 2 2 3 f ′ x = x - 2 - f x = 2 - x - 1 . 3 > > 1. Graph1 2 the function1 using 2 1 2 1 2 2 . Simplifying the viewing window = 3 3 x - 2 -4, 6, -2, 4 . Next, we find where2 f′ x does not exist or where f′ x 0. Note that f′ x does 2. Graph the first derivative. = 3 4 not exist at 2, although f x does. Thus, 2 is a critical value. Since the only way for a What happens to the graph of 1 2 1 2 1 2 the derivative at the critical fraction to be 0 is if its numerator is 0, we see that f′ x = 0 has no solution. Thus, 1 2 values? 2 is the only critical value. We use 2 to divide the x-axis into the intervals - ∞, 2 and 2, ∞ . 1 2 1 2 1 2 (2`, 2) (2, `)

x 2

Copyright 2016 Pearson. All rights reserved.

M02_BITT9391_11_SE_C02.1.indd 198 29/10/14 4:39 PM Not for Sale

2.1 ● Using First Derivatives to Classify Maximum and Minimum Values and Sketch Graphs 199

To determine the sign of f′ x we choose a test value in each interval. It is not necessary to find an exact value of the derivative; we need only determine the sign. Sometimes we can do this by just1 examining2 the formula for the derivative. We choose test values 0 and 3. 2 ∞, 2 : Test 0, f′ 0 0; - = 3 6 3 0 - 2 1 2 1 2 2 2 2, ∞ : Test 3, f′ 3 0. = 3 7 3 3 - 2 1 2 1 2 2 Interval 2 x +11+)++*(+ 11)++1+

Test Value x = 0 x = 3

Sign of f′ x f′ 0 6 0 f′ 3 7 0

Result 1 2 f is decreasing1 2 f is increasing1 2 on - ∞, 2 on 2, ∞ 1 2 1 2 Change indicates a relative minimum at x = 2.

Since we have a change from decreasing to increasing, we conclude from the First- Derivative Test that a relative minimum occurs at 2, f 2 , or 2, 1 . The graph has no tangent line at 2, 1 since f′ 2 does not exist. We use this information and some other function1 values1 22 to sketch1 2 the graph. 1 2 1 2

y x f x , approximately

-1 1 2 3.08 3 f(x) = (x ± 2)2/3 + 1 0 2.59 1 2 2 Relative 2 1 minimum 1 3 2 (2, 1) 4 2.59 ±1 1 2 3 4 x Decreasing Increasing Quick Check 3 f'(x) < 0 f'(x) > 0 Find the relative extrema of the function g given by ● f is decreasing over the interval - ∞, 2 . 1 3 ● f has a relative minimum at the point 2, 1 . g x = 3 - x . Then 1 2 sketch the graph.> ● f is increasing over the interval 2, ∞ . 3 1 2 1 2 1 2

Copyright 2016 Pearson. All rights reserved.

M02_BITT9391_11_SE_C02.1.indd 199 29/10/14 4:39 PM Not for Sale

200 Chapter 2 ● Applications of Differentiation

Technology connecTIon

Finding Relative Extrema The relative maximum seems to be nearly y = 54.61 at an x-value between 9.32 and 9.33. We could next set up a new To explore some methods for approximating relative table showing function values between f 9.32 and f 9.33 to extrema, let’s find the relative extrema of refine the approximation. 1 2 1 2 f x = -0.4x3 + 6.2x2 - 11.3x - 54.8. Method 3: MaxiMuM, MiniMuM We first 1graph2 the function, using a window that reveals the Using the maximum option from the calc menu, we find that a curvature. relative maximum of about 54.61 occurs at x ≈ 9.32.

y = ± 0.43x + 6.2x2 ± 11.3x± 54.8 150

–10 20 Maximum X = 9.3233222 Y = 54.607908

–100 Method 4: fMax or fMin This feature calculates a relative maximum or minimum Method 1: Trace value over any closed interval. We see from the initial graph Using trace, we can move the cursor along the curve, noting that a relative maximum occurs in -10, 20 . Using the fmax where relative extrema might occur. option from the math menu, we see that a relative maximum occurs on -10, 20 when x ≈ 9.32.3 4 Y1=–.4X^3+6.2X^2–11.3X–54.8 3 4 fMax(Y1,X,–10,20) 9.323324165

X = 9.4680851 Y = 54.502163

A relative maximum seems to be about y = 54.5 at x = 9.47. To refine the approximation, we zoom in, press To obtain the maximum value, we evaluate the function at trace, and move the cursor, again noting where the y-value is the given x-value, obtaining the following. largest. The approximation is about y = 54.61 at x = 9.31.

fMax(Y1,X,–10,20) Y1=–.4X^3+6.2X^2–11.3X–54.8 Y1=–.4X^3+6.2X^2–11.3X–54.8 9.323324165 Y1(Ans) 54.60790781

X = 9.4680851 Y = 54.502163 X = 9.3085106 Y = 54.606815

The approximation is about y = 54.61 at x = 9.32. We can repeat to refine the accuracy. Using any of these methods, we find the relative mini- mum y ≈ -60.30 at x ≈ 1.01. Method 2: Table We can also use the table feature, adjusting starting points and step values to improve accuracy:

TblStart = 9.3 ∆Tbl = .01

Minimum X Y1 X = 1.0100107 Y = –60.3005 9.3 54.605 9.31 54.607 9.32 54.608 9.33 54.608 9.34 54.607 exeRcIse 9.35 54.604 1. Using one of the methods described above, approximate 9.36 54.601 X = 9.32 the relative extrema of the function in Example 1.

Copyright 2016 Pearson. All rights reserved.

M02_BITT9391_11_SE_C02.1.indd 200 29/10/14 4:39 PM Not for Sale Exercise Set 2.1 201

Section Summary ● A function f is increasing over an interval I if, for all a over an open interval I if, for all x in I, the slope of the and b in I such that a 6 b, we have f a 6 f b . Equiv- tangent line is negative; that is, f′ x 6 0. alently, the slope of the secant line connecting a and b ● A critical value is a number c in the domain of f such 1 2 1 2 is positive: that f′ c = 0 or f′ c does not exist.1 2 The point f b - f a c, f c is called a critical point. 7 0. ● A relative1 2 maximum1 point2 is higher than all other b a 1 2 - 1 2 points1 1 22 in some interval containing it. Similarly, a rela- ● A function f is decreasing over an interval I if, for all a tive minimum point is lower than all other points in and b in I such that a 6 b, we have f a 7 f b . Equiv- some interval containing it. The y-value of such a point alently, the slope of the secant line connecting a and b is called a relative maximum (or minimum) value of the is negative: 1 2 1 2 function. ● Minimum and maximum points are collectively called f b - f a 6 0. extrema. b a 1 2 - 1 2 ● Critical values are candidates for possible relative ex- ● A function f is increasing over an open interval I if, for trema. The First-Derivative Test is used to classify a crit- all x in I, the slope of the tangent line at x is positive; ical value as a relative minimum, a relative maximum, that is, f′ x 7 0. Similarly, a function f is decreasing or neither. 1 2 2.1 Exercise Set

Find any relative extrema of each function. List each extremum 4x 2 29. g x x along with the x-value at which it occurs. Then sketch a graph of = 2 30. g x = x + 1 x2 + 1 the function. 1 2 3 1 2 1 3 2 2 31. f x = x 32. f x = x + 1 1. f x = x + 6x - 3 2. f x = x + 4x + 5 2 > 2 2 1 2 1 2 1 1 2 3. f1x2 = 2 - 3x - 2x 4. f1x2 = 5 - x - x 33. g x = x2 + 2x + 5 34. F x = x2 + 1 5. F x 0.5x2 2x 11 6. g x 1 6x 3x2 2 1 2 = + - 1 2 = + + 1 2 1 2 2 3 1 2 35–68. Check the results of Exercises 1–34 using a calculator. 7. g1x2 = x + 2x - 2x + 5 1 2 3 2 For Exercises 69–84, draw a graph to match the description 8. G1 x2 = x - x - x + 2 given. Answers will vary. 9. f x x3 3x2 10. f x x3 3x 6 1 2 = - = - + 69. f x is increasing over - ∞, 2 and decreasing over 11. f x x3 3x 12. f x 3x2 2x3 2, ∞ . 1 2 = + 1 2 = + 1 2 1 2 3 3 70. g x is decreasing over ∞, 3 and increasing over 13. F1 x2 = 1 - x 14. g1x2 = 2x - 16 1 2 - - -3, ∞ . 3 2 15. G1 x2 = x - 6x + 10 1 2 1 2 1 2 71. 1G x is 2decreasing over - ∞, 4 and 9, ∞ and in- 2 3 16. f 1x 2 = 12 + 9x - 3x - x creasing over 4, 9 . 1 2 1 2 1 2 3 4 4 3 17. g1x2 = x - x 18. f x = x - 2x 72. F x is increasing1 over2 - ∞, 5 and 12, ∞ and de- 1 3 2 creasing over 5, 12 . 19. f1x2 = 3x - 2x + 4x - 1 1 2 1 2 1 2 1 2 1 73. g x has a positive1 derivative2 over - ∞, -3 and a 20. F x = x3 3x2 9x 2 1 2 - 3 + - + negative derivative over -3, ∞ . 1 2 1 2 21. f x = 3x4 - 15x2 + 12 22. g x = 2x4 - 20x2 + 18 1 2 74. f x has a negative derivative1 over2 - ∞, 1 and a posi- 3 3 23. G x = x + 2 24. F x = x - 1 tive derivative over 1, ∞ . 1 2 1 2 1 2 1 2 2 2 3 2 2 3 25. f 1x 2 = 1 - x 26. f 1x 2 = x + 3 - 5 75. F x has a negative 1derivative2 over - ∞, 2 and 5, 9 > > and a positive derivative over 2, 5 and 9, ∞ . -8 5 1 2 1 2 1 2 27. 1 2 28. 1 2 1 2 G x = 2 F x = 2 1 2 1 2 x + 1 x + 1 1 2 1 Copyright2 2016 Pearson. All rights reserved.

M02_BITT9391_11_SE_C02.1.indd 201 29/10/14 4:39 PM Not for Sale

202 Chapter 2 ● Applications of Differentiation

76. G x has a positive derivative over - ∞, -2 and 4, 7 Using the graph and the intervals noted, explain how and a negative derivative over -2, 4 and 7, ∞ . a function being increasing or decreasing relates to the 1 2 1 2 1 2 first derivative. 77. f x has a positive derivative over1 -2 ∞, 31 and 2 3, 9 , a negative derivative over 9, ∞ , and a derivative equal APPLicAtions to1 02 at x = 3. 1 2 1 2 1 2 Business and Economics 78. g x has a negative derivative over - ∞, 5 and 5, 8 , a positive derivative over 8, ∞ , and a derivative equal 87. employment. According to the U.S. Bureau of Labor to1 02 at x = 5. 1 2 1 2 Statistics, the number of managerial and professional 1 2 employees since 2006 is modeled by 79. F x has a negative derivative over - ∞, -1 and a 3 2 positive derivative over -1, ∞ , and F′ -1 does not E t = 107.833t - 971.369t + 2657.917t + 50,347.833, 1 2 1 2 exist. where t is the number of years since 2006 and E is 1 2 1 2 1 2 thousands of employees. (Source: data.bls.gov.) Find the 80. G x has a positive derivative over - ∞, 0 and 3, ∞ and a negative derivative over 0, 3 , but neither G′ 0 relative extrema of this function, and sketch the graph. nor1 2G′ 3 exists. 1 2 1 2 Interpret the meaning of the relative extrema. 1 2 1 2 88. Advertising. Brody Electronics estimates that it will 81. f x has1 a2 negative derivative over - ∞, -2 and sell N units of a new toy after spending a thousands of 1, ∞ and a positive derivative over -2, 1 , and dollars on advertising, where f1′ 2-2 = 0, but f′ 1 does not exist.1 2 1 2 1 2 N a = -a2 + 300a + 6, 0 … a … 300. 82. g 1x has2 a positive derivative1 2 over - ∞, -3 and 0, 3 , Find the relative extrema and sketch a graph of the function. a negative derivative over -3, 0 and 3, ∞ , and a de- 1 2 1 2 1 2 1 2 rivative equal to 0 at x = -3 and x = 3, but g′ 0 does Life and Physical Sciences not exist. 1 2 1 2 1 2 89. solar eclipse. On July 2, 2019, a total solar eclipse will 83. H x is increasing over - ∞, ∞ , but the derivative occur over the South Pacific Ocean and parts of South does not exist at x = 1. America. (Source: eclipse.gsfc.nasa.gov.) The path of the 1 2 1 2 84. K x is decreasing over - ∞, ∞ , but the derivative eclipse is modeled by does not exist at x = 0 and x = 2. f t = 0.00259t2 - 0.457t + 36.237, 1 2 1 2 where f t is the latitude in degrees south of the equa- 85. Consider this graph. 1 2 tor at t minutes after the start of the total eclipse. What 1 2 y is the latitude closest to the equator, in degrees, at which the total eclipse will be visible? 90. Temperature during an illness. The temperature of a person during an intestinal illness is given by T t = -0.1t2 + 1.2t + 98.6, 0 … t … 12, where T is the temperature F at t days. Find the rela- 1 2 ° tive extrema and sketch a graph of the function. 1 2 x x1 x2 x 3 x4 x5 x6 x7 x8 x9 x10 synthesis In Exercises 91–96, the graph of a derivative f′ is shown. What makes an x-value a critical value? Which x-values Use the information in each graph to determine where f is in- above are critical values, and why? creasing or decreasing and the x-values of any extrema. Then 86. Consider this graph. sketch a possible graph of f. 91. 92. y y y

5 5 4 4 3 f' f' 3 2 2 1 1 −5 −4 −3 −2 −1 1 2 3 4 5 −5 −4 −3 −2 −1 1 2 3 4 5 x −1 x −1 −2 −2 −3 −3 −4 −4 −5 −5 a b c d e x

Copyright 2016 Pearson. All rights reserved.

M02_BITT9391_11_SE_C02.1.indd 202 29/10/14 4:39 PM Not for Sale Exercise Set 2.1 203

93. 94. y y Technology connecTIon

5 5 4 4 Graph each function. Then estimate any relative extrema. 3 3 f' f' 6 5 4 3 2 2 2 97. f x = -x - 4x + 54x + 160x - 641x 1 1 - 828x + 1200 −5 −4 −3 −2 −1 1 2 3 4 5 x −5 −4 −3 −2 −1 1 2 3 4 5 x 1 2 −1 −1 4 3 2 −2 −2 98. f x = x + 4x - 36x - 160x + 400 −3 −3 3 2 −4 −4 99. f1x2 = ͉ 4 - x ͉ + 1 −5 −5 2 2 100. f1x2 = x 9 - x 95. 96. Use a calculator’s2 absolute-value feature to graph each func- y y 1 2 tion and determine relative extrema and intervals over which 5 5 the function is increasing or decreasing. State any x-values at 4 4 3 f' 3 which the derivative does not exist. 2 2 f' 1 1 101. f x = ͉ x - 2 ͉ 102. f x = ͉ 2x - 5 ͉ −5 −4 −3 −2 −1 1 2 3 4 5 x −5 −4 −3 −2 −1 1 2 3 4 5 x −1 −1 103. f x x2 1 104. f x x2 3x 2 −2 −2 1 2 = ͉ - ͉ 1 2 = ͉ - + ͉ −3 −3 2 2 −4 −4 105. f1x2 = ͉ 9 - x ͉ 106. f1x2 = ͉ -x + 4x - 4 ͉ −5 −5 3 4 2 107. f1x2 = ͉ x - 1 ͉ 108. f1x2 = ͉ x - 2x ͉ life science: caloric intake and life expectancy. The data in the following table1 2 give, for various countries 1for2 the period 2007– 2011, daily caloric intake, projected life expectancy, and under-five mortality. Use the data for Exercises 109 and 110.

LiFE ExPECtAnCy UnDEr-FiVE MortALity DAiLy CALoriC At Birth (number of deaths before CoUntry intAkE (in years) age 5 per 1000 births)

Argentina 3030 76 14 Australia 3220 82 5 Bolivia 2100 66 51 Canada 3530 81 6 Dominican Republic 2270 73 25 Germany 3540 80 4 Haiti 1850 62 70 Mexico 3260 77 17 United States 3750 78 8 Venezuela 2650 74 16

(Source: U.N. FAO Statistical Yearbook, 2013.)

109. life expectancy and daily caloric intake. a) Use the regression procedures of Section R.6 to fit a Answers to Quick Checks cubic function y = f x to the data in the table, where 1. Relative maximum at 2. Relative minimum at 16 x is daily caloric intake and y is life expectancy. Then -3, 48 , 2, - 3 fit a quartic function1 and2 decide which fits best. relative minimum at 3, -60 y b) What is the domain of the function? 1 y2 1 4 2 c) Does the function have any relative extrema? 100 1 2 2 50 −1 132 x 110. Under-five mortality and daily caloric intake. −2 −4 −2 24x a) Use the regression procedures of Section R.6 to fit a −50 −4 g(x) 5 x3 2 27x 2 6 cubic function y = f x to the data in the table, where −100 h(x) 5 x 4 2 28 x 3 x is daily caloric intake and y is under-five mortality. 3 Then fit a quartic function1 2 and decide which fits best. 3. There are no extrema. y b) What is the domain of the function? 6 4 c) Does the function have any relative extrema? g (x) 5 3 2 x 1/3 2 111. Describe a procedure that can be used to select an ap- −4 −2 24x propriate viewing window for the functions given in −2 (a) Exercises 1–34 and (b) Exercises 97–100. Copyright 2016 Pearson. All rights reserved.

M02_BITT9391_11_SE_C02.1.indd 203 29/10/14 4:39 PM Not for Sale

204 Chapter 2 ● Applications of Differentiation

Using Second Derivatives to Classify Maximum and Minimum Values and 2.2 Sketch Graphs The graphs of two functions are shown below. The graph of f bends upward and ● Classify the relative extrema of a function using the Second- the graph of g bends downward. Let’s relate these observations to the functions’ Derivative Test. derivatives. Using a straightedge, draw tangent lines as you move along the graph of f from ● Graph a continuous function in a left to right. What happens to the slopes of the tangent lines? Do the same for the manner that shows concavity. graph of g. Look at how the graphs bend. Do you see a pattern?

y y g

f

abx abx

f'(x) increasing g'(x) decreasing f''(x) > 0 g''(x) < 0

Concavity: Increasing and Decreasing Derivatives For the graph of f, the slopes of the tangent lines are increasing. That is, f′is in-

creasing over the interval. This can be determined by noting that f ″ x is positive,

since the relationship between f′ and f ″ is like the relationship between f and f′. For the graph of g, the slopes are decreasing. This can be determined 1by2 noting that g′ is decreasing whenever g″ x is negative. The bending of each curve is called the graph’s concavity. 1 2

Definition Suppose that f is a function whose derivative f′ exists at every point in an open interval I. Then

f is concave up on I if f is concave down on I if f' is increasing over I. f' is decreasing over I.

The following theorem states how the concavity of a function’s graph and the sec- ond derivative of the function are related.

theorem 4 A Test for Concavity

1. If f ″ x 7 0 on an interval I, then the graph of f is concave up on I.

2. If f ″ x 0 on an interval I, then the graph of f is concave down on I. 1 2 6 1 2

Copyright 2016 Pearson. All rights reserved.

M02_BITT9391_11_SE_C02.2.indd 204 29/10/14 4:42 PM Not for Sale

2.2 ● Using Second Derivatives to Classify Maximum and Minimum Values and Sketch Graphs 205

A function can be decreasing and concave up, decreasing and concave down, TeChnology ConneCTIon increasing and concave up, or increasing and concave down. That is, concavity and Exploratory increasing/decreasing are separate concepts. It is the increasing or decreasing aspect of the derivative that tells us about the function’s concavity. Graph 1 3 2 f x = - 3x + 6x - 11x - 50 f′ x 6 0 f′ x 7 0 and1 2 its second derivative, f″ x 6 0 f″ x = -2x + 12, 1 2 1 2 1 2 using the1 2 viewing window -10, 25, -100, 150 , with Xscl = 5 and Yscl = 25. 3 Over what intervals4 is the Concave down, Concave down, graph of f concave up? Over Decreasing Increasing what intervals is the graph of f concave down? Over what f″ x 7 0 intervals is the graph of f″ 1 2 positive? Over what intervals is the graph of f″ negative? How do you think the sign of f″ x relates Concave up, Concave up, to the concavity of f? Decreasing Increasing Now graph the first1 2 derivative, 2 f′ x = -x + 12x - 11, Classifying Relative extrema Using and the1 second2 derivative, Second Derivatives f″ x = -2x + 12, Let’s see how we can use second derivatives to determine whether a function has a using the1 2 window -10, 25, relative extremum on an open interval. 200, 50 , with Xscl 5 and - = The following graphs show both types of concavity at a critical value, where Yscl = 25. 3 Over4 what intervals is the f′ c = 0. When the second derivative is positive (graph is concave up) at the critical first derivative increasing? value, the critical point is a relative minimum point, and when the second derivative f′ 1 2 Over what intervals is the first is negative (graph is concave down), the critical point is a relative maximum point. derivative f′ decreasing? Over what intervals is the graph of f″ y y positive? Over what intervals is the graph of f″ negative? How do you think the sign of f″ x relates to the concavity of f? 1 2 f(c)

f(c)

a c b x a c b x

f'(c) = 0, f'(c) = 0, f''(c) > 0, f''(c) < 0, The graph is concave The graph is concave up at c. down at c. Therefore, f(c) is a Therefore, f(c) is a relative minimum. relative maximum.

Copyright 2016 Pearson. All rights reserved.

M02_BITT9391_11_SE_C02.2.indd 205 29/10/14 4:42 PM Not for Sale

206 Chapter 2 ● Applications of Differentiation

This analysis is summarized in Theorem 5:

theorem 5 The Second-Derivative Test for Relative Extrema Suppose that f is differentiable for every x in an open interval a, b and that there is a critical value c in a, b for which f′ c = 0. Then: 1 2 1. f c is a relative minimum1 if2 f″ c 7 0. 1 2 2. f c is a relative maximum if f″ c 0. 1 2 1 2 6 For f″1 c2 = 0, the First-Derivative Test1 2 can be used to determine whether f x is a relative extremum. 1 2 1 2

When c is a critical value and f″ c = 0, an extremum may or may not exist at c. Consider the following graphs. In each one, f′ and f″ are both 0 at c = 2, but the first function has an extremum and the second1 2 function does not. An approach other than the Second-Derivative Test must be used to determine whether f c is an extremum in these cases. 1 2 y y 10 10 4 9 f(x) = (x ± 2) + 1 9 3 8 f(x) = (x ± 2)4 + 1 8 f(x) = (x ± 2) + 1 f(x) = (x ± 2)3 + 1 7 f'(x) = 4(x ± 2)3 7 f'(x) = 3(x ± 2)2 6 f''(x) = 12(x ± 2)2 6 f''(x) = 6(x ± 2) 5 f'(2) = 0 5 f'(2) = 0 4 f''(2) = 0 4 f''(2) = 0 3 Relative minimum at c = 2 3 No relative extremum 2 2 1 1

±1 1 c = 2 3 4 x ±1 1 c = 2 3 4 x ±1 ±1

The second derivative is used to help identify extrema and determine the overall behavior of a graph, as we see in the following examples.

eXAmPLe 1 Find any relative extrema and graph the function f given by 3 2 f x = x + 3x - 9x - 13. Solution1 2We find both the first and second derivatives: 2 f ′ x = 3x + 6x - 9, f ″ x 6x 6. 1 2 = + To find any1 2critical values, we solve f′ x = 0: 2 3x + 6x - 9 = 0 1 2 2 x + 2x - 3 = 0 Dividing both sides by 3 x + 3 x - 1 = 0 Factoring x 3 0 or x 1 0 Using the Principle of Zero Products 1 + 21= 2 - = x = -3 or x = 1. We next find second coordinates by substituting the critical values in the original function: 3 2 f -3 = -3 + 3 -3 - 9 -3 - 13 = 14; f 1 1 3 3 1 2 9 1 13 18. 1 2 = 1 2+ 1 -2 1- 2 = - 1 2 1 2 1 2 1 2 Copyright 2016 Pearson. All rights reserved.

M02_BITT9391_11_SE_C02.2.indd 206 29/10/14 4:42 PM Not for Sale

2.2 ● Using Second Derivatives to Classify Maximum and Minimum Values and Sketch Graphs 207

We now use the Second-Derivative Test with the critical values -3 and 1: f ″ -3 = 6 -3 + 6 = -12 6 0; Relative maximum at x = -3 f ″ 1 6 1 6 12 0. Relative minimum at x 1 1 2 = 1 2+ = 7 = Thus, f -3 1 =2 14 is1 a 2relative maximum and f 1 = -18 is a relative minimum. We plot both -3, 14 and 1, -18 , including short arcs at each point to indicate the concavity.1 By2 plotting more points, we can make1 a2 sketch, as shown below. 1 2 1 2

y y 40 40 f Relative Relative 30 30 maximum maximum

20 20 (−3, 14) (−3, 14) (3, 14) ( 2, 9) 10 (−4, 7) − 10

−5 −4 −3 −2 −1 1 2 3 4 x −5 −4 −3 −2 1 2 3 4 x (2, −11) −10 (−1, −2) (0, −13) −20 (1, −18) (−5, −18) −20 (1, −18) Relative Relative minimum minimum

eXAmPLe 2 Find any relative extrema and graph the function f given by 5 3 f x = 3x - 20x . Solution1 2We find both the first and second derivatives: 4 2 f ′ x = 15x - 60x , f ″ x 60x3 120x. 1 2 = - Then we solve1 2 f′ x = 0 to find any critical values: 4 2 151 x2 - 60x = 0 2 2 15x x - 4 = 0 15x2 x 2 x 2 0 Factoring + 1 - 2 = 15x2 0 or x 2 0 or x 2 0 Using the Principle of 1= 21 +2 = - = Zero Products x = 0 or x = -2 or x = 2. We next find second coordinates by substituting in the original function: 5 3 f -2 = 3 -2 - 20 -2 = 64; f 0 3 0 5 20 0 3 0. 1 2 = 1 2- 1 =2 f 2 3 2 5 20 2 3 64; 1 2 = 1 2 - 1 2 = - All three of 1these2 y-values1 2 are candidates1 2 for relative extrema. We now use the Second-Derivative Test with the values -2, 0, and 2: 3 f ″ -2 = 60 -2 - 120 -2 = -240 6 0; Relative maximum at x = -2 f ″ 0 60 0 3 120 0 0. The Second-Derivative Test 1 2 = 1 2- 1 =2 fails. Use the First-Derivative 1 2 1 2 1 2 Test. 3 f ″ 2 = 60 2 - 120 2 = 240 7 0; Relative minimum at x = 2 1 2 1 2 1 2 Copyright 2016 Pearson. All rights reserved.

M02_BITT9391_11_SE_C02.2.indd 207 29/10/14 4:42 PM Not for Sale

208 Chapter 2 ● Applications of Differentiation

Thus, f 2 64 is a relative - = Relative maximum y maximum, and f 2 64 = - 80 f(x) = 3x5 − 20x3 is a relative1 2 minimum. Since (−2, 64) 60 f′ -1 6 0 and 1f′21 6 0, we know that f is decreasing 40 1 2 1 2 20 on -2, 0 and 0, 2 . Thus, (0, 0) by the First-Derivative Test, −3 −2 −1 1 2 3 x f has1 no relative2 1 extremum2 −20 Relative at 0, 0 . We complete the −40 minimum graph, plotting other points −60 as needed.1 2 The extrema are 80 (2, −64) Quick Check − 1 shown in the graph at right. Find any relative extrema and Increasing Decreasing Increasing graph the function g given by 3 5 g x = 10x - 6x . 1 1 2 f(x) = x3 + 3x2 − 9x − 13 Points of Inflection y

30 f Concavity changes from down to up at -1, -2 in the graph in Example 1 (repeated at the left) and from up to down at 0, 0 in the graph in Example 2 (above). Such (−3, 14) 20 (−1, −2) 1 2 10 points are called points of inflection, or inflection points. At each such point, the di- rection of concavity changes. In the figures1 2 below, pointP is an inflection point. The −4 −3 −2 1 2 3 x −10 figures display the sign of f ″ x to indicate the concavity on either side of P. −20 (1, −18) y 1 2y y Concave Concave down up ± ± P P ± + P + +

x0 x x0 x x0 x '' = '' '' = f (x0) 0 f (x0) does not exist. f (x0) 0 As we move to the right along each curve, the concavity changes at P. Since, as we move through P, the sign of f ″ x changes, either the value of f ″ x0 at P must be 0 or f ″ x0 must not exist. 1 2 1 2 1 2 theorem 6 Finding Points of Inflection

If f has a point of inflection, it must occur at a value x0, in the domain of f, where

f ″ x0 = 0 or f ″ x0 does not exist. 1 2 1 2

The converse of Theorem 6 is not necessarily true. That is, if f ″ x0 is 0 or does not exist, then there is not necessarily a point of inflection at x0. There must be a 1 2 change in the direction of concavity on either side of x0 for x0, f x0 to be a point of inflection. For example, for f x = x - 2 4 + 1 (see the graph on p. 206), we have f ″ 2 = 0, but 2, f 2 is not a point of inflection since the1 graph1 22of f is concave up on both sides of x = 2. 1 2 1 2 1 To2 find candidates1 1 22for points of inflection, we look for x-values at which f ″ x = 0 or f ″ x does not exist. If f ″ x changes sign on either side of the x-value, we have a point of inflection. 1 2 1 2 1 2

Copyright 2016 Pearson. All rights reserved.

M02_BITT9391_11_SE_C02.2.indd 208 29/10/14 4:42 PM Not for Sale

2.2 ● Using Second Derivatives to Classify Maximum and Minimum Values and Sketch Graphs 209

Theorem 6 is analogous to Theorem 2. Whereas Theorem 2 tells us that can- didates for extrema occur when f′ x = 0 or f′ x does not exist, Theorem 6 tells us that candidates for points of inflection occur when f ″ x = 0 or f ″ x does not exist. 1 2 1 2 1 2 1 2 eXAmPLe 3 Use the second derivative to determine any point(s) of inflection for the function in Example 2. 5 3 Solution The function is given by f x = 3x - 20x , and its second derivative is 3 f ″ x = 60x - 120x. We set f ″ x equal to 0 and solve for x: 1 2 3 1 2 60x - 1201 x2 = 0 2 60x x - 2 = 0 Factoring out 60x 60x x 2 x 2 0 Factoring x2 2 as a difference of squares + 1- 2 = - x = 0 or2 x = - 22 or x = 2. Using the Principle of Zero Products 1 2 1 2 Next, we check the sign of f ″2x over the intervals2 bounded by these three x-values. We are looking for a change in sign from one interval to the next: 1 2

Interval x - 2 0 2 1++++)++*2(+++)++* (+++)++*2(+++)+++ Test Value x = -2 x = -1 x = 1 x = 2 Sign of f ″ x f ″ -2 6 0 f ″ -1 7 0 f ″ 1 6 0 f ″ 2 7 0 Result 1 2 Concave1 2 down Concave1 2 up Concave1 2 down Concave1 2 up on on - ∞, - 2 on - 2, 0 on 0, 2 2, ∞ 2 2 2 2 1 2 1 2 1 2 1 2 The graph changes from concave down to concave up at x = - 2, from concave up to concave down at x = 0, and from concave down to concave2 up at x = 2. Therefore, - 2, f - 2 , 0, f 0 , and 2, f 2 are points of inflec2 - tion. Since 2 2 2 2 1 1 22 5 3 f -1 2 =1 3 - 22 - 20 - 21 = 281 2,2 2 5 3 2f 0 = 3 0 2- 20 0 = 02, 2 and 1f 22 31 2 52 20 1 2 3 2 28 2, 1 2 = 1 2 -1 2 = - 2 2 2 2 these points 1are 2- 2, 128 2 , 0, 01, and2 2, -28 2 , as shown below. 2 2 2 2 1 2 1 2 y 1 2 (−√2, 28√2) 80 60 f 40 20 (0, 0) −3 −2 −1 123 x −20 −40 Quick Check 2 −60 80 Determine the points of inflec- − (√2, −28√2) tion for the function given by Concave Concave Concave Concave 3 5 g x = 10x - 6x . down up down up 2 1 2

Copyright 2016 Pearson. All rights reserved.

M02_BITT9391_11_SE_C02.2.indd 209 29/10/14 4:42 PM Not for Sale

210 Chapter 2 ● Applications of Differentiation

Curve Sketching The first and second derivatives enhance our ability to sketch curves. We use the fol- lowing strategy:

Strategy for Sketching Graphs* a) Derivatives and domain. Find f′ x and f ″ x . Note the domain of f. b) Critical values of f. Find the critical values by solving f′ x = 0 and finding where f′ x does not exist. These1 2 numbers1 yield2 candidates for relative maxima or minima. Find the function values at these points.1 2 c) Increasing1 and/or2 decreasing; relative extrema. Substitute each critical value, x0, from step (b) into f ″ x . If f ″ x0 6 0, then f x0 is a relative maximum and f is increasing on the left of x0 and decreasing on the right. If 1 2 1 2 1 2 f ″ x0 7 0, then f x0 is a relative minimum and f is decreasing on the left of x0 and increasing on the right. d) Inflection1 2 points. Determine1 2 candidates for inflection points by finding where f ″ x = 0 or where f ″ x does not exist. Find the function values at these points. e) Concavity.1 2 Use the candidates1 2 for inflection points from step (d) to define intervals. Substitute test values into f ″ x to determine where the graph is concave up f ″ x 7 0 and where it is concave down f ″ x 6 0 . f) Sketch the graph. Sketch the graph using1 the2 information from steps (a)–(e), plotting extra1 points1 2 as2 needed. 1 1 2 2

The examples that follow apply this step-by-step strategy.

eXAmPLe 4 Find any relative extrema and graph the function f given by 3 f x = x - 3x + 2. Solution1 2 a) Derivatives and domain. Find f′ x and f ″ x : 2 f ′ x = 3x - 3, 1 2 1 2 f ″ x 6x. 1 2 = Like any1 polynomial2 function, f, f′, and f ″ have as their domain - ∞, ∞ , or ℝ. y b) Critical values of f. Find the critical values by determining where f′ x does not 6 2 1 2 exist and by solving f′ x = 0. We know that f′ x = 3x - 3 exists for all values (−1, 4) 5 4 of x, so the only critical values are where f′ x is 0: 1 2 3 1 2 1 2 2 2 1 (1, 0) 3x - 3 = 0 Setting f′ x equal 1to2 0 −6−5−4−3−2−1 4321 5 6 2 −1 x 3x = 3 1 2 −2 2 −3 x = 1 −4 −5 x = {1. Using the Principle of Square Roots −6 We have f -1 = 4 and f 1 = 0, so we plot -1, 4 and 1, 0 . 1 2 1 2 1 2 1 2

*This strategy is refined further, for rational functions, in Section 2.3.

Copyright 2016 Pearson. All rights reserved.

M02_BITT9391_11_SE_C02.2.indd 210 29/10/14 4:42 PM Not for Sale

2.2 ● Using Second Derivatives to Classify Maximum and Minimum Values and Sketch Graphs 211

y c) Increasing and/or decreasing; relative extrema. Substitute the critical values into f ″ x : 6 (−1, 4) 5 f ″ 1 6 1 6 0, 4 - = - = - 6 1 2 3 2 so f -11 = 24 is a relative1 2 maximum, with f increasing on - ∞, -1 and decreasing 1 (1, 0) on -1, 1 . 1 2 1 2 −6−5−4−3−2−−11 4321 5 6 x −2 1 f ″ 12 = 6 # 1 = 6 7 0, −3 −4 so f 1 1= 20 is a relative minimum, with f decreasing on -1, 1 and increasing on −5 1, ∞ . −6 1 2 1 2 d) Inflection points. Find possible inflection points by finding where f ″ x does not 1 2 exist and by solving f ″ x = 0. We know that f ″ x = 6x exists for all values of x, y 1 2 6 so we solve f ″ x = 0: (−1, 4) 5 1 2 1 2 4 6x = 0 1 2 Setting f ″ x equal to 0 3 (0, 2) 2 x = 0. Dividing both1 2 sides by 6 1 (1, 0) We have f 0 = 2, which gives us another point, 0, 2 , on the graph. −6−5−4−3−2−−11 4321 5 6 x −2 e) Concavity. Find the intervals on which f is concave up or concave down. From step −3 1 2 1 2 −4 (c), we can conclude that f is concave down over the interval - ∞, 0 and concave −5 up over 0, ∞ . −6 1 2 1 2 y Interval x 6 0 (−1, 4) 5 Point of ++++)+++* (++++)+++ 4 inection, 3 (0, 2) Test Value x 1 x 1 2 = - = 1 (1, 0) Sign of f ″ x f ″ -1 6 0 f ″ 1 7 0 −6−5−4−3−2−−11 4321 5 6 x −2 1 2 f′ is1 decreasing;2 f′ is 1increasing;2 −3 Result −4 f is concave down f is concave up −5 on - ∞, 0 on 0, ∞ −6 1 Change2 indicates a 1 2 point of inflection at x = 0.

f) Sketch the graph. Sketch the graph using the information in steps (a)–(e), plotting some extra values if desired. The graph follows.

y Relative x f x 6 maximum f(x) = x3 ± 3 +x 2 -3 -1162 at (±1, 4) 5 TeChnology ConneCTIon -2 0 4 -1 4 3 Point of Check the results of Example 4 in ection 2 using a calculator. 0 2 at (0, 2) 1 0 1

2 4 ±61±5 ±4 ±3 ±2 ±1 23456x 3 20 ±1 ±2 Relative minimum ±3 at (1, 0) ±4 ±5 ±6

Increasing Decreasing Increasing

Concave down Concave up

Copyright 2016 Pearson. All rights reserved.

M02_BITT9391_11_SE_C02.2.indd 211 29/10/14 4:42 PM Not for Sale

212 Chapter 2 ● Applications of Differentiation

eXAmPLe 5 Find any relative extrema and graph the function f given by 4 2 f x = x - 2x . Solution1 2 a) Derivatives and domain. Find f′ x and f ″ x : 3 f ′ x = 4x - 4x, 1 2 1 2 f ″ x 12x2 4. 1 2 = - The domain1 2 of f, f′, and f ″ is ℝ. y b) Critical values. Since the domain of f′ is ℝ, the only critical values are where f′ x = 0: 3 2 4x - 4x = 0 Setting f′ x equal to 0 1 2 4x x2 1 0 1 - = 1 2 (0, 0) 4x 0 or x 2 1 0 1 = 2 - = −2 −1 21 x x = 0 or x 2 = 1 −1 (−1, −1) (1, −1) x = {1. Using the Principle of Square Roots −2 We have f 0 = 0, f -1 = -1, and f 1 = -1, so 0, 0 , -1, -1 , and 1, -1 are on the graph. 1 2 1 2 1 2 1 2 1 2 1 2 c) Increasing and/or decreasing; relative extrema. Substitute the critical values into f ″ x . For x = -1, we have 2 y 1 2 f ″ -1 = 12 -1 - 4 = 8 7 0,

2 so f -11 = 2-1 is a 1relative2 minimum, with f decreasing on - ∞, -1 and increasing on -1, 0 . For x = 0, we have 1 1 2 1 2 2 (0, 0) 1 f ″ 02 = 12 # 0 - 4 = -4 6 0, −2 −1 21 x so f 0 1= 20 is a relative maximum, with f increasing on -1, 0 and decreasing on −1 0, 1 . For x = 1, we have (−1, −1) (1, −1) 1 2 1 2 −2 2 1 2f ″ 1 = 12 # 1 - 4 = 8 7 0, so f 1 1= 2-1 is also a relative minimum, with f decreasing on 0, 1 and increasing on 1, ∞ . 1 2 1 2 d) Inflection points. Find where f ″ x does not exist and where f ″ x 0. Since the 1 2 = domain of f ″ is ℝ, we solve f ″ x = 0: TeChnology ConneCTIon 1 2 1 2 2 12x - 4 = 0 1 2Setting f ″ x equal to 0 exeRCISe 2 4 3x - 1 = 0 1 2 1. Consider 2 3x - 1 = 0 Dividing both sides by 4 f x x3 x 2 3. 1 2 = - 3x2 = 1 How1 many2 relative1 extrema2 1 do you anticipate finding? x 2 = Where do you think they 3 will be? 1 x = { . Using the Principle of Square Roots Graph f, f′, and f ″ using 3 -1, 3, -2, 6 as a viewing window. Estimate any rela- We have 1 tive3 extrema and4 any inflec- 1 1 4 1 2 tion points of f. Then check f = - 2 your work using the method 3 3 3 of Examples 4 and 5. a 2 b a12 2b 5a 2 b = - = - 9 3 9

Copyright 2016 Pearson. All rights reserved.

M02_BITT9391_11_SE_C02.2.indd 212 29/10/14 4:42 PM Not for Sale

2.2 ● Using Second Derivatives to Classify Maximum and Minimum Values and Sketch Graphs 213

y and 1 5 2 f - = - . 3 9 − 1 , − 5 1 1 , − 5 ()3 9 ()3 9 a 11 b 5 1 5 Thus, - , - and , - are possible inflection points. −2 −1 21 x 3 9 3 9 −1 e) Concavity.a 2 Find the intervalsb a 2 on which bf is concave up or concave down. From step −2 (c), we can conclude that f is concave up over the intervals - ∞, -1 3 and 1 3, ∞ and concave down over the interval -1 3, 1 3 . 2 2 2 1 2 > 2 >Interval > > 1 2 1 2 x -1 3 1 3 +++++)+++*2(+++++)++++*2(+++)+++11 y > > Test Value x = -1 x = 0 x = 1 2 Point of Point of Sign of f ″ x f ″ -1 7 0 f ″ 0 6 0 f ″ 1 7 0 1 inection inection Result 1 2 f′ is1 increasing;2 f′ is decreasing;1 2 f′ is 1increasing;2 −2 −1 21 x f is concave up f is concave down f is concave up −1 on - ∞, -1 3 on -1 3, 1 3 on 1 3, ∞ 2 2 2 2 −2 1 > 2 1 > > 2 1 > 2 Change indicates Change indicates a point of inflection a point of inflection at x = -1 3. at x = 1 3. 2 1 f) Sketch the graph. Sketch the graph using> the information in steps> (a)–(e). By solving 4 2 x - 2x = 0, we can find thex -intercepts easily. They are - 2, 0 , 0, 0 , and 2, 0 . Other function values can also be calculated. 2 1 2 2 y 1 2 1 2 2 f(x) = x4 – 2x2

1 (– 2, 0 ) (0, 0) ( 2, 0 )

–2 –1 1 2 x 1 1 5 , –– 5– , – – 3 9 –1 3 9 (–1, –1) (1, –1) Quick Check 3 Decreasing Increasing Decreasing Increasing Find the relative maxima and minima of the function f given by Concave up Concave Concave up down f x = 1 + 8x2 - x4, and sketch the graph. 3 1 2 EXAMPLE 6 Graph the function f given by f x = 2x - 5 1 3 + 1. > List the coordinates1 2 1 of any2 extrema and points of inflection. State where the function is increasing or decreasing, as well as where it is concave up or concave down. Solution a) Derivatives and domain. Find f′ x and f ″ x : 2 f x 1 2x 5 -2 3 21 2 2 2x 1 52 -2 3, or ; ′ = 3 - # = 3 - 2 3 > > 3 2x - 5 1 2 1 2 1 2 -8> f x 4 2x 5 -5 3 2 8 2x 5 -5 3, or1 2 . ″ = - 9 - # = - 9 - 5 3 > > 9 2x - 5 1 2 1 2 1 2 > Copyright 2016 Pearson. All rights reserved. 1 2

M02_BITT9391_11_SE_C02.2.indd 213 29/10/14 8:36 PM Not for Sale

214 Chapter 2 ● Applications of Differentiation

5 5 The domain of f is ℝ. However, the domain of f′ and f ″ is - ∞, 2 ∪ 2 , ∞ 5 since their denominators equal zero when x = 2. b) Critical values. Since 1 2 1 2 2 f′ x = 2 3 3 2x - 5 1 2 > is never 0 (a rational1 expression2 equals 0 only when its numerator is 0), the only critical value is when f′ x does not exist. The only instance where f′ x does not y exist is when its denominator is 0: 1 2 1 2 3 2 5 2 3 0 Setting the denominator equal to zero 3 x - = 2x 5 2>3 0 Dividing both sides by 3 1 - 2 = 2 2 2x - 5> = 0 Cubing both sides 5 1 2 1 , 1 2 )( 2x 5 0 Using the Principle of Square Roots 1 -2 = −1 1 23 x 2x = 5 5 −1 x = 2 5 5 1 3 5 We now have f 2 = 2 # 2 - 5 + 1 = 0 + 1 = 1, so we plot the point 2, 1 . 5 c) Increasing and/or decreasing; relative> extrema. Since f ″ 2 does not exist, the 5 Second-Derivative1 2 Test1 cannot be2 used at x = 2. Instead, we use the First-Derivative1 2 5 Test, selecting 2 and 3 as test values on either side of 2: 1 2 2 2 2 2 f ′ 2 0, = 2 3 = 2 3 = = 7 3 2 # 2 - 5 3 -1 3 # 1 3 1 2 2 > 2 > 2 2 and f ′ 3 1 2 1 2 0. = 2 3 = 2 3 = = 7 3 2 # 3 - 5 3 # 1 3 # 1 3 1 2 > 5 > 5 Since f′ x 7 01 on each side2 of x = 2, we know that f is increasing on both - ∞, 2 5 5 and , ∞ ; thus, f = 1 is not an extremum. 2 1 2 2 d) Inflection points. Find where f ″ x does not exist and where f ″ x = 0. Since1 f ″ x2 is never1 02 (why?), we1 2 only need to find where f ″ x does not exist. Since f ″ x 1 2 1 2 5 1 2 cannot exist for 2x - 5 = 0, there is a possible inflection point at 2, 1 . 1 2 5 1 2 y e) Concavity. We check the concavity on each side of x = 2. We choose x = 2 and x = 3 as our test values: 1 2 3 Point of -8 -8 f ″ 2 0, 2 inection = 5 3 = 7 9 2 # 2 - 5 9 -1 1 5 , 1 2 )( 1 2 5> so f is concave up1 on - ∞2, 2 . And1 2 −1 1 23 x -8 -8 f ″ 3 0, −1 = 1 5 23 = 6 9 2 # 3 - 5 9 # 1 1 2 5 > so f is concave down1 on 22, ∞ .

Interval 1 2 5 x +++++)+++++* 2 (++++++)++++++ Test Value x = 2 x = 3 Sign of f ″ x f ″ 2 7 0 f ″ 3 6 0 Result 1 2 f′ is increasing;1 2 f is f′ is decreasing;1 2 f is concave 5 5 concave up on - ∞, down on , ∞ 2 2 a b a b Change indicates a point of inflection at x = 5 2. Copyright 2016 Pearson. All rights reserved. >

M02_BITT9391_11_SE_C02.2.indd 214 29/10/14 4:42 PM Not for Sale

2.2 ● Using Second Derivatives to Classify Maximum and Minimum Values and Sketch Graphs 215

f) Sketch the graph. Sketch the graph using the information in steps (a)–(e). By solving 1 3 2x - 5 + 1 = 0, we can find thex -intercept, 2, 0 . Other function values can also be> calculated. 1 2 1 2

y f x , 1/3 x approximately 3 f(x) = (2x − 5) + 1 1 2 0 -0.71 2 (3, 2) 1 -0.44 5 , 1 1 2 )( 2 0 (2, 0) 5 1 2 −2 −1 1 2345 x 3 2 Quick Check 4 −1 4 2.44 Find any relative extrema and 5 2.71 graph the function f given by Increasing 1 3 f x = 1 - x - 2 . Concave up Concave down 4 > 1 2 1 2 The following figures illustrate some information concerning a function f that can be found from its first and second derivatives. Note that the x-coordinates of the x-intercepts of f′ are the critical values of f. Note that the intervals over which f is increasing or decreasing are those intervals for which f′ is positive or negative, respectively.

y f'(x) = 3x2 + 6x − 9 40 f Relative 30 y maximum 40 20 (−3, 14) Relative 30 maximum 10

20 (−3, 14) −5 −4 −3 −2 −1 1 2 3 4 x Concave 10 −10 down

−5 −4 −3 −2 −1 1 2 3 4 x −20 (1, −18) Concave −10 up Relative minimum f'(x) > 0 f'(x) < 0 f'(x) > 0 −20 (1, −18) f is f is f is increasing decreasing increasing f(x) = x3 + 3x2 − 9x − 13 Relative minimum f'(−3) = 0 f'(1) = 0

In the figures below, note that the intervals over which f′ is increasing or de- creasing are, respectively, those intervals over which f ″ is positive or negative. And fi- nally, note that when f ″ x 6 0, the graph of f is concave down, and when f ″ x 7 0, the graph of f is concave up. 1 2 1 2

Copyright 2016 Pearson. All rights reserved.

M02_BITT9391_11_SE_C02.2.indd 215 29/10/14 8:35 PM Not for Sale

216 Chapter 2 ● Applications of Differentiation

y y 40 40 f f' 30 30 f" 20 20 10 10

−5 −4 −3 −2 1 2 3 4 x −5 −4 −3 −2 −1 1 2 3 4 x (−1, −2) −10 f"(x) = 6x + 6 −10 −20 −20

f"(x) < 0 f"(x) > 0 f' is decreasing f' is increasing f is concave down f is concave up

f"(−1) = 0 f"(−1) = 0

TeChnology ConneCTIon Using Graphicus to Find Roots, Extrema, and Inflection Points and to Graph Derivatives After opening Graphicus, touch the blank rectangle at the top of the screen and enter y(x)=x^4-2x^2. Press + in the upper Graphicus has the capability of finding roots, rela- right. You will see the graph (Fig. 1). Then touch the fourth tive extrema, and points of inflection. Let’s consider icon from the left at the bottom, and you will see the roots 4 2 f x = x - 2x , from Example 5. highlighted on the graph (Fig. 2). Touch the left-hand root 1 2

FiGUre 1 FiGUre 2 FiGUre 3

FiGUre 4 FiGUre 5 FiGUre 6 FiGUre 7 (continued) Copyright 2016 Pearson. All rights reserved.

M02_BITT9391_11_SE_C02.2.indd 216 29/10/14 4:43 PM Not for Sale Exercise Set 2.2 217

symbol, and a value for one root, in this case, an approxima- exeRCISeS tion, -1.414214, is displayed (Fig. 3). Use Graphicus to graph each function and its first and second To find the relative extrema, touch the fifth icon from derivatives. Then find approximations for roots, relative ex- the left; the relative extrema will be highlighted on the graph trema, and points of inflection. in a different color. Touch each point to identify the relative 3 4 minima at -1, -1 and 1, -1 and a relative maximum at 1. f x = 2x - x 2. f x = x 200 - x 0, 0 (Fig. 4). 3 2 3. f x = x - 6x To find1 points of2 inflection,1 2 touch the sixth icon from 1 2 1 2 1 2 1 2 2 3 the left, and the points of inflection will be highlighted as 4. f1x2 = -4.32 + 1.44x + 3x - x shown in Fig. 5. Touch these points and the approximations 2 -0.577, -0.556 and 0.577, -0.556 are displayed. 5. g1x2 = x 4 - x Graphicus can graph derivatives. Go back to the origi- 2 2 1 2 4x x - 3x nal1 graph of f (Fig.2 1). Touch1 + ; then press2 Add derivative, and 6. g x = 7. f x = 2 x 1 you will see the original graph and the graph of the first x + 1 - 1 2 1 2 5 3 derivative as a dashed line of a different color (Fig. 6). Touch 8. f x = ͉ x + 2 ͉ - 3 9. f x = 3x - 5x + and Add derivative again, and you will see the graph of the original function, with dashed graphs of the first and second 1 2 1 2 derivatives in different colors (Fig. 7). You can toggle be- tween the derivatives by pressing the colored squares above the graphs.

Section Summary ● The second derivative f ″ is used to determine the con- ● If c is a critical value and f ″ c = 0, the First- cavity of the graph of f. Derivative Test must be used to determine if f c is a ● If f ″ x 7 0 for all x in an open interval I, then the relative extremum. 1 2 1 2 graph of f is concave up over I. ● If f ″ x0 = 0 or f ″ x0 does not exist, and there is a ● 1 2 If f ″ x 6 0 for all x in an open interval I, then the change in concavity on the left and on the right of x0, 1 2 1 2 graph of f is concave down over I. then the point x0, f x0 is called a point of inflection. ● If c is1 a2 critical value and f ″ c 7 0, then f c is a rela- ● Finding relative extrema, intervals over which a func- tive minimum. tion is increasing1 or1 decreasing,22 intervals of upward or ● If c is a critical value and f ″1c2 6 0, then f1c2 is a rela- downward concavity, and points of inflection form a tive maximum. strategy for accurate curve sketching. 1 2 1 2

2.2 exercise Set

For each function, find all relative extrema and classify each Sketch the graph of each function. List the coordinates of any as a maximum or minimum. Use the Second-Derivative Test extrema or points of inflection. State where the function is where possible. increasing or decreasing and where its graph is concave up or concave down. 1. f x = 4 - x2 2. f x = 5 - x2 3 3 2 2 9. f x = x - 27x 10. f x = x - 12x 3. f1x2 = x + x - 1 4. f1x2 = x - x 3 2 2 2 11. f1x2 = 2x - 3x - 36x + 28 1 2 5. f1x2 = -4x + 3x - 1 6. f1x2 = -5x + 8x - 7 3 2 3 3 12. f1x2 = 3x - 36x - 3 13. f x = 80 - 9x - x 7. f1x2 = x - 12x - 1 1 2 8 3 1 3 3 14. f1x2 = 3x - 2x + 3 15. f1x2 = -x + 3x - 2 8. f1x2 = 8x - 6x + 1 1 2 1 2 1 2

Copyright 2016 Pearson. All rights reserved.

M02_BITT9391_11_SE_C02.2.indd 217 29/10/14 4:43 PM Not for Sale

218 Chapter 2 ● Applications of Differentiation

3 2 16. f x = -x + 3x - 4 53. f′ -3 = 0, f ″ -3 6 0, f -3 = 8; f′ 9 = 0, f ″ 9 0, f 9 6; f ″ 2 0, and f 2 1 4 3 2 7 = - = = 17. f1x2 = 3x - 16x + 18x 1 2 1 2 1 2 1 2 (Round results to three decimal places.) 54. f′1-21 = 0,1 f ″2 -1 7 0,1 f 2-1 = -5; 1f′ 27 = 0, 1 2 f ″ 7 0, f 7 10; f ″ 3 0, and f 3 2 4 3 2 6 = = = 18. f x = 3x + 4x - 12x + 5 1 2 1 2 1 2 1 2 (Round results to three decimal places.) 55. f′102 = 0, f 1″ 02 6 0, f 01 2= 5; f′ 2 1= 20, f ″ 2 7 0, 1 2 f 2 2; f′ 4 0, f ″ 4 0, and f 4 3 4 2 2 4 = = 6 = 19. f x = x - 6x 20. f x = 2x - x 1 2 1 2 1 2 1 2 1 2 56. f′ 1 0, f ″ 1 0, f 1 2; f′ 1 0, 3 2 1 -2 = 1 2 - 7 1 2 - = -1 2 = 21. f1x2 = x - 6x + 9x + 1 1 2 f ″ 1 7 0, f 1 = -2; f′ 0 = 0, f ″ 0 6 0, 3 2 and1 f 02 = 0 1 2 1 2 1 2 22. f1x2 = x - 2x - 4x + 3 1 2 1 2 1 2 1 2 4 3 4 3 1 2 23. f1x2 = x - 2x 24. f x = 3x + 4x APPLicAtionS 3 2 25. f1x2 = x - 6x - 135x 1 2 Business and economics 3 2 26. f1x2 = x - 3x - 144x - 140 Total revenue, cost, and profit. Using the same set of axes, graph R, C, and P, the total-revenue, total-cost, and total- 27. f x x4 4x3 10 28. f x 4x3 2x2 x 1 2 = - + = 3 - + profit functions. 3 2 29. f x = x - 6x + 12x - 6 2 1 2 1 2 57. R x = 50x - 0.5x , C x = 4x + 10 3 3 5 30. f1x2 = x + 3x + 1 31. f x = 5x - 3x 2 58. R1x2 = 50x - 0.5x , C1x2 = 10x + 3 32. f x 20x3 3x5 1 2 = - 1 2 59. Small business. The percentage of the U.S. national (Round results to three decimal places.) 1 2 1 2 1 2 income generated by nonfarm proprietors between 1970 2 2 33. f x = x 1 - x and 2000 can be modeled by the function f given by 3 2 (Round results to three decimal places.) 13x - 240x - 2460x + 585,000 1 2 1 2 p x , 2 2 = 34. f x = x 3 - x 75,000 (Round results to three decimal places.) where1 2 x is the number of years since 1970. (Source: Based 1 2 1 2 2 3 2 3 on data from www.bls.gov.) Sketch the graph of this 35. f x = x - 1 36. f x = x + 1 function for 0 … x … 40. 37. f x x 3 1>3 1 38. f x x 2 1>3 3 1 2 = 1 - 2 - 1 2 = 1 - 2 + 60. labor force. The percentage of the U.S. civilian labor > 2 3 > 39. f1x2 = 1-2 x -2 4 + 5 1 2 1 2 force aged 45–54 between 1970 and 2000 can be mod- eled by the function f given by 2>3 40. f1x2 = -31x - 22 + 3 2 f x = 0.025x - 0.71x + 20.44, 2> 2 41. f1x2 = -x1 1 - 2x 42. f x = x 4 - x where x is the number of years after 1970. (Source: Based 1 2 2 2 on data from www.bls.gov.) Sketch the graph of this 1 2 8x 1 2 x 43. f x = 2 44. f x = 2 function for 0 … x … 30. x + 1 x + 1 1 2 1 2 Life and Physical Sciences -4 3 45. f x = 2 46. f x = 2 61. Rainfall in Reno. The average monthly rainfall in Reno, x + 1 x + 1 Nevada, is approximated by 1 2 1 2 4 3 2 For Exercises 47–56, sketch a graph that possesses the charac- R t = -0.006t + 0.213t - 1.702t + 0.615t + 27.745, teristics listed. Answers may vary. where R t is the amount of rainfall in millimeters at 1 2 47. f is decreasing and concave up on - ∞, 2 , t months since December. (Source: Based on data from f is decreasing and concave down on 2, ∞ . weather.com.)1 2 1 2 48. f is increasing and concave up on - ∞1 , 4 ,2 y f is increasing and concave down on 4, ∞ . 1 2 30 49. f is decreasing and concave down on , 3 , 1 - ∞ 2 25 f is decreasing and concave up on 3, ∞ . 20 1 2 50. f is increasing and concave down on ∞, 1 , 15 1 -2 10

is increasing and concave up on 1, . Average rainfall f ∞ (in millimeters) 1 2 5 51. f is concave up at 1, -3 , concave1 down2 at 8, 7 , and 1 2 3 4 5 6 7 8 9 10 11 12 t has an inflection point at 5, 4 . Month 1 2 1 2 52. f is concave down at 1, 51, concave2 up at 7, -2 , and Find the point(s) of inflection for R. What is the signifi- has an inflection point at 4, 1 . cance of these point(s)? 1 2 1 2 Copyright 2016 Pearson. All rights reserved. 1 2

M02_BITT9391_11_SE_C02.2.indd 218 29/10/14 4:43 PM Not for Sale Exercise Set 2.2 219

62. Coughing velocity. A person coughs when a foreign 68. If f has exactly two critical values at x = a and x = b, object is in the windpipe. The velocity of the cough de- where a 6 b, then there must exist at least one point of pends on the size of the object. Suppose a person has a inflection at x = c such that a 6 c 6 b. In other words, windpipe with a 20-mm radius. If a foreign object has a at least one point of inflection must exist between any radius r, in millimeters, then the velocity V, in millime- two critical points. ters per second, needed to remove the object by a cough is given by 69. A function f can have no extrema but still have at least 2 3 one point of inflection. V r = k 20r - r , 0 … r … 20, 70. If f has two points of inflection, then there is a critical where k is some positive constant. For what size ob- 1 2 1 2 value located between those points of inflection. ject is the maximum velocity required to remove the object? 71. The function f can have a point of inflection at a critical value. 72. The function f can have a point of inflection at an ex- treme value. 73. A function f can have exactly one extreme value but no points of inflection. 74. hours of daylight. The number of hours of daylight in Chicago is represented in the graph below. On what dates is the number of hours of daylight changing most rapidly? How can you tell?

SyntheSiS 18 In each of Exercises 63 and 64, determine which graph is the derivative of the other and explain why. 15 63. 64. 12 y h h y 9 4 g 100 6 2

Number of hours daylight 3 ±4 ±2 24x ±2 ±1 12x

±2 0 60 120 180 240 300 360 g ±100 Number of days since January 1

65. Use calculus to prove that the relative minimum or max- (Source: Astronomical Applications Dept., U.S. Naval imum for any function f for which Observatory. ) 2 f x = ax + bx + c, a  0, occurs at x b 2a . 1 2 = - TeChnology ConneCTIon 66. Use calculus to prove>1 that2 the point of inflection for any function g given by Graph each function. Then estimate any relative extrema. 3 2 Where appropriate, round to three decimal places. g x = ax + bx + cx + d, a  0, 2 3 occurs at x b 3a . 75. f x = 4x - 6x 1 2 = - 2 3 > For Exercises 67–73, assume>1 2 that f is differentiable over 76. f1x2 = 3x - 2x  , . Classify each of the following statements as either > H H 77. f x x2 1 x 3 true or false. If a statement is false, explain why. 1 2 = - 1 2 2 3 78. f1x2 = x 1x - 22 67. If f has exactly two critical values at x = a and x = b, 2 3 2 3 where a 6 b, then there must exist exactly one point of 79. f1x2 = x1- 1 2 - x + 1 inflection at x = c such that a 6 c 6 b. In other words, > > exactly one point of inflection must exist between any 80. f1x2 = x1 - 2x 1 2 two critical points. 2 1 2

Copyright 2016 Pearson. All rights reserved.

M02_BITT9391_11_SE_C02.2.indd 219 29/10/14 4:43 PM Not for Sale

220 Chapter 2 ● Applications of Differentiation

81. Head of household education. The data in the follow- ing table relate the percentage of households headed by answers to Quick checks someone with a bachelor’s degree or higher to the num- 1. Relative minimum: y g(x) 5 10x3 2 6x5 ber of years since 2000. -4 at x = -1; relative 4 maximum: 4 at x = 1 2 −1.0 −0.5 0.5 1.0 x Number of Years PerceNtage of −2 siNce 2000 HouseHolds −4 0 66.0 2. 0, 0 , 2, 2.475 , and 2, 2.475 1 75.2 22 - 22 - y 3 78.3 3. Relative maxima: 17 at1 x =2 -2 and x = 2; 10 7 84.0 1 2 1 2 relative minimum: 1 at −2 −1 12x 9 88.5 x = 0 −10 ( ) 5 1 1 8 2 2 4 10 89.2 −20 f x x x 30 11 89.9 −

y (Source: www.census.gov/prod/2013pubs/ 4. No relative extrema p20-569.pdf.) 4 2

a) Use regression (see Section R.6) to fit linear, cubic, −4 −2 24x −2 1/3 and quartic functions y = f x to the data, where f(x) 5 1 2 (x 2 2) x is the number of years since 2000 and y is the percentage of households headed1 2 by someone with a bachelor’s degree or higher. Which function f best fits the data? b) What is the domain of f? c) Does f have any relative extrema? How can you tell?

graph sketching: asymptotes and rational functions 2.3 Thus far we have developed a strategy for graphing continuous functions. We now consider discontinuous functions, most of which are rational functions. Our graphing ● Find limits involving infinity. skills must allow for discontinuities and certain lines called asymptotes. ● Determine the asymptotes of a Recall the definition of a rational function: function’s graph. ● Graph rational functions. Definition A rational function is a function f that can be described by P x f x = , Q x 1 2 1 2 where P x and Q1 2x are polynomials, with Q x not the zero polynomial. The domain of f consists of all x for which Q x 0. 1 2 1 2 1 2 1 2 We wish to graph rational functions in which the denominator is not a constant. Be- fore doing so, we need to reconsider limits.

Copyright 2016 Pearson. All rights reserved.

M02_BITT9391_11_SE_C02.2.indd 220 29/10/14 8:16 PM Not for Sale

2.3 ● Graph Sketching: Asymptotes and Rational Functions 221

Vertical and Horizontal Asymptotes Consider the rational function x2 - 1 x - 1 x + 1 f x = = , 2 x 2 x 3 x + x - 6 1 - 21 + 2 1 2 whose graph is shown below. 1 21 2

y

6 5 x2 ± 1 4 f(x) = Ð Ð ± Ð Ð x2 + x ± 6 3

2 1 0, ± ± 126 y = 1, horizontal (±1, 0)

±6 ±5 ±4 ±3 ±2 ±1 (1, 0) 3 4 5 678910 11 x ±1 TEcHnology connEcTion ±2 Asymptotes x = ±, 3vertical ±3 x = 2, vertical asymptote asymptote ±4 Graph f x = 8 x2 - 4 in connected mode, using the ±5 1 2 >1 2 window -6, 6, -8, 8 . Vertical ±6 asymptotes occur at x = -2 and x = 2. These3 lines are4 not part of the graph. Note that as x gets closer to 2 from the left, the function values approach - ∞. As x gets closer to 2 from the right, the function values approach ∞. Thus, y = 8 x2 ± 4 lim f x = - ∞ and lim f x = ∞. 8 xS2- xS2+ For this graph,1 2we can regard the line1 x2= 2 as a “limiting line” called a vertical asymptote. The line x = -3 is another vertical asymptote. –6 6

Definition –8 Not part of the graph! The line x = a is a vertical asymptote if any of these limit statements is true: lim f x = ∞, lim f x = - ∞, lim f x = ∞, or lim f x = - ∞. ExErcisEs xSa- xSa- xSa+ xSa+ Graph each of the following in 1 2 1 2 1 2 1 2 both dot and connected modes. Try to locate the vertical asymp- If a rational function f is simplified, meaning that the numerator and denomina- totes visually. Then verify your tor have no common factor other than -1 or 1, then if a makes the denominator 0, the results using the method of Ex- line x = a is a vertical asymptote. For example, amples 1 and 2. You may need to 2 vary the viewing windows. x - 9 x - 3 x + 3 f x = = 2 x - 3 x - 3 x + 7x + 10 1 21 2 1. f x = 1 2 x2 + 3x - 28 does not have a vertical asymptote at x = 3 because x - 3 is a common factor of 1 2 the numerator and the denominator. In contrast, 3x - 4 1 2 2. f x = 2 x x - 4 x + 2 x - 2 g x = = 2 x 3 x 4 1 2 x + x - 12 1 - 21 + 2 1 2 1 21 2

Copyright 2016 Pearson. All rights reserved.

M02_BITT9391_11_SE_C02.3.indd 221 29/10/14 4:46 PM Not for Sale

222 Chapter 2 ● Applications of Differentiation

has x = 3 and x = -4 as vertical asymptotes since neither x - 3 nor x + 4 is a factor of the numerator. The figures below show four ways in which a vertical asymptote1 2 can1 occur.2 The dashed lines represent the asymptotes. They are shown for visual assistance only; they are not part of the graphs of the functions.

y y y y y = f(x) y = f(x) y = f(x)

x x x x y = f(x)

x = a x = a x = a x = a lim f(x) = ∞ lim f(x) = –∞ lim f(x) = ∞ lim f(x) = –∞ x → a– x → a– x → a+ x → a+ Quick Check 1 Determine the vertical 3x - 2 asymptotes of eXAMPLe 1 Determine the vertical asymptotes of f x = . x x - 5 x + 3 1 Solution The expression is simplified. Thus, the vertical1 2 asymptotes are the lines f x = 2 . 1 21 2 x x - 16 x = 0, x = 5, and x = -3. 1 1 2 1 2 x2 - 2x eXAMPLe 2 Determine the vertical asymptotes of f x = . x3 - x 1 2 Solution We first simplify the expression: 2 Quick Check 2 x - 2x x x - 2 f x = = 3 x x 1 x 1 Determine the vertical x - x -1 2+ asymptotes of 1 2 x - 2 = 1 , x21 0. 2 x 1 x 1 x - 2 - + g x = 3 2 . x - x - 2x We now see that 1the vertical21 asymptotes2 are the lines x = -1 and x = 1. 2 1 2 Look again at the graph on p. 221. Note that function values approach 1 as x approaches - ∞, meaning that f x S 1 as x S - ∞. Also, function values approach 1 as x approaches ∞, meaning that f x S 1 as x S ∞. Thus, 1 2 lim f x = 1 and lim f x = 1. xS - ∞ xS1∞2 The line y = 1 1is 2called a horizontal asymptote.1 2

Definition The line y = b is a horizontal asymptote if either or both of the following limit statements is true: lim f x = b or lim f x = b. xS - ∞ xS ∞ 1 2 1 2

Horizontal asymptotes occur when the degree of the numerator is less than or equal to the degree of the denominator. (The degree of a polynomial in one variable is the highest power of that variable.) The following figures show three ways in which horizontal asymptotes can occur.

Copyright 2016 Pearson. All rights reserved.

M02_BITT9391_11_SE_C02.3.indd 222 29/10/14 4:46 PM Not for Sale

2.3 ● Graph Sketching: Asymptotes and Rational Functions 223

y y y y = f(x)

y = f(x) lim f(x) = b x → –∞ y = b y = b y = f(x) y = b lim f(x) = b lim f(x) = b x → ∞ x → ∞ x x x

A horizontal asymptote is the limit of a rational function as inputs approach - ∞ or ∞.

3x - 4 eXAMPLe 3 Determine the horizontal asymptote of f x = . x Solution To find the horizontal asymptote, we consider1 2 3x - 4 lim f x = lim . xS ∞ xS ∞ x As the inputs approach1 2 ∞, the outputs approach 3. Thus, 3x - 4 lim = 3. xS ∞ x y Another way to find this limit is to use the fact that 3x − 4 f(x) = 6 x 1 b as x S ∞, we have S 0, and more generally, S 0, 5 Horizontal x axn 4 asymptote: y = 3 3 for any positive integer n and any constants a and b, a  0. We multiply by 1, using 2 1 x , 1 x : 1 1 > 2 1 > 32x - 4 3x - 4 1 x lim = lim # xS ∞ x xS ∞ x 1 x −4 −3 −2 −1 1234 x 1 > 2 −1 3x 4 1 > 2 −2 x - x = lim A visualization of Example 3 xS ∞ x x 4 3 - x = lim Simplifying, we assume x  0. xS ∞ 1 4 = lim 3 - Horizontal xS ∞ x y 3x2 + 2x − 4 asymptote: f(x )= 3 2 3 a0 3. b y = − 2x − x + 1 = - = 2 2 In a similar manner, it can be shown that 1 lim f x = 3. xS ∞ −3 −2 −1 321 x - −1 The horizontal asymptote1 2 is the line y = 3. −2 3x2 + 2x - 4 −3 eXAMPLe 4 Determine the horizontal asymptote of f x = . 2x2 - x + 1 −4 Solution As in Example 3, the degree of the numerator1 is2 the same as the degree of A visualization of Example 4 the denominator. Let’s adapt the algebraic approach used in that example.

Copyright 2016 Pearson. All rights reserved.

M02_BITT9391_11_SE_C02.3.indd 223 29/10/14 4:46 PM Not for Sale

224 Chapter 2 ● Applications of Differentiation

To do so, we multiply by 1 x2 , 1 x2 : 1 >1 2 1 >2 2 4 2 3 + - 3x + 2x - 4 x2 x x2 f x = 2 # = . 2x - x + 1 1 1 1 2 - + 1 2 x2 x x2 Quick Check 3 As ͉ x ͉ approaches ∞, the numerator approaches 3 and the denominator approaches 2. 3 Determine the horizontal Therefore, the values of the function approach 2. Thus, asymptote of 3 3 lim f x = and lim f x = . 2x - 1 x + 1 xS -∞ 2 xS ∞ 2 f x = . 3x 2 5x 6 The line y 3 1is 2a horizontal asymptote.1 2 1 + 21 + 2 = 2 3 1 2 1 21 2 Examples 3 and 4 lead to the following result.

When the degree of the numerator is the same as the degree of the denominator, the line y = a b is a horizontal asymptote, where a is the leading coefficient of the numerator and b is the leading coefficient of the denominator. >

2x + 3 y eXAMPLe 5 Determine the horizontal asymptote of f x = . x3 - 2x2 + 4 4 1 2 2x + 3 3 f(x) = Solution Since the degree of the numerator is less than the degree of the denomi- x3 2x2 4 2 − + nator, there is a horizontal asymptote. To identify that asymptote, we divide both 1 the numerator and denominator by the highest power of x in the denominator, as in Examples 3 and 4, and find the limits as ͉ x ͉ S ∞: −4 −3 −2 −1 1234 x −1 1 2 3 Horizontal + −2 asymptote: 2x + 3 x3 x2 x3 y = 0 f x = # = . −3 x3 - 2x2 + 4 1 2 4 1 - + −4 1 2 x3 x x3 A visualization of Example 5 As ͉ x ͉ approaches ∞, the expressions with a denominator that is a power of x ap- proach 0. Thus, the numerator of f x approaches 0 as its denominator approaches 1; hence, the entire expression approaches 0. That is, for x S - ∞ or x S ∞, 1 2 0 + 0 f x ≈ , 1 - 0 + 0 TEcHnology connEcTion 1 2 so lim f x = 0 and lim f x = 0, xS - ∞ xS ∞ ExErcisEs Graph each of the following. and the x-axis, the1 2 line y = 0, is a1 horizontal2 asymptote. Try to locate the horizontal asymptotes using the table and trace features. Verify your When the degree of the numerator is less than the degree of the denominator, results using the method of the x-axis, or the line y = 0, is a horizontal asymptote. Examples 3–5.

9x4 - 7x2 - 9 1. f x = slant Asymptotes 3x4 + 7x2 + 9 1 2 Some asymptotes are neither vertical nor horizontal. For example, in the graph of 135x5 - x2 2. f x = 7 x2 - 4 x f x = , 1 2 x - 1 1 2 Copyright 2016 Pearson. All rights reserved.

M02_BITT9391_11_SE_C02.3.indd 224 29/10/14 4:46 PM Not for Sale

2.3 ● Graph Sketching: Asymptotes and Rational Functions 225

y shown at left, as ͉ x ͉ approaches ∞, the curve approaches the line y = x + 1. The line 10 y = x + 1 is called a slant asymptote, or oblique asymptote. In Example 6, we will see how the line y = x + 1 is determined. 1 x + 2 5 = rtical asymptote y f(x) = Ðбx ± 4 x ± 1 Ve Definition A linear asymptote that is neither vertical nor horizontal is called a slant ±3 ±2 ±1 1 2 3 x asymptote, or an oblique asymptote. For any rational function of the form f x = P x Q x , a slant asymptote occurs when the degree of P x is exactly asymptote 1 more than the degree of Q x . Oblique ±5 1 2 1 2> 1 2 1 2 1 2 We find a slant asymptote by division. ±10 x = 1 2 x - 4 eXAMPLe 6 Find the slant asymptote of f x = . x - 1 Solution When we divide the numerator by1 the2 denominator, we obtain a quotient of x + 1 and a remainder of -3: x + 1 2 x - 1; x - 4 2 2 x - x x - 4 -3 f x = = x + 1 + . x - 4 x - 1 x - 1 Quick Check 4 x - 1 1 2 1 2 Find the slant asymptote of - 3 2 2x + x - 1 Note that as ͉ x ͉ approaches ∞, 3 x 1 approaches 0. Thus, y = x 1 is the g x = . - - + x - 3 slant asymptote. 4 >1 2 1 2 intercepts If they exist, x-intercepts occur at values of x for which y = f x = 0 and are points at which the graph crosses (or touches) the x-axis. If it exists, the y-intercept occurs at the value of y for which x = 0 and is the point at which the graph1 2 crosses (or touches) the y-axis.

y eXAMPLe 7 Find the intercepts of the function given by 30 x3 - x2 - 6x f x = . x2 3x 2 20 x3 − x2 − 6x - + f(x) = 2 1 2 x − 3x + 2 Solution We factor the numerator and the denominator: (−2, 0) 10 x x + 2 x - 3 f x = . −3 −2 −1 12345 x x 1 x 2 1 - 21 - 2 (0, 0) −10 (3, 0) 1 2 To find thex -intercepts,1 21we solve2 f x = 0. Such values occur when the numerator is 0 and the denominator is not. Thus, we solve −20 1 2 x x + 2 x - 3 = 0. A visualization of Example 7 The x-values1 that21 make the2 numerator 0 are 0, -2, and 3. Since none of these make the denominator 0, they yield the x-intercepts 0, 0 , 2, 0 , and 3, 0 . Quick Check 5 - To find they -intercept, we let x = 0: Find the intercepts of the func- 1 2 1 2 1 2 tion given by 03 - 02 - 6 0 f 0 = = 0. 3 2 x - x 0 - 3 0 +1 22 h x = 2 . 1 2 x - 4 In this case, the y-intercept1 2is also an x-intercept, 0, 0 . 5 1 2 Copyright 2016 Pearson. All rights reserved. 1 2

M02_BITT9391_11_SE_C02.3.indd 225 29/10/14 4:46 PM Not for Sale

226 Chapter 2 ● Applications of Differentiation

sketching graphs We can now refine our strategy for graphing.

Strategy for Sketching Graphs a) Intercepts. Find any x- and y-intercepts. b) Asymptotes. Find any vertical, horizontal, or slant asymptotes. c) Derivatives and domain. Find f′ x and f ″ x . Find the domain of f. d) Critical values of f. Find any inputs for which f′ x is not defined or for which f′ x = 0. 1 2 1 2 e) Increasing and/or decreasing; relative extrema. Substitute1 2 each critical 1 2 value, x0,from step (d) into f ″ x . If f ″ x0 6 0, then x0 yields a relative maximum and f is increasing on the left of x0 and decreasing on the right. 1 2 1 2 If f ″ x0 7 0, then x0 yields a relative minimum and f is decreasing on the left of x0 and increasing on the right. On intervals where no critical value exists,1 use2 f′ and test values to find wheref is increasing or decreasing. f) Inflection points. Determine candidates for inflection points by finding x-values for which f ″ x does not exist or for which f ″ x = 0. Find the function values at these points. If a function value f x does not exist, then the function does not1 have2 an inflection point atx . 1 2 g) Concavity. Use the values from step (f) as endpoints 1of2 intervals. Determine the concavity over each interval by checking to see where f′ is increasing— that is, where f ″ x 7 0—and where f′ is decreasing—that is, where f ″ x 6 0. Do this by substituting a test value from each interval into f ″ x . Use the results1 2 of step (d). h) Sketch1 2 the graph. Use the information from steps (a)–(g) to sketch the graph, plotting1 2 extra points as needed.

8 eXAMPLe 8 Sketch the graph of f x = . x2 - 4 1 2 Solution a) Intercepts. The x-intercepts occur at values for which the numerator is 0 but the denominator is not. Here, the numerator is a constant, so there are no x-intercepts. To find they- intercept, we compute f 0 :

8 8 1 2 f 0 = 2 = = -2. 0 - 4 -4 1 2 This gives us one point on the graph, 0, -2 . b) Asymptotes. 1 2 Vertical: The denominator, x2 - 4 = x + 2 x - 2 , is 0 for x-values of -2 and 2. Thus, the graph has the lines x = -2 and x = 2 as vertical asymptotes. We draw them as dashed lines (they are guidelines,1 21 not part2 of the actual graph). Horizontal: The degree of the numerator is less than the degree of the denominator, so the x-axis, y = 0, is the horizontal asymptote. Slant: There is no slant asymptote since the degree of the numerator is not 1 more than the degree of the denominator. c) Derivatives and domain. We find f′ x and f ″ x using the Quotient Rule: 2 -16x 1 2 161 32x + 4 f′ x = 2 2 and f ″ x = 2 3 . x - 4 x1 - 4 2 1 2 1 2 The domain of 1f, f′, and2 f ″ is - ∞, -2 ∪ 1-2, 2 ∪2 2, ∞ , as determined in step (b). Copyright 2016 Pearson. All 1rights reserved.2 1 2 1 2

M02_BITT9391_11_SE_C02.3.indd 226 29/10/14 4:46 PM Not for Sale

2.3 ● Graph Sketching: Asymptotes and Rational Functions 227

d) Critical values of f. We look for values of x for which f′ x = 0 or for which f′ x does not exist. From step (c), we see that f′ x = 0 for values of x for which -16x = 0, but the denominator is not 0. The only such1 2 number is 0 itself. The1 2 derivative f′ x does not exist at -2 and 2, but1 neither2 value is in the domain of f. Thus, the only critical value is 0. 1 2 e) Increasing and/or decreasing; relative extrema. We use the undefined values and the critical values to determine the intervals over which f is increasing and the intervals over which f is decreasing. The values to consider are -2, 0, and 2. 16 3 # 02 + 4 64 Since f ″ 0 = = 6 0, 2 3 64 10 - 4 2 - 1 2 we know that a relative1 2maximum exists at 0, f(0 ), or 0, -2 . Thus, f is increasing over the interval -2, 0 and decreasing over 0, 2 . Since f ″ x does not exist for the x-values -1 2 and2 2, we1 use f2′ x and test values to see if f is increasing1 or decreasing2 on - ∞, -2 1 and 2 2, ∞ . 1 2 1 2 -16 -3 48 1 2 1 2 Test -3: f ′ -3 = = 7 0, so f is increasing on - ∞, -2 . 2 2 25 -3 1 - 24 1 2 -16 3 -48 1 2 Test 3: f ′ 3 = 31 2 4 = 6 0, so f is decreasing on 2, ∞ . 2 2 25 3 -1 24 1 2 1 2 f) Inflection points. We determine31 2 candidates4 for inflection points by finding where f ″ x does not exist and where f ″ x = 0. The only values for which f ″ x does not exist are where x2 - 4 = 0, or -2 and 2. Neither value is in the domain of f, so we1 focus2 solely on where f ″ x = 0,1 or2 1 2 2 16 3x + 4 = 0. 1 2 2 Since 1613x + 42 7 0 for all real numbers x, there are no points of inflection. g) Concavity. Since no values were found in step (f), the only place where concavity could 1 2 change is on either side of the vertical asymptotes, x = -2 and x = 2. To check, we see where f ″ x is positive or negative. The numbers -2 and 2 divide the x-axis into three intervals. We choose a test value in each interval and substitute into f ″ x . 1 2 2 16 3 -3 + 4 1 2 Test -3: f ″ -3 = 2 3 7 0. 3-13 2- 4 4 2 1 2 16 3 0 + 4 31 2 4 We already knew this from step (e). Test 0: f ″ 0 = 2 3 6 0. 301 2- 4 4 1 2 16 3 3 2 4 31 2 +4 Test 3: f ″ 3 = 2 3 7 0. 331 2- 4 4 1 2 31 2 4 Interval x -2 2 ++11)1++1* (++11)++11* (++11)+1+1 Test Value x = -3 x = 0 x = 3

Sign of f″ x f ″ -3 7 0 f ″ 0 6 0 f ″ 3 7 0

Result 1 2 f′ is1 increasing;2 f′ is decreasing;1 2 f′ is 1increasing;2 f is concave up f is concave down f is concave up on - ∞, -2 on -2, 2 on 2, ∞ 1 2 1 2 1 2 Change does not Change does not indicate a point indicate a point of inflection since of inflection since f -2 does not exist. f 2 does not exist. Copyright 2016 Pearson. All rights reserved.1 2 1 2

M02_BITT9391_11_SE_C02.3.indd 227 29/10/14 4:46 PM Not for Sale

228 Chapter 2 ● Applications of Differentiation

The function is concave up over the intervals - ∞, -2 and 2, ∞ . The function is concave down over -2, 2 . 1 2 1 2 h) Sketch the graph. We sketch the graph using the information in the following table, 1 2 plotting extra points as needed. The graph is shown below.

y x f x 8 2 7 -4 1 3 2 6 5 f(x) = Ðб8 -3 1.6 2 2 4 x ± 4 -1 -23 3 0 -2 2 2 1 1 -23 ±5 ±4 ±3 ±2 ±1 123 4 5 x 3 1.6 ±1 Relative maximum 4 2 ±2 3 ±3 ± x = ± 2 4 (0, ±2) x = 2 ±5 ±6 ±7 ±8

Increasing Increasing Decreasing Decreasing

Concave up Concave down Concave up

2 x + 4 eXAMPLe 9 Sketch the graph of the function given by f x = . x Solution 1 2 a) Intercepts. The equation f x = 0 has no solution, so there is no x-intercept. Since 0 is not in the domain, there is no y-intercept either. 1 2 b) Asymptotes. Vertical: Since replacing x with 0 makes the denominator 0, the line x = 0 (the y-axis) is a vertical asymptote. Horizontal: The degree of the numerator is greater than the degree of the denominator, so there is no horizontal asymptote. Slant: The degree of the numerator is 1 greater than the degree of the denominator, so there is a slant asymptote. Note that 2 2 x + 4 x 4 4 = = x . x x + x + x As ͉ x ͉ approaches ∞, 4 x approaches 0, so the line y = x is a slant asymptote. c) Derivatives and domain. We find f′ x and f ″ x : > 2 41 2 1 2 f′ x = 1 - 4x- = 1 - ; x2 1 2 8 f ″ x = 8x-3 = . x3 1 2 The domain of f, f′, and f ″ is - ∞, 0 ∪ 0, ∞ , or all real numbers except 0. 1 2 1 2

Copyright 2016 Pearson. All rights reserved.

M02_BITT9391_11_SE_C02.3.indd 228 29/10/14 4:46 PM Not for Sale

2.3 ● Graph Sketching: Asymptotes and Rational Functions 229

d) Critical values of f. We see from step (c) that f′ x is undefined at x = 0, but 0 is not in the domain of f. Thus, to find critical values, we solve f′ x = 0, looking for solutions other than 0: 1 2 1 2 4 1 - = 0 Setting f′ x equal to 0 x2 4 1 2 1 = x2 x2 = 4 Multiplying both sides by x2 x = {2. Thus, -2 and 2 are critical values. e) Increasing and/or decreasing; relative extrema. We use the values found in step (d) to determine where f is increasing or decreasing. The points to consider are -2, 0, and 2. Since 8 f ″ -2 = 3 = -1 6 0, -2 1 2 we know that a relative1 2 maximum exists at -2, f(-2 ), or -2, -4 . Thus, f is increasing over - ∞, -2 and decreasing over -2, 0 . Since 1 2 1 2 1 2 1 2 8 f ″ 2 = = 1 7 0, 2 3 1 2 we know that a1 relative2 minimum exists at 2, f(2 ), or 2, 4 . Thus, f is decreasing over 0, 2 and increasing over 2, ∞ . 1 2 1 2 f) Inflection points. We determine candidates for inflection points by finding where 1 2 1 2 f ″ x does not exist or where f ″ x = 0. The only value for which f ″ x does not exist is 0, but 0 is not in the domain of f. Thus, the only place an inflection point could1 2 occur is where f ″ x = 0: 1 2 1 2 8 1 2 = 0. x3 But this equation has no solution. Thus, there are no points of inflection. g) Concavity. Since no values were found in step (f), the only place where concavity could change would be on either side of the vertical asymptote, x = 0. In step (e), we used the Second-Derivative Test to determine relative extrema. From that work, we know that f is concave down over - ∞, 0 and concave up over 0, ∞ . 1 2 1 2 Interval 0 x ˚˚˚˚˘˚˚˚˙¯ ˚ ˚˘ ˚ ˚˚˚

Test Value x = -2 x = 2

Sign of f ″ x f ″ -2 6 0 f ″ 2 7 0

Result 1 2 f′ is1 decreasing;2 f′ is 1increasing;2 f is concave down f is concave up on - ∞, 0 on 0, ∞ 1 2 1 2 Change does not indicate a point of inflection since f 0 does not exist. 1 2 Copyright 2016 Pearson. All rights reserved.

M02_BITT9391_11_SE_C02.3.indd 229 29/10/14 4:46 PM Not for Sale

230 Chapter 2 ● Applications of Differentiation

h) Sketch the graph. We sketch the graph, shown below, using the preceding information and additional computed values of f.

y x f x 9 8 -4 -15 2 7 -2 -4 Relative 6 (2, 4) -1 -5 minimum 5 4 1 5 2 3 f(x) = Ðx + б4 x 2 4 2 4 5 1 ±9 ±8 ±71±6 ±5 ±4 ±3 ±2 ±1 24635798 x ±1 ±2 Slant asymptote: y = x Relative ±3 maximum ±4 ±5 (±2, ±4) ±6 ±7 Quick Check 6 ±8 ±9 Sketch the graph of the function given by 2 Increasing Decreasing Decreasing Increasing x - 9 f x = . x - 1 Concave down Concave up 6 1 2 Section Summary ● A line x = a is a vertical asymptote if lim f x = ∞ Division of polynomials can be used to determine the xSa- { or lim f x = ∞. equation of the slant asymptote. x a+ { S 1 2 ● ● A line y = b is a horizontal asymptote if lim f x = b Vertical, horizontal, and slant asymptotes can be used 1 2 xS ∞ as guides for accurate curve sketching. Asymptotes are or lim f x = b. xS -∞ 1 2 not a part of a graph but are visual guides only. ● A slant asymptote occurs when the degree of the numer- 1 2 ator is 1 greater than the degree of the denominator.

2.3 Exercise Set

Determine the vertical asymptote(s) of each function. If none 7 6 9. f x = 2 10. f x = 2 exists, state that fact. x + 49 x + 36 x + 4 2x - 3 1 2 1 2 1. f x = 2. f x = Determine the horizontal asymptote of each function. If none x - 2 x - 5 exists, state that fact. 1 2 1 2 5x 3x 6x 3x2 3. f x 4. f x = 2 = 2 11. f x = 12. f x = 2 x - 25 x - 9 8x + 3 6x + x 1 2 1 2 1 2 1 2 x 3 x 2 4x 2x + + 13. f x 14. f x 5. f x = 3 6. f x = 3 2 = 2 = 3 2 x - x x - 6x + 8x x - 3x 3x - x 1 2 1 2 1 2 1 2 x 2 x 6 2 3 + + 15. f x 4 16. f x 5 7. f x = 2 8. f x = 2 = + = - x + 6x + 8 x + 7x + 6 x x 1 2 1 2 1 2 1 2

Copyright 2016 Pearson. All rights reserved.

M02_BITT9391_11_SE_C02.3.indd 230 29/10/14 4:46 PM Not for Sale Exercise Set 2.3 231

6x3 4x 8x4 5x2 10 1 + - 51. f x = 52. f x = 17. f x = 2 18. f x = 3 2 2 2 3x - x 2x + x x + 4 x - 1 1 2 1 2 1 2 2 1 2 3 4x3 3x 2 6x4 4x2 7 x + 1 x - + + - 53. f x = 54. f x = 19. f x = 3 20. f x = 5 x 2 x + 2x - 4 2x - x + 3 x - 1 1 2 2 1 2 2 1 2 3 1 2 4 3 x - 16 x - 9 2x - 4x + 1 5x - 2x + x 55. f x = 56. f x = 21. f x = 3 22. f x = 5 3 x + 4 x - 3 4x + 2x - 3 x - x + 8 1 2 1 2 1 2 1 2 Sketch the graph of each function. Indicate where each func- APPLicAtionS tion is increasing or decreasing, where any relative extrema occur, where asymptotes occur, where the graph is concave Business and Economics up or concave down, where any points of inflection occur, and 57. Average cost. The total cost, in thousands of dollars, for where any intercepts occur. Acme, Inc., to produce x pairs of rocket skates is given by 2 4 5 C x = 3x + 80. 23. f x = 24. f x = - x x a) The average cost is given by A x C x x. Find 1 2 = 1 2 1 2 A x . -2 1 25. f x = 26. f x = b) Graph the average cost. 1 2 1 2> x 5 x 5 - - c) Find1 2 the slant asymptote for the graph of y = A x , 1 2 1 1 2 1 and interpret its significance. 27. f x = 28. f x = 1 2 x - 3 x + 2 58. Depreciation. Suppose that the value V of the inventory 1 2 1 2 at Fido’s Pet Supply, in hundreds of dollars, decreases -2 -3 29. f x = 30. f x = (depreciates) after t months, where x + 5 x - 3 25t2 1 2 1 2 V t 50 . 2x + 1 3x - 1 = - 2 31. f x = 32. f x = t + 2 x x a) Find1 2 V 0 , V 5 , V 10 , and V 70 . 1 2 1 2 2 1 2 9 b) Find the maximum value of the inventory over the 33. f x = x + 34. f x = x + 1 2 1 2 1 2 1 2 x x interval 0, ∞). c) Draw a graph of V. 1 2 1 2 -1 2 d) Does there3 seem to be a value below which V t will 35. f x = 36. f x = x2 x2 never fall? Explain. 1 2 1 2 1 2 x x 59. Total cost and revenue. The total cost and total revenue, 37. f x = 38. f x = in dollars, from producing x couches are given by x 3 x 2 - + 1 2 C x = 5000 + 600x and R x = - 2x + 1000x. 1 2 1 1 2 1 - a) Find the total-profit function, P x . 39. f x = 2 40. f x = 2 1 2 1 2 x + 3 x + 2 b) The average profit is given by A x = P x x. 1 2 1 2 Find A x . 1 2 x 3 + (Hint: Simplify.) c) Find the slant asymptote for the1 graph2 1of 2>y A x . 41. f x = 2 = x - 9 d) Graph 1the2 average profit. 1 2 1 2 x - 1 60. cost of pollution control. Cities and companies find 42. f x = 2 x - 1 that the cost of pollution control increases along with 1 2 the percentage of pollutants being removed. Suppose the x - 1 x - 2 43. f x = 44. f x = cost C, in dollars, of removing p% of the pollutants from x + 2 x + 1 a chemical spill is given by 1 2 2 1 2 2 x - 9 x - 4 48,000 45. f x = 46. f x = C p = . x + 1 x + 3 100 - p 1 2 1 2 1 2 x - 3 x + 1 a) Find C 0 , C 20 , C 80 , and C 90 . 47. f x = 48. f x = b) Find the domain of C. x2 2x 15 x2 2x 3 + - - - c) Draw a1 graph2 1 of 2C. 1 2 1 2 1 2 2 1 2 2 2x x + x - 2 d) Can the company or city afford to remove 100% of 49. f x = 2 50. f x = 2 the pollutants due to this spill? Explain. x - 16 2x - 2 1 2 1 2

Copyright 2016 Pearson. All rights reserved.

M02_BITT9391_11_SE_C02.3.indd 231 29/10/14 4:46 PM Not for Sale

232 Chapter 2 ● Applications of Differentiation

61. Purchasing power. The purchasing power of the U.S. a) Complete the following table, rounding to two dollar t years after 1990 can be modeled by the function decimal places: 1 P x = . Innings 1 + 0.0362x 2 1 Pitched, n 9 6 3 1 3 3 (Source1 2 : Based on data from the Consumer Price Index.) y Earned-Run Average, E 1.0 0.8 P(x) = ÐÐÐ1 ÐÐ 1 + 0.0362x b) The number of innings pitched n is equivalent to 0.6 the number of outs that a pitcher is able to get while pitching, divided by 3. For example, a pitcher who (in dollars) 0.4 1 gets just 1 out is credited with pitching 3 of an Purchasing power Purchasing 0.2 inning. Find lim E n . Under what circumstances nS0 might this limit be plausible? 0 5 10 15 20 25 30 x 1 2 Years since 1990 a) Find P 5 , P 10 , and P 25 . SyntheSiS b) When was the purchasing power $0.50? 64. Explain why a vertical asymptote is only a guide and is c) Find lim1 2 P x1 . 2 1 2 xS ∞ not part of the graph of a function. 1 2 Life and Physical Sciences 65. Using graphs and limits, explain how three types of asymptotes are used when graphing rational functions. 62. Medication in the bloodstream. After an injection, the amount of a medication A, in cubic centimeters (cc), in Find each limit, if it exists. 2 the bloodstream decreases with time t, in hours. Sup- -3x + 5 ͉ x ͉ pose that under certain conditions A is given by 66. lim 67. lim xS - ∞ 2 - x xS0 x A 0 3 3 A t = 2 , x + 8 -6x + 7x t + 1 68. lim 2 69. lim 2 xS - 2 x - 4 xS - ∞ 2x - 3x - 10 where1 2 A is the initial amount of the medication. Assume 0 3 that an initial amount of 100 cc is injected. x - 1 70. lim 2 a) Find A 0 , A 1 , A 2 , A 7 , and A 10 . xS1 x - 1 b) Find the maximum amount of medication in the bloodstream1 2 1over2 the1 2 interval1 2 0, ∞1). 2 c) Graph the function. TEcHnology connEcTion d) According to this function, does the3 medication ever com- pletely leave the bloodstream? Explain your answer. Graph each function using a graphing utility.

General Interest 2 1 71. f x = x + x2 63. Baseball: earned-run average. A pitcher’s earned-run 1 2 average (the average number of runs given up in every x 72. f x 9 innings, or 1 complete game) is given by = 2 x + 1 r 1 2 E = 9 # , 23 2 n x + 4x + x - 6 73. f x = 2 where r is the number of earned runs allowed in n innings. x - x - 2 1 2 3 2 If we set the number of earned runs allowed at 4 and x + 2x - 15x let n vary, we get a function given by 74. f x = 2 x - 5x - 14 4 E n = 9 # . 1 2 3 2 n x + 2x - 3x 75. f x = 2 1 2 x - 25 1 2 1 76. f x = - 2 x 1 2 Á Á

While pitching for the St. Louis Cardinals in 1968, Bob Gibson had an earned-run average of 1.12, a record low. Copyright 2016 Pearson. All rights reserved.

M02_BITT9391_11_SE_C02.3.indd 232 29/10/14 4:46 PM Not for Sale

2.4 ● Using Derivatives to Find Absolute Maximum and Minimum Values 233

77. Graph the function given by e) How close were your estimates of part (a)? Would you 2 have been able to identify the relative minimum point x - 3 f x = . without calculus? 2x - 4 a) 1Find2 any x-intercepts. In Exercises 81–86, determine a rational function that meets b) Find the y-intercept if it exists. the given conditions, and sketch its graph. c) Find any asymptotes. 81. The function f has a vertical asymptote at x = 2, a hori- 78. Graph the function given by zontal asymptote at y = -2, and f 0 = 0. 2 82. The function f has a vertical asymptote at x 0, a hori- x + 3x + 2 1 2 = f x = . zontal asymptote at y = 3, and f 1 = 2. 2 x - 3 a) 1Estimate2 lim f x and lim f x using the graph 83. The function g has vertical asymptotes1 2 at x = -1 and xS ∞ xS ∞ and input–output tables as- needed to refine your x = 1, a horizontal asymptote at y = 1, and g 0 = 2. estimates. 1 2 1 2 84. The function g has vertical asymptotes at x =1 -22 b) What appears to be the domain of the function? and x = 0, a horizontal asymptote at y = -3, and Explain. g 1 = 4. c) Find lim f x and lim f x . xS - 2- xS - 1+ 85. The1 2 function h has vertical asymptotes at x = -3 79. Not all asymptotes are linear. Use long division to find an 1 2 1 2 and x = 2, a horizontal asymptote at y = 0, and equation for the nonlinear asymptote that is approached h 1 = 2. by the graph of 1 5 86. The1 2 function h has vertical asymptotes at x = - 2 x + x - 9 1 f x = 3 . and x = 2, a horizontal asymptote at y = 0, and x + 6x h 0 = -3. Then1 2 graph the function and its asymptote. 1 2 80. Refer to the graph on p. 221. The function is given by 2 Answers to Quick Checks x - 1 f x = 2 . 1. The lines x = 0, x = 4, and x = -4 are vertical x + x - 6 asymptotes. 2. The lines x = -1 and x = 0 are vertical 1 2 2 a) Inspect the graph and estimate the coordinates of any asymptotes. 3. The line y = 15 is a horizontal asymp- extrema. tote. 4. The line y = 2x + 7 is a slant asymptote. b) Find f′ and use it to determine the critical values. 5. x-intercepts: 1, 0 , -1, 0 , 0, 0 ; y-intercept: 0, 0 (Hint: you will need the quadratic formula.) Round 6. y x = 1 the x-values to the nearest hundredth. 10 1 2 1 2 1 2 1 2 c) Graph f in the window 0, 0.2, 0.16, 0.17 . Use trace 5 or maximum to confirm your results from part (b). d) Graph f in the window 39.8, 10, 0.9519, 0.951954 . −10 −5 5 10 x Use trace or minimum to confirm your results from −5 x2 − 9 f(x) = 3 4 x − 1 part (b). −10 y = x + 1

Using Derivatives to Find Absolute Maximum and Minimum Values 2.4 If an extremum is the highest or lowest value over a function’s entire domain, it is 2 called an absolute extremum. For example, the parabola given by f x = x + 1 has ● Find absolute extrema using a relative minimum at 0, 1 . This is also the lowest point for the entire graph of f, so Maximum–Minimum Principle 1. it is also the absolute minimum. In applications, we are often interested1 2 in absolute 1 2 ● Find absolute extrema using extrema. Maximum–Minimum Principle 2. Absolute Maximum and Minimum Values A relative minimum may or may not be an absolute minimum, meaning the smallest value of the function over its entire domain. Similarly, a relative maximum may or may not be an absolute maximum, meaning the greatest value of a function over its entire domain.

Copyright 2016 Pearson. All rights reserved.

M02_BITT9391_11_SE_C02.3.indd 233 29/10/14 4:47 PM Not for Sale

234 Chapter 2 ● Applications of Differentiation

The function in the following graph has relative minima at c1 and c3.

y Absolute Relative maximum at c Maxima 2 maximum at b

Absolute Relative minimum minima at c1 at c1 and c3

Minima

x a c1 c2 c3 b

The relative minimum at c1 is also the absolute minimum. On the other hand, the relative maximum at c2 is not the absolute maximum. The absolute maximum occurs at b.

Definition Suppose that f is a function with domain I. f c is an absolute minimum if f c … f x for all x in I. f c is an absolute maximum if f c Ú f x for all x in I. 1 2 1 2 1 2 1 2 1 2 1 2 Finding Absolute Maximum and Minimum Values over Closed Intervals Let’s consider continuous functions for which the domain is a closed interval. Look at the graphs of f and g below, and try to determine where the absolute maxima and minima (extrema) occur for each function.

y y

f g

a c c b x x 1 2 a c1 c2 b

Note that each function does indeed have an absolute maximum value and an abso- lute minimum value. This leads us to the following theorem.

theorem 7 The Extreme-Value Theorem A continuous function f defined over a closed interval a, b must have an absolute maximum value and an absolute minimum value over a, b . 3 4 3 4

Copyright 2016 Pearson. All rights reserved.

M02_BITT9391_11_SE_C02.4.indd 234 29/10/14 4:51 PM Not for Sale

2.4 ● Using Derivatives to Find Absolute Maximum and Minimum Values 235

Look again at f and g on the previous page and consider the critical values and the endpoints. The graph of f starts at f a , falls to f c1 , rises from f c1 to f c2 , and falls to f b . The graph of g starts at g a , rises to g c1 , falls from g c1 to g c2 , and rises to g b . It seems reasonable that1 2whatever the1 maximum2 and 1minimum2 1 values2 are, they1 occur2 at a critical value or endpoint.1 2 1 2 1 2 1 2 1 2

theorem 8 Maximum–Minimum Principle 1 Suppose f is a continuous function defined over a closed interval a, b . To find the absolute maximum and minimum values over a, b : 3 4 a) First find f′ x . 3 4 b) Then determine all critical values in a, b . That is, find allc in a, b for which 1 2 3 4 3 4 f′ c = 0 or f′ c does not exist. c) List the1 2values from step1 2(b) and the endpoints of the interval:

a, c1, c2, c, cn, b. d) Evaluate f x for each value in step (c):

f a , f 1c12 , f c2 , c , f cn , f b . The largest1 2 of1 these2 1 is 2the absolute1 2 maximum1 2 of f over a, b . The smallest of these is the absolute minimum of f over a, b . 3 4 3 4

eXAmPLe 1 Find the absolute maximum and minimum values of 3 f x = x - 3x + 2 3 over the interval1 2 -2, 2 . 3 Solution Keep in mind that we are considering only the interval -2, 2 . 3 4 2 a) Find f′ x : f′ x = 3x - 3. b) Find the critical values. The derivative exists for all real numbers.3 Thus,4 we solve 1 2 1 2 f′ x = 0: 2 1 2 3x - 3 = 0 3x2 = 3 x 2 = 1 y f(x) = x3 − 3x + 2 Absolute x = {1. maximum = 4, 5 4 3 occurs at c) List the critical values and the endpoints: -2, -1, 1, and 2. x = −1 3 2 d) Evaluate f x at each value in step (c): 1 3 f -21 2= -2 - 3 -2 + 2 = -8 + 6 + 2 = 0; −3 −2 −1 1 3 2 3 x 3 −1 2 f -1 = -1 - 3 -1 + 2 = -1 + 3 + 2 = 4; −2 1 2 1 2 1 2 Absolute f 1 1 3 3 1 2 1 3 2 0; −3 minimum = 0, 1 2= 1 -2 1+ 2= - + = −4 occurs at 3 3 3 3 27 9 7 f 2 = 2 - 3 2 + 2 = 8 - 2 + 2 = 8 −5 x = −2 and x = 1 1 2 1 2 1 2 The largest1 2 of these1 2 values,1 2 4, is the maximum. It occurs at x = -1. The smallest of these 3 A visualization of Example 1 values is 0. It occurs twice: at x = -2 and x = 1. Thus, over the interval -2, 2 , the absolute maximum = 4 at x = -1 3 4 and the absolute minimum = 0 at x = -2 and x = 1.

Copyright 2016 Pearson. All rights reserved.

M02_BITT9391_11_SE_C02.4.indd 235 29/10/14 4:51 PM Not for Sale

236 Chapter 2 ● Applications of Differentiation

As we see in Example 1, an absolute maximum or minimum value can occur at more than one point.

TEChnology ConnECTIon Finding Absolute Extrema Method 4 To find the absolute extrema in Example 1, we can use any of Example 2 considers the same function as in Example 1, the methods described in the Technology Connection on but over a different interval. Because there are no relative p. 200. In this case, we adapt Methods 3 and 4. extrema, we can use fMax and fMin from the math menu. The minimum and maximum values occur at the endpoints, as Method 3 the following graphs show. Method 3 is selected because there are relative extrema in 3 fMin(Y1,X,–3,–1.5) fMax(Y1,X,–3,–1.5) the interval -2, 2 . This method gives approximations of –2.999994692 –1.500005458 these relative extrema. Y1(Ans) Y1(Ans) 3 4 –15.99987261 3.124979532 y = x3 ± 3x + 2 y = x3 ± 3x + 2

ExErCIsE 1. Use a graph to estimate the absolute extrema of Maximum Minimum f x x3 x2 x 2, first over the interval 2, 1 X = –1.000002 Y = 4 X = .99999689 Y = 2.9E–11 = - - + - and then over -1, 2 . Then check your work using the methods1 2 of Examples 1 and 2. 3 4 Next, we check function values at these x-values and at the end- 3 4 points, using Maximum–Minimum Principle 1 to determine 3 the absolute maximum and minimum values over -2, 2 .

XY1 3 4 –2 0 Min –1 4 Max 1 0 Min 1.5 .875

X =

1 Maximum = 3−8, eXAmPLe 2 Find the absolute maximum and minimum values of occurs at 3 x = −−3 y f(x) = x − 3x + 2 3 2 f x = x - 3x + 2 4 3 2 over -3,1 -22 . −3 −2 3 −1 1 2 3 − x 2 −2 Solution As in Example 1, the derivative is 0 at -1 and 1. But neither -1 nor 1 is in −4 3 4 3 the interval -3, - 2 , so there are no critical values in this interval. Thus, the maxi- −6 mum and minimum values occur at the endpoints: −8 3 −10 f -33 = -4 3 - 3 -3 + 2 −12 27 9 2 16; −14 1 2 = 1- 2+ +1 =2 - −16 3 3 3 3 −18 f - = - - 3 - + 2 2 2 2 Minimum = −16, occurs at x = −3 a b a 27b 9 a b25 1 = - + + 2 = = 3 . A visualization of Example 2 8 2 8 8 3 1 3 Thus, the absolute maximum over -3, - , is 3 , which occurs at x = - , and the Quick Check 1 2 8 2 absolute minimum is -16, which occurs at x = -3. 1 Find the absolute maximum 3 4 and minimum values of the function given in Example 2 over the interval 0, 3 . Copyright 2016 Pearson. All rights reserved. 3 4

M02_BITT9391_11_SE_C02.4.indd 236 29/10/14 4:51 PM Not for Sale

2.4 ● Using Derivatives to Find Absolute Maximum and Minimum Values 237

Finding Absolute Maximum and Minimum Values over other Intervals When there is only one critical value c in I, we may not need to check endpoint values to determine whether the function has an absolute maximum or minimum value at that point.

theorem 9 Maximum–Minimum Principle 2 Suppose f is a function such that f′ x exists for every x in an interval I and there is exactly one (critical) value c in I, for which f′ c = 0. Then 1 2 f c is the absolute maximum value over I if f ″1c2 6 0 or 1 2 1 2 f c is the absolute minimum value over I if f ″ c 7 0. 1 2 1 2 Theorem 9 holds no matter what the interval I is—whether open, closed, or infi- nite in length. If f″ c = 0, either we must use Maximum–Minimum Principle 1 or we must know more about the behavior of the function over the given interval. 1 2 2 TEChnology ConnECTIon eXAmPLe 3 Find the absolute extrema of f x = 4x - x . Solution When no interval is specified, we 1consider2 the entire domain of the func- Finding Absolute Extrema tion. In this case, the domain is the set of all real numbers. Let’s do Example 3 graphically, a) Find f′ x : by adapting Methods 1 and 2 of the Technology Connection on f′1x2 = 4 - 2x. p. 200. Strictly speaking, we can- b) Find the1 critical2 values. The derivative exists for all real numbers. Thus, we need not use the fMin or fMax option only solve f′ x = 0: from the math menu or the maxi- mum or minimum option from the 4 - 2x1=2 0 calc menu since we do not have a -2x = -4 closed interval. x = 2. Methods 1 and 2 c) Since there is only one critical value, we apply Maximum–Minimum Principle 2 We create a graph, examine its using the second derivative: shape, and use trace and/or table. This procedure leads us to see f ″ x = -2. that there is indeed no absolute minimum. We do find an absolute The second1 2 derivative is constant. Thus, f ″ 2 = -2, and since this is negative, we maximum: f x = 4 at x = 2. have the absolute maximum: 1 2 f 2 = 4 # 2 - 22, ExErCIsE 1 2 8 4 4. 1. Use a graph to estimate 1 2 = - = the absolute maximum The function has no minimum, as the graph, shown below, indicates. and minimum values of f x = x2 - 4x. Then check y your work using the method 5 1 2 of Example 3. Max = 4 f(x) = 4x ±x 2 3 2 1

±5 ±4 ±3 ±2 ±1 1 2 345 x ±1

Copyright 2016 Pearson. All rights reserved.

M02_BITT9391_11_SE_C02.4.indd 237 29/10/14 4:51 PM Not for Sale

238 Chapter 2 ● Applications of Differentiation

eXAmPLe 4 Find the absolute extrema of f x = 4x - x2 over the interval 1, 4 . Solution From Example 3, we know that the1 absolute2 maximum of f on - ∞,3 ∞ is4 f 2 , or 4. Since 2 is in 1, 4 , we know that the absolute maximum of f over 1, 4 will occur at 2. To find the absolute minimum, we need to check the endpoints:1 2 1 2 3 4 3 4 f 1 = 4 # 1 - 12 = 3 y Maximum = 4 at x = 2 4 and 1 2 3 f(x) = 4x ±x 2 over [1, 4] 2 f 4 = 4 # 4 - 4 = 0. 2 1 We also see1 2from the graph that the minimum Minimum = 0 at x = 4 Quick Check 2 is 0. It occurs at x = 4. Thus, the 01234x Find the absolute extrema of absolute maximum = 4 at x = 2, f x = x2 - 10x over each interval: and the 1 2 a) 0, 6 ; b) 4, 10 . absolute minimum = 0 at x = 4. 2

3 4 3 4 A strategy for Finding Absolute Maximum and Minimum Values The following general strategy can be used when finding absolute maximum and min- imum values of continuous functions.

A Strategy for finding Absolute maximum and minimum Values To find the absolute extrema of a continuous function over an interval: a) Find f′ x . b) Find the critical values. c) If the interval1 2 is closed and there is more than one critical value, use Maximum–Minimum Principle 1. d) If the interval is closed and there is exactly one critical value, use either Maximum–Minimum Principle 1 or Maximum–Minimum Principle 2. If it is easy to find f ″ x , use Maximum–Minimum Principle 2. e) If the interval is not closed, such as - ∞, ∞ , 0, ∞ , or a, b , and the function has only1 one2 critical value, use Maximum–Minimum Principle 2. In such a case, if the function has a maximum,1 2 1 it will2 have1 no2 minimum; and if it has a minimum, it will have no maximum.

Finding absolute maximum and minimum values when more than one critical value occurs in an interval that is not closed, such as any of those listed in step (e) above, requires a detailed graph or techniques beyond the scope of this book.

eXAmPLe 5 Find the absolute maximum and minimum values of f x = x - 2 3 + 1. Solution1 2 1 2 a) Find f′ x . 2 f′1x2 = 3 x - 2 . 1 2 1 2

Copyright 2016 Pearson. All rights reserved.

M02_BITT9391_11_SE_C02.4.indd 238 29/10/14 4:51 PM Not for Sale

2.4 ● Using Derivatives to Find Absolute Maximum and Minimum Values 239

y b) Find the critical values. The derivative exists for all real numbers. Thus, we solve 10 f′ x = 0: 9 3 2 8 f(x) = (x ± 2) + 1 1 2 3 x - 2 = 0 2 7 x - 2 = 0 6 1 2 x 2 0 5 1 -2 = 4 x = 2. 3 2 c) Since there is only one critical value and there are no endpoints, we can try to 1 apply Maximum–Minimum Principle 2 using the second derivative: ±1 x = 2 4 x ±1 f ″ x = 6 x - 2 . A visualization of Example 5 We have1 2 1 2 f ″ 2 = 6 2 - 2 = 0, so Maximum–Minimum1 2 1 2 Principle 2 does not apply. We cannot use Maximum– Quick Check 3 Minimum Principle 1 because there are no endpoints. But note that f′ x = n Let f x = x , where n is a 3 x - 2 2 is never negative. Thus, f x is increasing everywhere except at x = 2, positive odd integer. Explain so there is no maximum and no minimum. For x 6 2, say x = 1, we1 have2 1 2 why functions of this form never f 1″ 1 =2-6 6 0. For x 7 2, say x =1 23, we have f ″ 3 = 6 7 0. Thus, at x = 2, have an absolute minimum or the function has a point of inflection. 3 maximum. 1 2 1 2 eXAmPLe 6 Find the absolute maximum and minimum values of TEChnology ConnECTIon 35 f x = 5x + Finding Absolute Extrema x over the interval1 2 0, ∞ . Let’s do Example 6 using maximum and minimum from the Solution 1 2 calc menu. The shape of the a) Find f′ x . Since f x 5x 35x-1, we have graph leads us to see that there = + -2 is no absolute maximum, but f ′1x2 = 5 - 1352x there is an absolute minimum. 35 1 2 = 5 - 2. Y1=5X+(35/X) x b) Find the critical values. Since f′ x exists for all values of x in 0, ∞ , the only critical values are those for which f′ x = 0: 1 2 1 2 Minimum 35 1 2 X = 2.6457515 Y = 26.457513 5 - = 0 Setting f′ x equal to 0 x2 [0, 10, 0, 50] 35 1 2 Since 5 = x2 7 ≈ 2.6458, 5x2 = 35 Multiplying both sides by x2, since x  0 2 the results in the window confirm x 2 = 7 the solution found in Example 6. x = 7. Taking the positive root only, since the negative root is not in 0, ∞ ExErCIsE 2 1. Use a graph to estimate c) Since 0, ∞ is not closed 1and 2there is only one critical value, we can apply the absolute extrema of Maximum–Minimum Principle 2 using the second derivative, f x 10x 1 x over 1 2 = + 70 0, ∞ . Then check your f ″ x = 70x-3 = . work1 2 using the method> of x3 Example1 2 6. 1 2 Since 70 f ″ 7 0, = 3 7 2 7 1 2 2 an absolute minimum1 occurs2 at x = 7: Copyright 2016 Pearson. All rights reserved. 2

M02_BITT9391_11_SE_C02.4.indd 239 29/10/14 4:51 PM Not for Sale

240 Chapter 2 ● Applications of Differentiation

Absolute minimum = f 7 2 1 2 35 = 5 # 7 + 2 7 352 7 = 5 7 + # 2 2 7 7 352 72 = 5 7 + 72 2 = 5 7 + 5 7 = 102 7 ≈ 226.458 at x = 7. The function has no maximum2 value, which can2 happen since the interval 0, ∞ is not closed. y 1 2 40 35 30 Quick Check 4 Min = 10√7 Find the absolute maximum and 35 20 f(x) = 5x + Ðx minimum values of 15 2x2 + 18 10 g x = x 5

over the1 interval2 0, ∞ . 10 √7 456 x 4

1 2 Section Summary ● An absolute minimum of f is a value f c such that ● Endpoints of a closed interval may be absolute extrema, f c … f x for all x in the domain of f. but not relative extrema. ● An absolute maximum of f is a value f1 c2 such that ● If there is exactly one critical value c in the domain of f f1c2 Ú f1x2 for all x in the domain of f. such that f′ c = 0, then Maximum–Minimum Principle ● If the domain of f is a closed interval 1and2 f is continu- 2 may be used to find the absolute extrema. Otherwise, ous1 2 over 1that2 domain, then the Extreme–Value Theorem Maximum–Minimum1 2 Principle 1 has to be used. guarantees the existence of both an absolute minimum and an absolute maximum.

2.4 Exercise Set

1. Fuel economy. According to the U.S. Department 35 of Energy, a vehicle’s fuel economy, in miles per 30 gallon (mpg), decreases rapidly for speeds over 25 60 mph. 20 a) Estimate the speed at which the absolute maximum 15 gasoline mileage is obtained. 10 b) Estimate the speed at which the absolute minimum 5 Fuel economy (mpg) gasoline mileage is obtained. 0 c) What is the mileage obtained at 70 mph? 5415 25 35 5556575 Speed (mph) 2. Fuel economy. Using the graph in Exercise 1, estimate (Sources: U.S. Dept. of Energy; a study by West, B.H., McGill, the absolute maximum and the absolute minimum fuel R.N., Hodgson, J.W., Sluder, S.S., and Smith, D.E., Oak Ridge economy over the interval 30, 70 . National Laboratory, 1999; www.mpgforspeed.com, 2014.)

3 4 Copyright 2016 Pearson. All rights reserved.

M02_BITT9391_11_SE_C02.4.indd 240 29/10/14 4:51 PM Not for Sale Exercise Set 2.4 241

Find the absolute maximum and minimum values of each func- 9. f x = 2x + 4; -1, 1 tion over the indicated interval, and indicate the x-values at which they occur. 10. f1x2 = 5x - 7; 3-2, 34 2 11. f x 7 4x; 2, 5 3. f x = 5 + x - x ; 0, 2 1 2 = - 3- 4 12. f x 2 3x; 10, 10 1 2 y3 4 1 2 = - - 3 - 4 6 13. f x 5; 1, 1 5 1 2 = - - 3 4 2 4 14. f1x2 = x - 63x - 3;4 -1, 5 3 15. g x 24; 4, 13 2 1 2 = 3 4 2 1 16. f1x2 = 3 - 23x - 54x ; -3, 3 ±3 ±2 ±1 132 4 x 17. f x x2 4x 5; 1, 3 ±1 1 2 = - + -3 4 3 2 18. f1x2 = x - 3x ; 0, 53 4 2 2 4. f x = 4 + x - x ; 0, 2 19. f1x2 = 1 + 6x - 3x3 ; 40, 4 3 1 2 y3 4 20. f1x2 = x - 3x + 6; -3 1, 34 3 5 21. f1x2 = x - 3x; -5, 31 4 4 22. f x 3x2 2x3; 5, 1 3 1 2 = - 3 - 4 3 2 23. f1x2 = 1 - x ; -8,3 8 4 1 3 24. f1x2 = 2x ; -10,3 10 4 ±33±2 ±1 21 4 x 3 2 ±1 25. f1x2 = x - 63x + 10;4 0, 4 ± 2 2 3 26. f1x2 = 12 + 9x - 3x - 3x ; 4 -3, 1 3 1 2 3 4 5. f x = x - 2x - 2x + 5; -2, 1 27. f1x2 = x - x ; -1, 1 3 4 4 3 1 2 y 3 4 28. f1x2 = x - 2x ; 3 -2, 42 4 2 7 29. f1x2 = x - 2x + 35; -42, 2 6 4 2 5 30. f1x2 = x - 8x + 3; 3-3, 34 2 3 4 31. f1x2 = 1 - x ; -8, 38 4 3 >2 3 2 32. f1x2 = x + 3 -3 5; 4 -4, 5 > 1 1 2 1 4 2 3 4 33. f x = x + ; -8, -1 −2 −1 21 x x −1 1 2 1 3 4 34. f x = x + ; 1, 20 x 6. f x = x3 - x2 - x + 2; -1, 2 1 2 x2 3 4 1 2 y 3 4 35. f x = ; -2, 2 x2 + 1 5 1 2 3 4 4 4x 36. f x = 2 , -3, 3 3 x + 1 2 1 2 3 4 37. f x = x + 1 1 3; -2, 26 1 3 > 38. f1x2 = 1 x; 28, 64 3 4 −3 −2 −1 21 3 x −1 2 39–48.1 2 Check Exercises3 43, 5, 9, 13, 19, 23, 33, 35, 37, and −2 38 with a graphing calculator.

3 1 2 7. f x = x + 2x - 2x + 4; -2, 0 3 2 8. f1x2 = x - x - x + 3; -31, 0 4 1 2 3 4 Copyright 2016 Pearson. All rights reserved.

M02_BITT9391_11_SE_C02.4.indd 241 29/10/14 4:51 PM Not for Sale

242 Chapter 2 ● Applications of Differentiation

Find the absolute extrema of each function, if they exist, over 81. g x = x2 3 the indicated interval. Also indicate the x-value at which each 1 3> 2 extremum occurs. When no interval is specified, use the real 82. f1x2 = 3x - 2x + x; 0, 4 numbers,  H, H . 1 3 1 2 83. f1x2 = 3x - 2x - 2x +31 4 2 49. f x 1 = 30x -2 x 1 3 2 84. g1x2 = 3x + 2x + x; -4, 0 2 50. f1x2 = 12x - x 4 2 85. t 1x 2 = x - 2x 3 4 2 51. f1x2 = 2x - 40x + 270 4 2 86. f1x2 = 2x - 4x + 2 2 52. f1x2 = 2x - 20x + 340 87–96.1 2Check Exercises 49, 51, 53, 57, 61, 65, 67, 69, 73, and 4 3 53. f1x2 = 16x - 3x ; 0, ∞ 85 with a graphing calculator. 4 3 54. f1x2 = x - 3x ; 0,1 ∞ 2 APPLicAtionS 55. f1x2 = x 60 - x 1 2 Business and Economics 56. f x x 25 x 1 2 = 1 - 2 97. Monthly productivity. An employee’s monthly produc- 1 3 tivity M, in number of units produced, is found to be a 57. f1x2 = 3x1 - 5x;2 -3, 3 function of t, the number of years of employment. For a 1 3 58. f1x2 = 3x - 3x; 3-2, 24 certain product, a productivity function is given by 2 2 59. f1x2 = -0.001x +34.8x -4 60 M t = -2t + 100t + 180, 0 … t … 40. 2 Find the maximum productivity and the year in which 60. f1x2 = -0.01x + 1.4x - 30 1 2 it is achieved. 3 2 61. f1x2 = -x + x + 5x - 1; 0, ∞ 98. Advertising. Sound Software estimates that it will sell 1 3 2 62. f1x2 = - 3x + 6x - 11x - 50;1 20, 3 N units of a program after spending a dollars on adver- 2 1 3 tising, where 63. f1x2 = 15x - 2x ; 0, 30 1 2 2 N a = -a + 300a + 6, 0 … a … 300, 64. f x 4x2 1x3; 0, 8 1 2 = - 2 3 4 and a is in thousands of dollars. Find the maximum 1 2 1 2 72 3 4 number of units that can be sold and the amount that 65. f x = 2x + ; 0, ∞ x must be spent on advertising in order to achieve that maximum. 1 2 3600 1 2 66. f x = x + ; 0, ∞ 99. Unemployment. The percentage of unemployed work- x ers in service occupations can be modeled by 1 2 1 2 3 2 2 432 p x = 0.039x 0.594x 1.967x 7.555, 67. f x = x + ; 0, ∞ - + - + x where x is the number of years since 2003. (Source: 1 2 1 2 1 2 Based on data from www.bls.gov.) According to this 2 250 68. f x = x + ; 0, ∞ x model, in what year during the period 2003–2013 was this percentage a maximum? 1 2 1 2 69. f x = 2x4 + x; -1, 1 100. labor. The percentage of women aged 21–54 in the 4 70. f1x2 = 2x - x; 3-1, 14 U.S. civilian labor force can be modeled by 2 3 f x = 0.0135x 0.265x 74.6, 71. f1x2 = x; 0, 83 4 - + + where x is the number of years since 1992. (Source: 2 1 2 72. f1x2 = x; 30, 44 based on data from www.bls.gov.) According to this 2 3 model, in what year during the period 1992–2012 was 73. f1x2 = x - 13 4 this percentage a maximum? 3 74. f1x2 = 1x + 12 101. Worldwide oil production. One model of worldwide oil 75. f1x2 = 12x - 3;2 -1, 1 production is 4 3 2 76. f1x2 = 9 - 5x; 3-10, 104 P t = 2.69t - 63.941t + 459.895t - 688.692t + 24,150.217, where P t is the number of barrels, in thousands, pro- 77. f x = 2x 3; 1, 5) 1 2 1 2 - 3- 4 duced t years after 2000. (Source: Based on data from 2 3 1 2 78. f1x2 = x ; -1,3 1 the U.S. Energy Information Administration.) Accord- > ing to this model, in what year did worldwide oil pro- 79. f1x2 = 9 - 5x;3 -2,4 3) duction achieve an absolute minimum? What was that 1 3 2 minimum? 80. f1x2 = 3x - x +33

1 2 Copyright 2016 Pearson. All rights reserved.

M02_BITT9391_11_SE_C02.4.indd 242 29/10/14 4:51 PM Not for Sale Exercise Set 2.4 243

102. Maximizing profit. Corner Stone Electronics deter- 112. Physical science: dry lake elevation. Dry lakes are mines that its weekly profit, in dollars, from the pro- common in the Western deserts of the United States. duction and sale of x amplifiers is These beds of ancient lakes are notable for having per- fectly flat terrain. Rogers Dry Lake in California has 1500 been used as a landing site for space shuttle missions in P x = 2 . x - 6x + 10 recent years. The graph shows the elevation E, in feet, Find1 the2 number of amplifiers, x, for which the total as a function of the distance x, in miles, from a point weekly profit is a maximum. near one side x = 0 of Rogers Dry Lake to a point near the other side. (Source: www.mytopo.com.) Maximizing profit. The total-cost and total-revenue 1 2 functions for producing x items are E 1 2295 C x  5000  600x and R x   x2  1000x, 2 2290 1 2 1 2 where 0 " x " 600. Use these functions for Exercises 103 2285 and 104. 2280 103. a) Find the total-profit function P x . 2275 b) Find the number of items, x, for which total profit is Elevation (in feet) a maximum. 1 2 2270 2265 104. a) Average profit is given by A x = P x x. Find A x . 0 1 2 3 4 5 6 x b) Find the number of items, x, for which average Distance (in miles) profit is a maximum. 1 2 1 2> 1 2 Life and Physical Sciences 105. Blood pressure. For a dosage of x cubic centimeters (cc) of a certain drug, the resulting blood pressure B is approximated by 2 3 B x = 305x - 1830x , 0 … x … 0.16. Find the maximum blood pressure and the dosage at 1 2 which it occurs.

SyntheSiS 106. How is the second derivative useful in finding the absolute extrema of a function? For Exercises 107–110, find the absolute maximum and mini- a) What is the maximum elevation? mum values of each function, and sketch the graph. b) What is the minimum elevation? 2x + 1 for -3 … x … 1, 107. f x = 2 4 - x , for 1 6 x … 2 Find the absolute maximum and minimum values of the function, if they exist, over the indicated interval. 1 2 2 x , for -2 … x … 0, 108. g x = 113. g x = x x + 3; -3, 3 5x, for 0 6 x … 2 2 1 2 b 2 114. h1x2 = x 1 - x; 30, 1 4 1 - x , for -4 … x 6 0, 2 109. h x = 1 - x, for 0 … x 6 1, 115. Business:1 2 total cost. Certain3 4 costs in business can be b x - 1, for 1 … x … 2 separated into two components: those that increase 1 2 with volume and those that decrease with volume. For 2 x + 4, for -2 … x 6 0, example, customer service becomes more expensive 110. F x = 4 - x, for 0 … x 6 3, as its quality increases, but part of the increased cost c is offset by fewer customer complaints. Katie’s Clocks x - 2, for 3 … x … 67 1 2 determines that its cost of service, C x , in thousands 2 111. Consider the piecewise-defined function f defined by: of dollars, is modeled by 2 1 2 x + 2, for -2 … x … 0, 2 c C x = 2x + 4 + , x 7 6, f x = 2, for 0 6 x 6 4, x - 6 x - 2, for 4 … x … 6. where1 2 x represents1 2 the numbera b of “quality units.” Find 1 2 a) Sketch its graph. the number of “quality units” that the firm should use b) Identify the absolute maximum. in order to minimize its total cost of service. c) What is thec absolute minimum? Copyright 2016 Pearson. All rights reserved.

M02_BITT9391_11_SE_C02.4.indd 243 29/10/14 4:51 PM Not for Sale

244 Chapter 2 ● Applications of Differentiation

116. Let 123. life and physical sciences: contractions during 2 2 pregnancy. The following table gives the pressure y = x - a + x - b . of a pregnant woman’s contractions as a function of For what value of x is y a minimum? 1 2 1 2 time. 117. Business: worldwide oil production. Refer to Exercise 101. In what year(s) during the period TimE, t PrESSurE 2000–2008 was worldwide oil production increasing (in minutes) (in millimeters of mercury) most rapidly, and at what rate was it increasing? 0 10 118. How is the first derivative useful in finding the abso- 1 8 lute extrema of a function? 2 9.5 119. Business: U.s. oil production. One model of oil pro- 3 15 duction in the United States is given by 4 12 4 3 2 P t = 0.0000000219t - 0.0000167t + 0.00155t 5 14 + 0.002t + 0.22, 0 … t … 110, 6 14.5 1 2 where P t is the number of barrels of oil, in billions, produced in a year, t years after 1910. (Source: Beyond Use a calculator that has the regression option. Oil, by Kenneth1 2 S. Deffeyes, p. 41, Hill and Wang, New York, 2005.) a) Fit a linear equation to the data. Predict the pressure of the contractions after 7 min. a) According to this model, what is the absolute b) Fit a quartic polynomial to the data. Predict the maximum amount of oil produced in the United pressure of the contractions after 7 min. Find the States and in what year did that production smallest contraction over the interval 0, 10 . occur? b) According to this model, at what rate was United 3 4 States oil production declining in 2010 and in 2015? Graph each function over the given interval. Visually estimate Answers to Quick Checks where any absolute extrema occur. Then use the table feature 1. Absolute maximum is 20 at x = 3; absolute mini- to refine each estimate. mum is 0 at x = 1. 2. (a) Absolute maximum is 0 at 2 3 x = 0; absolute minimum is -25 at x = 5. (b) Absolute 120. f x = x x - 5 ; 1, 4 maximum is 0 at x = 10; absolute minimum is -25 at > x 5. 3. The derivative is f′ x nxn - 1. If n is odd, 1 2 3 21 22 3 3 1 4 = = 121. f x = x - 1 ; , ∞ n - 1 4 2 n - 1 is even. Thus, nx is always positive or zero, > never negative. 4. No absolute1 maximum;2 absolute mini- 1 2 1 24 x 3 2 mum is 12 at x = 3. 122. f x = x - 5 ; ℝ 2 1 2 a b

maximum–minimum Problems; Business, Economics, and General Applications 2.5 An important use of calculus is solving maximum–minimum problems, that is, find- ing the absolute maximum or minimum value of some variable. ● Solve maximum–minimum problems using calculus. eXAmPLe 1 Maximizing Area. A hobby store has 20 ft of fencing to fence off a rectangular area for an electric train in one corner of its display room. The two sides up against the wall require no fence. What dimensions of the rectangle will maximize the area? What is the maximum area?

Copyright 2016 Pearson. All rights reserved.

M02_BITT9391_11_SE_C02.4.indd 244 29/10/14 4:52 PM Not for Sale

2.5 ● Maximum–Minimum Problems; Business, Economics, and General Applications 245

Solution Let’s first make a drawing and express the area using one variable. If we let x = the length, in feet, of one side and y = the length, in feet, of the other, then, since the sum of the lengths must be 20 ft, we have x + y = 20 and y = 20 - x. Thus, the area is given by A = xy = x 20 - x 20x x2. x = 1 - 2 A A(x) = 20x – x2 We are trying to find the maxi- 100 mum value of 80 A x 20x x2 60 = - y 5 20 2 x 40 over the 1interval2 0, 20 . We 20 consider the interval 0, 20 Area (in square feet) (in square Area because x is a length 1and cannot2 be negative or 0. Since there is only 20 ft of fencing, x 5101520 x cannot be greater than 20. 1Also, x2 cannot be 20 because then the length of y would be 0. Length (in feet) a) We first find A′ x : A′ x = 20 - 2x. b) This derivative exists for all values of x in 0, 20 . Thus, the only critical values are 1 2 1 2 where 1 2 A ′ x = 20 - 2x = 0 2x 20 1 2 - = - x = 10. Since there is only one critical value, we can use the second derivative to determine whether we have a maximum. Note that A″ x = -2, Quick Check 1 Since A 10 is negative, A 10 is a maximum. Now Repeat Example 1 starting first ″1 2 with 50 ft of fencing, and then A 101 2= 10 20 - 101 2 with 100 ft of fencing. Do you = 10 # 10 detect a pattern? If you had n feet 1 2 1 2 100. of fencing, what would be the = dimensions of the maximum area Thus, the maximum area of 100 ft2 is obtained using 10 ft for the length of one side (in terms of n)? and 20 - 10, or 10 ft for the other. 1

Here is a general strategy for solving maximum–minimum problems.

A Strategy for Solving Maximum–Minimum Problems 1. Read the problem carefully. If relevant, make a drawing. 2. Make a list of appropriate variables and constants, noting what varies, what stays fixed, and what units are used. Label the measurements on your drawing, if one exists. 3. Translate the problem to an equation involving a quantity Q to be maximized or minimized. Try to represent Q in terms of the variables of step 2. 4. Try to express Q as a function of one variable. Use the procedures developed in Sections 2.1–2.4 to determine the maximum or minimum values and the points at which they occur.

Copyright 2016 Pearson. All rights reserved.

M02_BITT9391_11_SE_C02.5.indd 245 29/10/14 4:56 PM Not for Sale

246 Chapter 2 ● Applications of Differentiation

EXAMPLE 2 Maximizing Volume. From a thin piece of cardboard 8 in. by 8 in., square corners are cut out so that the sides can be folded up to make a box. What di- mensions will yield a box of maximum volume? What is the maximum volume? Solution We make a drawing in which x is the length, in inches, of each square to be cut. It is important to note that since the original square is 8 in. by 8 in., after the smaller squares are removed, the lengths of the sides of the box will be 8 - 2x in. by 8 - 2x in. 1 2 1 2 x x xx

8 in. 8 − 2x x x 8 − 2x x 8 − 2x x x 8 − 2x

8 in.

After the four small squares are removed and the sides are folded up, the volume V of the resulting box is V = l # w # h = 8 - 2x # 8 - 2x # x, or V x 4x3 32x2 64x. = - 1 + 2 1 2 Since 18 2- 2x 7 0, this means that x 6 4. Thus, we need to maximize 3 2 V x = 4x - 32x + 64x over the interval 0, 4 . To do so, we1 2 first find V′ x : 1 2 2 V′ x = 12x -1 642 x + 64. Since V′ x 1 exists2 for all x in the interval 0, 4 , we can set it equal to zero to find the critical values: 1 2 1 2 2 12x - 64x + 64 = 0 2 4 3x - 16x + 16 = 0 4 3x 4 x 4 0 1 - - 2 = 3x 4 0 or x 4 0 1 -21 = 2 - = 3x = 4 or x = 4 4 x = 3 or x = 4. 4 The only critical value in 0, 4 is 3. Thus, we can use the second derivative, V″ x = 24x - 64,1 2 to determine1 2whether we have a maximum. Since 4 4 V″ 3 = 24 # 3 - 64 = 32 - 64 6 0, 4 we know that1 2 V 3 is a maximum. 4 Thus, to maximize the box’s volume, small squares with edges measuring 3 in., or 1 1 2 13 in., should be cut from each corner of the original 8 in. by 8 in. piece of cardboard. Quick Check 2 When the sides are folded up, the resulting box will have sides of length Repeat Example 2 starting with 4 24 8 16 1 a sheet of cardboard measur- 8 - 2x = 8 - 2 # = - = = 5 in. ing 8.5 in. by 11 in. (the size of 3 3 3 3 3 a typical sheet of paper). Will 1 and a height of 13 in. The maximum volume is this box hold 1 liter (L) of liq- 3 2 uid? (Hint: 1 L 1000 cm3 and 4 4 4 4 1024 25 3 = V = 4 - 32 + 64 = = 37 in . 1 in3 = 16.38 cm3.) 3 3 3 3 27 27 2 a b a b a b a b Copyright 2016 Pearson. All rights reserved.

M02_BITT9391_11_SE_C02.5.indd 246 29/10/14 4:56 PM Not for Sale

2.5 ● Maximum–Minimum Problems; Business, Economics, and General Applications 247

In manufacturing, minimizing the amount of material used is always preferred, both from a cost standpoint and in terms of efficiency.

EXAMPLE 3 Minimizing Cost of Material. Mendoza Manufacturers special- izes in food-storage containers and makes a cylindrical soup can with a volume of 250 cm3. The cost of material for the two circular ends is $0.0008 cm2, and the cost of material for the side is $0.0015 cm2. What dimensions minimize the cost of mate- rial for the soup can? What is the minimum cost? > > Solution Recall from Section R.1 that we estimated the dimensions that give the lowest cost using a spreadsheet.

ABC 1 2 Radius Height Cost 3 r h C 4 4.1 4.733322705 0.267434061 5 4.2 4.510609676 0.267251237 Possible minimum cost 6 4.3 4.303253363 0.267371533

The spreadsheet shows that a radius of about 4.2 cm gives a minimal cost of $0.267251237 per can. r We can use calculus to find exact values for the radius and height that will minimize the cost per can. We let h = height of the can and r = radius, both measured in centimeters. The formula for h volume of a cylinder is V = pr2h. Since we know the volume is 250 cm3, this formula allows us to relate h and r, express- ing one in terms of the other. We solve for h in terms of r: pr2h = 250 250 h = . pr2 The can has two circular ends, each with an area of pr2. Thus, the cost of material for the two circular ends is 2 0.0008 pr2 = 0.0016pr2. The side of the can, when disassembled, is a rectangle of area 2prh, as shown at r left. The cost of material for the side is1 2 2 0.0015 prh = 0.003prh 250 l = 2p r 1 2 = 0.003pr Substituting pr2 h 0.75 a b = . Simplifying r

r Thus, the total cost of material for one can is given by 0.75 C r = 0.0016pr2 , where r 0. + r 7 Each of the circular ends of the disassem- 2 bled can has an area of pr , and the side Differentiating,1 2 we have has an area of 2prh. 0.75 C′ r = 0.0032pr - . r2 1 2

Copyright 2016 Pearson. All rights reserved.

M02_BITT9391_11_SE_C02.5.indd 247 29/10/14 4:56 PM Not for Sale

248 Chapter 2 ● Applications of Differentiation

We set the derivative equal to zero and solve to find the critical value(s): 0.75 0.0032pr - = 0 r2 0.75 0.0032pr = r2 0.0032pr3 = 0.75 Multiplying both sides by r2 0.75 r 3 = 0.0032p

3 0.75 r = 0.0032p

y r ≈ 4.21A Using a calculator C(r) 0.32 Note that the critical value r ≈ 4.21 is the only critical value in the interval 0, ∞ . To determine whether this critical value results in a minimum or maximum value of 0.24 (4.21, 0.2673) C, we use the Second-Derivative Test: 1 2 1.5 0.16 p C″ r = 0.0032 + 3

Cost (in dollars) r 0.08 1 2 1.5 p C″ 4.21 = 0.0032 + 3 7 0. 20 4 6 8 r 4.21 Radius (in centimeters) 1 2 The second derivative is positive, so 1the critical2 value indicates a minimum point. Thus, the cost of material for one can is minimized when r = 4.21 cm. The can will have a height of Quick Check 3 h = 250 p 4.21 2 ≈ 4.49 cm, and the cost of material will be $0.2673 per can. Repeat Example 3, assuming that We rounded our answer above. If we do not, we find that a radius of 4.209725752c > 1 2 the can has an open top. That is, cm will give a minimal cost of +0.26723831cper can. A manufacturer who produces it consists of one circular end and millions of cans may be interested in carrying the precision of the calculations to six the side. or seven decimal places. 3

EXAMPLE 4 Business: Maximizing Revenue. Cruzing Tunes determines that in order to sell x units of a new car stereo, the price per unit, in dollars, must be p x = 1000 - x. It also determines1 2 that the total cost of producing x units is given by C x = 3000 + 20x. a) Find the1 total2 revenue, R x . b) Find the total profit, P x . 1 2 c) How many units must be made and sold in order to maximize profit? 1 2 d) What is the maximum profit? e) What price per unit must be charged in order to make this maximum profit? Solution a) R x = Total revenue Number of units Price per unit 1 2 = # x # p = 1 2 1 2 = x # 1000 - x Substituting 1000x x2. = - 1 2 b) P x = Total revenue - Total cost R x C x 1 2 = - 1000x x2 3000 20x = 1 2 -1 2 - + x2 980x 3000 = 1- + 2- 1 2 Copyright 2016 Pearson. All rights reserved.

M02_BITT9391_11_SE_C02.5.indd 248 29/10/14 4:56 PM Not for Sale

2.5 ● Maximum–Minimum Problems; Business, Economics, and General Applications 249

c) To find the maximum value of P x , we first find P′ x :

P′ x = -2x + 980. 1 2 1 2 Note that1 2x Ú 0. The critical value(s) are found by solving P′ x = 0: P ′ x = -2x + 980 = 0 1 2 2x 980 1 2 - = - x = 490. There is only one critical value. We can therefore try to use the second derivative to determine whether we have an absolute maximum. Note that P ″ x = -2, a constant.

Quick Check 4 Since, P ″1 4902 is negative, profit is maximized when 490 units are produced and sold. d) The maximum profit is given by Repeat Example 4 with the price 1 2 2 function P 490 = - 490 + 980 # 490 - 3000 p x 1750 2x +237,100. = - 1 2 = 1 2 and the1 cost2 function Thus, Cruzing Tunes make a maximum profit of $237,100 by producing and selling 490 stereos. C x = 2250 + 15x. e) The price per unit needed to maximize profit is Round your1 2 answers when necessary. p = 1000 - 490 = +510. 4

Let’s take a general look at the profit, cost, and revenue. y C(x) Figure 1 shows an example of total-cost and total-revenue functions. We can Maximum Loss prot B estimate what the maximum profit is by looking for the widest gap between R x A 2 and C x , when R x 7 C x . Points B0 and B2 are break-even points. R(x) Figure 2 shows the related total-profit function. Note that when production1 2 B0 1 2 1 2 1 2 B1 is too low 6 x0 , there is a loss, perhaps due to high fixed or initial costs and low revenue. When production is too high 7 x2 , there is also a loss, perhaps due to the 1increased2 cost of overtime pay or expansion. 1 2 Loss The business operates at a profit everywhere between x0 and x2. Note that max- imum profit occurs at a critical value x1. Assuming that P′ x exists for all x in some interval, usually [0, ∞), the critical value x1 occurs at some number x such that x x x x 1 2 0 1 2 P′ x = 0 and P ″ x 6 0. Figure 1 Since P x 1=2R x - C x , it follows1 2 that 1P2′ x =1 R2 ′ x 1- 2C′ x and P ″ x = R″ x - C″ x . y Maximum prot Thus, maximum1 2 profit1 2 occurs1 at2 some number1 2 x such1 that2 1 2 M P(x) = R(x) − C(x) P′ x = R′ x - C′ x = 0 and P ″ x = R″ x - C″ x 6 0, or R′ x = C′ x and R″ x 6 C″ x . x0 x1 x2 x 1 2 1 2 1 2 1 2 1 2 1 2 In summary,1 2we have1 the2 following1 theorem.2 1 2 Figure 2 ThEorEM 10 Maximum profit occurs at thosex -values for which R′ x = C′ x and R″ x 6 C″ x .* 1 2 1 2 1 2 1 2

*In Section 2.6, the concepts of marginal revenue and marginal cost are introduced, allowing R′ x = C′ x to be regarded as Marginal revenue = Marginal cost. 1 2 1 2 Copyright 2016 Pearson. All rights reserved.

M02_BITT9391_11_SE_C02.5.indd 249 29/10/14 4:56 PM Not for Sale

250 Chapter 2 ● Applications of Differentiation

You can check that the results in parts (c) and (d) of Example 4 can be easily found using Theorem 10.

EXAMPLE 5 Business: Determining Ticket Price. By keeping records, a theater determines that at a ticket price of $26, it averages 1000 people in attendance. For every drop in price of $1, it gains 50 customers. Each customer spends an average of $4 on concessions. What ticket price should the theater charge in order to maximize total revenue? Solution Let x be the number of dollars by which the ticket price of $26 should be decreased. (If x is negative, the price is increased.) We first express the total revenue R as a function of x. Note that the increase in ticket sales is 50x when the price drops x dollars: R x = Revenue from tickets + Revenue from concessions Number of people Ticket price Number of people 4 1 2 = 1 # 2 1 + 2 # 1000 50x 26 x 1000 50x 4 = 1 + -2 1 + +2 1 # 2 50x2 500x 30,000. = 1- + 21 + 2 1 2 To findx such that R x is a maximum, we first find R′ x :

R′ x = -1001 2x + 500. 1 2 This derivative1 2 exists for all real numbers x. Thus, the only critical values are where R′ x = 0; so we solve that equation:

Quick Check 5 1 2 -100x + 500 = 0 A baseball team charges $30 per -100x = -500 ticket and averages 20,000 people x = 5 This corresponds to lowering the price by $5. in attendance per game. Each Since this is the only critical value, we can use the second derivative, person spends an average of $8 on concessions. For every drop of $1 R″ x = -100, in the ticket price, the attendance to determine whether we have a maximum. Since R″ 5 is negative, R 5 is a maxi- rises by 800 people. What ticket 1 2 mum. Therefore, in order to maximize revenue, the theater should charge price should the team charge to 1 2 1 2 maximize total revenue? $26 - $5, or $21 per ticket. 5

Minimizing Inventory Costs A retail business outlet needs to be concerned about inventory costs. Suppose, for ex- ample, that a home electronics store sells 2500 television sets per year. It could operate by ordering all the sets at once. But then the owners would face the carrying costs (insurance, building space, and so on) of storing them all. Thus, they might make several smaller orders, so that the largest number they would ever have to store is 500. However, each time they reorder, there are costs for paperwork, delivery charges, la- bor, and so on. It seems, therefore, that there must be some balance between carrying costs and reorder costs. Let’s see how calculus can help determine what that balance might be. We are trying to minimize the following function: Total inventory costs = Yearly carrying costs + Yearly reorder costs . The lot size x is the largest number1 ordered each reordering2 1 period. If x units2 are ordered each period, then during that time somewhere between 0 and x units are in stock. To have a representative expression for the amount in stock at any one time in the period, we can use the average, x 2. This represents the average amount held in stock over the course of each time period. Refer to the graphs shown on the> next page. If the lot size is 2500, then during the period between orders, there are somewhere between 0 and 2500 units in stock.

Copyright 2016 Pearson. All rights reserved.

M02_BITT9391_11_SE_C02.5.indd 250 29/10/14 4:56 PM Not for Sale

2.5 ● Maximum–Minimum Problems; Business, Economics, and General Applications 251

On average, there are 2500 2, or 1250 units in stock. If the lot size is 1250, then during the period between orders, there are somewhere between 0 and 1250 units in stock. On average, there are 1250> 2, or 625 units in stock. In general, if the lot size is x, the average inventory is x 2. > 2500 >

Average lot size (average inventory) Lot size 1250 (inventory)

Time (in months) 0 12

1250 Average lot size (average inventory) Lot size 625 (inventory) Time (in months) 0 6 12

Average lot size (average inventory) Lot size 625 312.5 ≈ 313 (inventory) Time (in months) 0 396 12

Average lot size x (average inventory) Lot size ±x ± (inventory) 2

0 Time (in months)

EXAMPLE 6 Business: Minimizing Inventory Costs. Fred’s Electronics sells 2500 television sets per year. It costs $20 to store one set for a year. To reorder, there is a fixed cost of $20, plus a fee of $9 per set. How many times per year should the store reorder, and in what lot size, to minimize inventory costs? Solution Let x = the lot size. Inventory costs are given by C x = Yearly carrying costs + Yearly reorder costs . We consider1 2each1 component of inventory2 costs1 separately. 2 a) Yearly carrying costs. The average amount held in stock is x 2, and it costs $20 per set for storage. Thus, > Yearly cost Average number Yearly carrying costs = per item # of items a x b a b = 20 # . 2 b) Yearly reorder costs. We know that x is the lot size, and we let N be the number of reorders each year. Then Nx = 2500, and N = 2500 x. Thus, Cost of each Number of Yearly reorder costs > = order # reorders a 2500b a b = 20 9x . + x Copyright 2016 Pearson. All rights reserved.1 2

M02_BITT9391_11_SE_C02.5.indd 251 29/10/14 4:56 PM Not for Sale

252 Chapter 2 ● Applications of Differentiation

c) Thus, we have x 2500 C x = 20 # + 20 + 9x 2 x 1 2 50,0001 2 = 10x 22,500 = 10x 50,000x-1 22,500. + x + + + d) To find a minimum value ofC over 1, 2500 , we first find C′ x :

50,000 3 4 1 2 C′ x = 10 - . x2 e) Note that1 2C′ x exists for all x in 1, 2500 , so the only critical values are those x-values such that C′ x = 0. We solve C′ x = 0: 1 2 3 4 50,000 1 2 1 2 10 - = 0 x2 50,000 10 = x2 y 10x2 50,000 50,000 = C(x) = 10x + ± ± Ð Ð + 22,500 ± x x 2 = 5000 x = 5000 ≈ 70.7. Since there is only one 2critical value in 1, 2500 , that is, x ≈ 70.7, we can use the second derivative to see whether that value yields a maximum or a minimum: 3 4 100,000 C″ x = . x3 1 2 70.7 x C″ x is positive for all x in 1, 2500 , so we have a minimum at x ≈ 70.7. However, it is impossible to reorder 70.7 sets each time, so we consider the two whole1 2 numbers closest to 70.7,3 which4 are 70 and 71. Since C 70 ≈ $23,914.29 and C 71 ≈ $23,914.23, Quick Check 6 it follows1 that2 the lot size that will minimize1 2 cost is 71, although the difference, Repeat Example 6 with a storage $0.06, is not much. (Note: Such a procedure will not work for all functions but will cost of $30 per set and assuming work for the type we are considering here.) The number of times an order should be that the store sells 3000 sets per placed is 2500 71 with a remainder of 15, indicating that 35 orders should be placed. year. Of those, 35 - 15 = 20 will be for 71 items and 15 will be for 72 items. 6 >

The lot size that minimizes total inventory costs is often referred to as the eco- nomic ordering quantity. Three assumptions are made in using the preceding method to determine the economic ordering quantity. First, the demand for the product is the same year round. For television sets, this may be reasonable, but for seasonal items such as clothing or skis, this assumption is unrealistic. Second, the time between placing an order and receiving the product remains consistent year-round. Finally, the costs involved, such as storage, shipping charges, and so on, do not vary. This assumption may not be reasonable in a time of inflation, although variation in these costs can be allowed for by anticipating what they might be and using average costs. Regardless, the model described above is useful, allowing us to analyze a seemingly difficult problem using calculus.

Copyright 2016 Pearson. All rights reserved.

M02_BITT9391_11_SE_C02.5.indd 252 29/10/14 4:56 PM Not for Sale

2.5 ● Maximum–Minimum Problems; Business, Economics, and General Applications 253

TEChnology ConnECTIon Using Excel Spreadsheets to Numerically Estimate Minimum and Maximum Values An Excel spreadsheet can numerically estimate absolute extrema. For example, to estimate the values for Example 2, we open a blank spreadsheet and enter text in the cells as shown below:

ABC D 1 Starting value 2 Step size: 3 4 Height Length Width Volume 5 H=x L=8–2x W=8–2x V=H*L*W

In cell A6, we enter =b1; in cells B6 and C6, we enter =8-2*a6; and in cell D6, we enter =a6*b6*c6. In cell A7, we enter =a6+$b$2. We copy cells B6, C6, and D6 and paste them into cells B7, C7, and D7, then copy cells A7 through D7 and paste them into cells A8 through D14. This creates nine active rows. More rows can be added if necessary. In cell B1, we enter a starting x-value, and in cell B2, a step size. For example, if we enter 0 into cell B1 and 0.25 into cell B2, the values in cells A6–A14 will be x = 0, 0.25, 0.5, 0.75, 1, and so on. The result is the following table:

ABC 1 Starting value 0 2 Step size: 0.25 3 4 Height Length Width Volume 5 H=x L=8–2x W=8–2x V=H*L*W 6 0 8 8 0 7 0.25 7.5 7.5 14.0625 8 0.5 7 7 24.5 9 0.75 6.5 6.5 31.6875 10 1 6 6 36 Possible 11 1.25 5.5 5.5 37.8125 largest 12 1.5 5 5 37.5 volume 13 1.75 4.5 4.5 35.4375 14 2 4 4 32

We see that a possible maximum volume of 37.8125 in2 occurs when x = 1.25 in. To refine our estimate, we enter 1 into cell B1 and 0.1 into cell B2, giving the following table:

ABC 1 Starting value 1 2 Step size: 0.1 3 4 Height Length Width Volume 5 H=x L=8–2x W=8–2x V=H*L*W 6 1 6 6 36 7 1.1 5.8 5.8 37.004 8 1.2 5.6 5.6 37.632 Possible 9 1.3 5.4 5.4 37.908 largest 10 1.4 5.2 5.2 37.856 volume 11 1.5 5 5 37.5 12 1.6 4.8 4.8 36.864

By reducing the step size, we obtain a new possible maximum volume of 37.908 in3 when x = 1.3 in. Although we could fur- ther reduce the step size, we now have an estimate of the maximum volume and the x-value at which it occurs. We use calculus to determine these values exactly.

ExERCIsEs 1. Use a spreadsheet to verify the results of Example 1. 3. Use a spreadsheet to verify the results of Example 5. 2. Use a spreadsheet to verify the results of Example 4. 4. Use a spreadsheet to verify the results of Example 6.

Copyright 2016 Pearson. All rights reserved.

M02_BITT9391_11_SE_C02.5.indd 253 29/10/14 4:56 PM Not for Sale

254 Chapter 2 ● Applications of Differentiation

Section Summary ● In many real-life applications, we need to determine ● Maximum profit occurs at thosex -values for which the minimum or maximum value of a function model- R′ x = C′ x and R″ x 6 C″ x . ing a situation. 1 2 1 2 1 2 1 2 2.5 exercise Set

1. Of all numbers whose sum is 70, find the two that have 12. Maximizing area. A lifeguard needs to rope off a rect- the maximum product. That is, maximize Q = xy, where angular swimming area in front of Long Lake Beach, x + y = 70. using 180 yd of rope and floats. What dimensions of the rectangle will maximize the area? What is the 2. Of all numbers whose sum is 50, find the two that have maximum area? (Note that the shoreline is one side of the maximum product. That is, maximize Q = xy, where the rectangle.) x + y = 50. 3. Of all numbers whose difference is 6, find the two that have the minimum product. 4. Of all numbers whose difference is 4, find the two that x have the minimum product.

5. Maximize Q = xy2, where x and y are positive num- x 2 bers such that x + y = 4. 6. Maximize Q = xy2, where x and y are positive num- 2 bers such that x + y = 1. 2 2 7. Minimize Q = x + 2y , where x + y = 3. 13. Maximizing area. Hentz Industries plans to enclose 2 2 three parallel rectangular areas for sorting returned 8. Minimize Q = 2x + 3y , where x + y = 5. goods. The three areas are within one large rectangular 9. Maximize Q = xy, where x and y are positive numbers area and 1200 yd of fencing is available. What is the 4 2 such that x + 3y = 1. largest total area that can be enclosed? 10. Maximize Q = xy, where x and y are positive numbers 4 2 such that 3x + y = 16. 11. Maximizing area. A rancher wants to enclose two rectangular areas near a river, one for sheep and one for cattle. There are 240 yd of fencing available. What is the largest total area that can be enclosed?

14. Maximizing area. Grayson Farms plans to enclose three parallel rectangular livestock pens within one large rectangular area using 600 ft of fencing. One side x of the enclosure is a pre-existing stone wall. x a) If the three rectangular pens have their longer sides parallel to the stone wall, find the largest possible x total area that can be enclosed.

Copyright 2016 Pearson. All rights reserved.

M02_BITT9391_11_SE_C02.5.indd 254 29/10/14 4:56 PM Not for Sale Exercise Set 2.5 255

b) If the three rectangular pens have their shorter sides APPLicATionS perpendicular to the stone wall, find the largest possible total area that can be enclosed. Business and economics Maximizing profit. For Exercises 23–28, find the maximum profit and the number of units that must be produced and sold in order to yield the maximum profit. Assume that revenue, R(x), and cost, C(x), are in dollars for Exercises 23–26. 2 23. R x = 50x - 0.5x , C x = 4x + 10 2 24. R1x2 = 50x - 0.5x , C1x2 = 10x + 3 15. Maximizing area. Of all rectangles that have a perim- 2 eter of 42 ft, find the dimensions of the one with the 25. R1x2 = 2x, C x = 0.011x2 + 0.6x + 30 largest area. What is its area? 2 26. R1x2 = 5x, C1x2 = 0.001x + 1.2x + 60 16. Maximizing area. A carpenter is building a rectangular 27. R x 9x 2x2, C x x3 3x2 4x 1; shed with a fixed perimeter of 54 ft. What are the di- 1 2 = - 1 2 = - + + assume that R x and C x are in thousands of dollars, mensions of the largest shed that can be built? What is and1 2 x is in thousands of1 units.2 its area? 1 2 1 2 28. R x 100x x2, C x 1x3 6x2 89x 100; 17. Maximizing volume. From a thin piece of cardboard = - = 3 - + + assume that R x and C x are in thousands of dollars, 20 in. by 20 in., square corners are cut out so that the and1 2 x is in thousands 1of2 units. sides can be folded up to make a box. What dimen- 1 2 1 2 sions will yield a box of maximum volume? What is the 29. Maximizing profit. Riverside Appliances is marketing maximum volume? a new refrigerator. It determines that in order to sell x refrigerators, the price per refrigerator must be 18. Maximizing volume. From a 50-cm-by-50-cm sheet of aluminum, square corners are cut out so that the sides p = 280 - 0.4x. can be folded up to make a box. What dimensions will It also determines that the total cost of producing x yield a box of maximum volume? What is the maxi- refrigerators is given by mum volume? 2 C x = 5000 + 0.6x . 19. Minimizing surface area. Mendoza Soup Company is a) Find the total revenue, R x . constructing an open-top, square-based, rectangular 1 2 3 b) Find the total profit, P x . metal tank that will have a volume of 32 ft . What c) How many refrigerators must1 2 the company produce dimensions will minimize surface area? What is the and sell in order to maximize1 2 profit? minimum surface area? d) What is the maximum profit? 20. Minimizing surface area. Drum Tight Containers is e) What price per refrigerator must be charged in order designing an open-top, square-based, rectangular box to maximize profit? 3 that will have a volume of 62.5 in . What dimensions 30. Maximizing profit. Raggs, Ltd., a clothing firm, will minimize surface area? What is the minimum sur- determines that in order to sell x suits, the price per face area? suit must be 21. Minimizing surface area. Open Air Waste Management p = 150 - 0.5x. is designing a rectangular construction dumpster that 3 It also determines that the total cost of producing x will be twice as long as it is wide and must hold 12 yd suits is given by of debris. Find the dimensions of the dumpster that will 2 minimize its surface area. C x = 4000 + 0.25x . a) Find the total revenue, R x . 1 2 b) Find the total profit, P x . c) How many suits must the1 company2 produce and sell in order to maximize profit?1 2 ? d) What is the maximum profit? e) What price per suit must be charged in order to ? x maximize profit? 31. Maximizing profit. Gritz-Charlston is a 300-unit 22. Minimizing surface area. Ever Green Gardening is luxury hotel. All rooms are occupied when the hotel designing a rectangular compost container that will be charges $80 per day for a room. For every increase of x twice as tall as it is wide and must hold 18 ft3 of com- dollars in the daily room rate, there are x rooms vacant. posted food scraps. Find the dimensions of the com- Each occupied room costs $22 per day to service and post container with minimal surface area (include the maintain. What should the hotel charge per day in or- bottom and top). der to maximize profit?

Copyright 2016 Pearson. All rights reserved.

M02_BITT9391_11_SE_C02.5.indd 255 29/10/14 4:56 PM Not for Sale

256 Chapter 2 ● Applications of Differentiation

32. Maximizing revenue. Edwards University wants to attendance of 180 people. For every $0.10 increase in determine what price to charge for tickets to football admission, there is a loss of 1 customer from the aver- games. At a price of $18 per ticket, attendance averages age number. What admission should be charged in or- 40,000 people per game. Every decrease of $3 to der to maximize revenue? the ticket price adds 10,000 people to the average 38. Minimizing costs. A rectangular box with a volume of attendance. Every person at a game spends an average 3 of $4.50 on concessions. What price per ticket should 320 ft is to be constructed with a square base and top. be charged to maximize revenue? How many people The cost per square foot for the bottom is 15., for the will attend at that price? top is 10., and for the sides is 2.5.. What dimensions will minimize the cost? 33. Maximizing parking tickets. Oak Glen currently x employs 8 patrol officers who each write an average of 24 parking tickets per day. For every additional officer x placed on patrol, the average number of parking tickets per day written by each officer decreases by 4. How many additional officers should be placed on patrol in order to maximize the number of parking tickets writ- y ten per day? 34. Maximizing yield. Hood Apple Farm yields an average x of 30 bushels of apples per tree when 20 trees are planted x on an acre of ground. If 1 more tree is planted per acre, the yield decreases by 1 bushel (bu) per tree as a result of 39. Minimizing cost. A rectangular parking area mea- crowding. How many trees should be planted on an acre suring 5000 ft2 is to be enclosed on three sides using in order to get the highest yield? chain-link fencing that costs $4.50 per foot. The fourth side will be a wooden fence that costs $7 per foot. What 35. nitrogen prices. During 2001, nitrogen prices fell by dimensions will minimize the total cost to enclose this 41%. Over the same year, nitrogen demand went up by area, and what is the minimum cost (rounded to the 12%. (Source: Chemical Week.) nearest dollar)? a) Assuming a linear change in demand, find the 40. Minimizing cost. A rectangular garden measuring demand function, q x , by finding the equation 2 of the line that passes through the points 1200 yd is to be enclosed on two parallel sides by 1, 1 and 0.59, 1.121 2. Here x is the price as stone wall that costs $35 per yd and the other two sides a fraction of the January 2001 price, and q x by wooden fencing that costs $28 per yd. What dimen- is1 the2 demand1 as a fraction2 of the demand in sions will minimize the total cost of enclosing this January. 1 2 garden, and what is the minimum cost (rounded to the b) As a percentage of the January 2001 price, what nearest dollar)? should the price of nitrogen be to maximize 41. Maximizing area. Bradley Publishing decides that each revenue? page in a new book must have an area of 73.125 in2, 36. Vanity license plates. According to a pricing model, a 0.75-in. margin at the top and at the bottom of each increasing the fee for vanity license plates by $1 de- page, and a 0.5-in. margin on each of the sides. What creases the percentage of a state’s population that should the outside dimensions of each page be so that will request them by 0.04%. (Source: E. D. Craft, “The the printed area is a maximum? demand for vanity (plates): Elasticities, net revenue 42. Minimizing inventory costs. A sporting goods store maximization, and deadweight loss,” Contemporary sells 100 pool tables per year. It costs $20 to store one Economic Policy, Vol. 20, 133–144 (2002).) pool table for a year. To reorder, there is a fixed cost a) Recently, the fee for vanity license plates in of $40 per shipment plus $16 for each pool table. How Maryland was $25, and the percentage of the state’s many times per year should the store order pool tables, population that had vanity plates was 2.13%. Use and in what lot size, in order to minimize inventory this information to construct the demand function, costs? q x , for the percentage of Maryland’s population that will request vanity license plates for a fee of x 43. Minimizing inventory costs. A pro shop in a bowling dollars.1 2 center sells 200 bowling balls per year. It costs $4 to b) Find the fee, x, that will maximize revenue from store one bowling ball for a year. To reorder, there is a vanity plates. fixed cost of $1, plus $0.50 for each bowling ball. How many times per year should the shop order bowling 37. Maximizing revenue. When the Marchant The- balls, and in what lot size, in order to minimize inven- ater charges $5 for admission, there is an average tory costs?

Copyright 2016 Pearson. All rights reserved.

M02_BITT9391_11_SE_C02.5.indd 256 29/10/14 4:56 PM Not for Sale Exercise Set 2.5 257

44. Minimizing inventory costs. A retail outlet for Boxo- 53. Minimizing cost. A rectangular play area of 48 yd2 witz Calculators sells 720 calculators per year. It costs is to be fenced off in a person’s yard. The next-door $2 to store one calculator for a year. To reorder, there is neighbor agrees to pay half the cost of the fence on the a fixed cost of $5, plus $2.50 for each calculator. How side of the play area that lies along the property line. many times per year should the store order calculators, What dimensions will minimize the cost of the fence? and in what lot size, in order to minimize inventory costs? Half the cost 45. Minimizing inventory costs. Bon Temps Surf and Scuba Shop sells 360 surfboards per year. It costs $8 to x x store one surfboard for a year. Each reorder costs $10, plus an additional $5 for each surfboard ordered. How y many times per year should the store order surfboards, and in what lot size, in order to minimize inventory 54. Maximizing light. A Norman window is a rectangle costs? with a semicircle on top. Suppose that the perim- 46. Minimizing inventory costs. Repeat Exercise 44 eter of a particular Norman window is to be 24 ft. using the same data, but assume yearly sales of What should its dimensions be in order to allow 256 calculators with the fixed cost of each reorder the maximum amount of light to enter through the set at $4. window? 47. Minimizing inventory costs. Repeat Exercise 45 using the same data, but change the reorder costs from an additional $5 per surfboard to $6 per surf- x board.

48. Minimizing surface area. A closed-top cylindrical y container is to have a volume of 250 in2. What dimen- sions (radius and height) will minimize the surface area? 49. Minimizing surface area. An open-top cylindrical container is to have a volume of 400 cm2. What dimensions (radius and height) will minimize the surface area? 50. Minimizing cost. Assume that the costs of the materi- als for making the cylindrical container described in Exercise 48 are +0.005 in2 for the circular base and top and +0.003 in2 for the wall. What dimensions will minimize the cost of materials?> 55. Maximizing light. Repeat Exercise 54, but assume > that the semicircle is to be stained glass, which trans- 51. Minimizing cost. Assume that the costs of the mate- mits only half as much light as clear glass does. rials for making the cylindrical container described in Exercise 49 are +0.0015 cm2 for the base and 56–110. Use a spreadsheet to numerically verify the result +0.008 cm2 for the wall. What dimensions will mini- of Exercises 1–55. mize the cost of materials? > > SynThESiS general interest 111. For what positive number is the sum of its reciprocal 52. Maximizing volume. The postal service places a limit and five times its square a minimum? of 84 in. on the combined length and girth of (distance around) a package to be sent parcel post. What dimen- 112. For what positive number is the sum of its reciprocal sions of a rectangular box with square cross-section will and four times its square a minimum? contain the largest volume that can be mailed? (Hint: There are two different girths.) 113. Business: maximizing profit. The amount of money that customers deposit in a bank in savings accounts is directly proportional to the interest rate that the bank pays on that money. Suppose a bank is able to loan out all the money deposited in its savings accounts at an interest rate of 18%. What interest rate should it pay on its savings accounts in order to maximize profit? x

y x Copyright 2016 Pearson. All rights reserved.

M02_BITT9391_11_SE_C02.5.indd 257 29/10/14 4:56 PM Not for Sale

258 Chapter 2 ● Applications of Differentiation

114. A 24-in. piece of wire is cut in two pieces. One piece Island is used to form a circle and the other to form a square. How should the wire be cut so that the sum of the ar- C eas is a minimum? A maximum?

24

x 24 ± x 3

B S A Home x 8 ±x loft 8 115. Business: minimizing costs. A power line is to be con- structed from a power station at point A to an island at 117. A rectangular field is to be divided into two paral- point C, which is 1 mi directly out in the water from a lel rectangular areas, as shown in the figure below. point B on the shore. Point B is 4 mi downshore from If the total fencing available is k units, show that the the power station at A. It costs $5000 per mile to lay the length of each of the three parallel fences will be k 6 power line under water and $3000 per mile to lay the line units. under ground. At what point S downshore from A should > the line come to the shore in order to minimize cost? Note that S could very well be B or A. (Hint: The length of CS is 1 + x2.) 2 Island 118. Business: minimizing distance. A road is to be built between two cities C1 and C2, which are on opposite C sides of a river of uniform width r. C1 is a units from the river, and C2 is b units from the river, with a … b. A bridge will carry the traffic across the river. Where should the bridge be located in order to minimize the total distance between the cities? Give a general solution 1 using the constants a, b, p, and r as shown in the figure.

C1

a B S A x 4 ±x p ±x 4 Bridge r River 116. life science: flights of homing pigeons. It is known x that homing pigeons tend to avoid flying over water in the daytime, perhaps because downdrafts of air over water make flying difficult. Suppose a homing pigeon b is released on an island at point C, which is 3 mi di- rectly out in the water from a point B on shore. Point B C2 is 8 mi downshore from the pigeon’s home loft at point p A. Assume that a pigeon flying over water uses energy at a rate 1.28 times the rate over land. Toward what 119. Business: minimizing cost. The total cost, in dollars, point S downshore from A should the pigeon fly in or- of producing x units of a certain product is given by der to minimize the total energy required to get to the x3 C x = 8x + 20 + . home loft at A? Assume that 100 Total energy = a) Find1 2 the average cost, A x = C x x. Energy rate over water # Distance over water b) Find C′ x and A′ x . + Energy rate over land # Distance over land . 1 2 1 2> c) Find the minimum of A x and the value x0 at 1 2 1 2 1 2 1 2 which it occurs. Find C′ x0 . 1 2 1 2 1 2 d) Compare A x0 and C′ x0 . 1 2 1 2 1 2

Copyright 2016 Pearson. All rights reserved.

M02_BITT9391_11_SE_C02.5.indd 258 29/10/14 4:56 PM Not for Sale

2.6 ● Marginals and Differentials 259

120. Business: minimizing cost. Consider TEChnology ConnECTIon A x = C x x. a) Find A x in terms of C x and C x . In Exercises 125–128, use a spreadsheet to maximize Q. 1 2 ′ 1 2> ′ b) Show that if A x has a minimum, then it will occur Assume that all variables are positive, and state all answers to 1 2 1 2 1 2 two decimal places. at that value of x0 for which 1 2 C ′ x0 = A x0 125. Q = xy, where 2x + 3y = 100 C x0 2 2 2 1 2 = 1 2 . 126. Q = x y, where x + y = 16 x0 1 2 127. Q xy3, where x2 2y2 20 This result shows that if average cost can be mini- = + = 2 2 2 2 mized, such a minimum will occur when marginal cost 128. Q = x y , where x + y = 50 equals average cost. 3 3 121. Minimize Q = x + 2y , where x and y are positive numbers, such that x + y = 1. Answers to Quick Checks 3 2 2 122. Minimize Q = 3x + y , where x + y = 2. 1. With 50 ft of fencing, the dimensions are 25 ft by 25 ft 2 123. Business: minimizing inventory costs—a general solu- 625 ft area ; with 100 ft of fencing, they are 50 ft by 50 ft 2 tion. A store sells Q units of a product per year. It 2500 ft area ; in general, n feet of fencing gives n 2 ft 1 22 2 costs a dollars to store one unit for a year. To reorder, by n 2 ft n 4 ft area . 2. The dimensions are 1 2 > there is a fixed cost of b dollars, plus c dollars for each approximately 1.585 in. by 5.33 in. by 7.83 in.; the volume > 13 > 2 3 unit. How many times per year should the store reor- is 66.15 in , or 1083.5 cm , slightly more than 1 L. der, and in what lot size, in order to minimize inven- 3. r ≈ 5.3 cm, h ≈ 2.83 cm, c ≈ $0.212 can 2 2 tory costs? 4. (a) R x = 1750x - 2x ; (b) P x = -2x + 1735x - 2250; (c) x = 434 units; (d) >maximum 124. Business: minimizing inventory costs. Use the gen- profit = 1+374,0282 ; (e) price per unit1 2= +882.00 eral solution found in Exercise 123 to find how many 5. $23.50 6. x ≈ 63; the store should place 8 orders times per year a store should reorder, and in what for 63 sets and 39 orders for 64 sets. lot size, when Q = 2500, a = $10, b = $20, and c = $9.

Marginals and Differentials In this section, we consider ways of using calculus to make linear approximations. 2.6 Suppose, for example, that a company is considering an increase in production. Usually the company wants at least an approximation of what the resulting changes in cost, revenue, and profit will be. ● Find marginal cost, revenue, and profit. ● Find ∆y and dy. Marginal Cost, Revenue, and Profit ● Use differentials for Suppose an animal rescue organization that produces its own public service an- approximations. nouncements (PSAs) is considering an increase in monthly production from 12 PSAs to 13. To estimate the resulting increase in cost, it would be reasonable to find the rate at which cost is increasing when 12 PSAs are produced and add that to the cost of producing 12 PSAs. That is, C 13 ≈ C 12 + C′ 12 . The number1 C2 ′ 12 1 is called2 the1 2marginal cost at 12. Remember that C′ 12 is the 3 slope of the tangent line at the point 12, C 12 . If, for example, this slope is 4, we can regard it as1 a vertical2 change of 3 with a horizontal change of 4, or1 a vertical2 3 1 1 22 change of 4 with a horizontal change of 1. It is this latter interpretation that we use for estimating. Graphically, this interpretation can be viewed as shown in the follow- ing graph. Note in the figure that C′ 12 is slightly more than the difference between C 13 and C 12 , or C 13 - C 12 . For other curves, C′ 12 may be slightly less than C 13 - C 12 . Almost always,1 however,2 it is simpler to compute C′ 12 than it is1 to compute2 1 C 213 - 1C 122 . 1 2 1 2 1 2 1 2 1 2 Copyright 2016 Pearson.1 2 All 1rights2 reserved.

M02_BITT9391_11_SE_C02.5.indd 259 29/10/14 4:56 PM Not for Sale

260 Chapter 2 ● Applications of Differentiation

C(x)

C(12) + C′(12)

C(13) C′(12)

C(12)

Cost of production 1

12 13 x Number of PSAs produced

Generalizing, we have the following.

Definitions Let C x , R x , and P x represent, respectively, the total cost, revenue, and profit from the production and sale ofx items. 1 2 1 2 1 2 The marginal cost at x, given by C′ x , is the approximate cost of the x + 1 st item: 1 2 1 2C′ x ≈ C x + 1 - C x , or C x + 1 ≈ C x + C′ x . The marginal1 2 revenue1 at x2, given1 by2 R′ x1 , is the2 approximate1 2 revenue1 2 from the x + 1 st item: 1 2 1 2R′ x ≈ R x + 1 - R x , or R x + 1 ≈ R x + R′ x . The marginal1 2 profit1 at x, given2 by1 2 P′ x ,1 is the 2approximate1 2 profit1 2 from the x + 1 st item: 1 2 1 2P′ x ≈ P x + 1 - P x , or P x + 1 ≈ P x + P′ x . 1 2 1 2 1 2 1 2 1 2 1 2 The student can confirm that P′ x = R′ x - C′ x .

eXAMPLe 1 Business: Marginal1 2 Cost,1 Revenue,2 1 2 and Profit. Given 2 C x = 62x + 27,500 and R x x3 12x2 40x 10, 1 2 = - + + find each 1of 2the following. a) Total profit, P x b) Total cost, revenue, and profit from the production and sale of 50 units of the 1 2 product c) The marginal cost, revenue, and profit when 50 units are produced and sold Solution a) Total profit = P x = R x - C x x3 12x2 40x 10 62x2 27,500 1 2 = 1 -2 1 +2 + - + x3 74x2 40x 27,490 = - + - 1 2

Copyright 2016 Pearson. All rights reserved.

M02_BITT9391_11_SE_C02.6.indd 260 29/10/14 4:59 PM Not for Sale

2.6 ● Marginals and Differentials 261

b) We have 2 C 50 = 62 # 50 + 27,500 = +182,500 The total cost of producing the first 50 units 1 2 3 2 R 50 = 50 - 12 # 50 + 40 # 50 + 10 = +97,010 The total revenue from the sale of the first 1 2 50 units P 50 = R 50 - C 50 = +97,010 - +182,500 We could also use P x from part (a). Quick Check 1 1 2 1 2 1 2 = - +85,490 There is a loss of $85,4901 2 when 50 units are Given produced and sold. c) We have C x = 10x + 3 Once 50 units have been made, C ′ x 124x, and R x 50x 0.5x2, = the approximate cost of the 51st 1 2 = - C ′ 50 = 124 # 50 = +6200 unit (marginal cost) is $6200. find each1 2of the following 1 2 2 Once 50 units have been sold, the a) P x R 1′ x2 = 3x - 24x + 40, approximate revenue from the 51st b) Total cost, revenue, and profit R ′ 50 3 502 24 50 40 +6340 1 2 1 2 = # - # + = unit (marginal revenue) is $6340. from the production and sale 2 Once 50 units have been pro- of 40 units P 1′ x2 = 3x - 148x + 40, duced and sold, the approximate P ′ 50 3 # 502 148 # 50 40 +140 c) The marginal cost, revenue, 1 2 = - + = profit from the sale of the 51st and profit when 40 units are 1 2 item (marginal profit) is $140 produced and sold 1

TEChNology CoNNECTioN Often, in business, formulas for C x , R x , and P x are not known, but informa- tion may exist about the cost, revenue, and profit trends at a particular value x = a. To check the accuracy of R′ 50 as For example, C a and C′ a may be 1known,2 1 2 allowing1 2a reasonable prediction to be an estimate of R 51 - R 50 , let made about C a + 1 . In a similar manner, predictions can be made for R a + 1 and 3 2 1 2 1 2 1 2 y1 = x - 12x + 40x + 10, P a + 1 . In Example 1, formulas do exist, so it is possible to see how accurate our 1 2 1 2 y2 = y1 x + 1 - y1 x , and predictions were.1 We2 check C 51 - C 50 and leave the checks of R 511 - R2 50 y3 = nDeriv y1, x, x . By using and1 P 512 - P 50 to the student (see the Technology Connection in the margin): table with1 indpnt2: ask, we1 can2 1 2 1 2 1 2 1 2 1 2 2 2 display a table in which y2 (the 1 C 2 51 1- C2 50 = 62 # 51 + 27,500 - 62 # 50 + 27,500 difference between y1 x + 1 6262, 1 2 1 2 = 1 2 and y1 x ) can be compared with 1 2 whereas C ′ 50 = 6200. y1′ x . 1 2 In this case, C′ 50 provides an approximation of C 51 - C 50 that is within 1% 1 2 X Y2 Y3 1 2 40 3989 3880 of the actual value. 48 5933 5800 1 2 1 2 1 2 50 6479 6340 Note that marginal cost is different from average cost: C 50 Total cost of 50 units Average cost per unit for 50 units = 50 The number of units, 50 X = 1 2 182,500 = = +3650, ExERCisE 50 1. Create a table to check the whereas accuracy of P′ 50 as an esti- Marginal cost when 50 units are produced = +6200 mate of P 51 - P 50 . 1 2 ≈ cost of the 51st unit. 1 2 1 2 Differentials and Delta Notation

Just as marginal cost C′ x0 is used to estimate C x0 + 1 , the value of f′ x0 , can be used to estimate values of f x for x-values near x0. To do so, we need to develop some notation. Recall that the difference1 2 quotient, 1 2 1 2 1 2 f x + h - f x , h 1 2 1 2 Copyright 2016 Pearson. All rights reserved.

M02_BITT9391_11_SE_C02.6.indd 261 29/10/14 4:59 PM Not for Sale

262 Chapter 2 ● Applications of Differentiation

y represents the slope of a secant line, as shown in the graph. The number h is regarded as the change in x. Another notation for such a change is ∆x (read “delta x”), called delta notation. The expression ∆x is not the product of ∆ and x, but f(x) f(x + h) – f(x) rather a new type of variable that represents the change in the value of x from a m = ±± ÐÐ ±ÐÐÐ h first value to a second. Thus, ∆x = x + h - x = h. Δy = f(x + h) ±f (x) If subscripts1 are2 used for the first and second values of x, we have Δx = h ∆x = x2 - x1, or x2 = x1 + ∆x. x = x 1 x x2 = x + h = x + Δx Note that the value of ∆x can be positive or negative. For example,

if x1 = 4 and ∆x = 0.7, then x2 = 4.7, and if x1 = 4 and ∆x = -0.7, then x2 = 3.3. We generally omit the subscripts and use x and x + ∆x. Now suppose we have a function given by y = f x . A change in x from x to x + ∆x yields a change in y from f x to f x + ∆x . The change in y is given by 1 2 1 2 1∆y = f x2 + ∆x - f x .

eXAMPLe 2 1Find ∆y2 in each1 2 case. a) y = x2, x = 4, and ∆x = 0.1 3 b) y = x , x = 2, and ∆x = -0.1 Solution a) We have 2 2 ∆y = 4 + 0.1 - 4 4.1 2 42 16.81 16 0.81. = 1 -2 = - = b) We have Quick Check 2 1 2 4 3 3 For y = 2x + x, x = 2, and ∆y = 2 + -0.1 - 2 ∆x 0.05, find ∆y. 1.9 3 23 6.859 8 1.141. 2 = - = 3 1 - 24= - = - Let’s now use1 calculus2 to predict function values. If delta notation is used, the dif- ference quotient f x + h - f x h 1 2 1 2 becomes f x + ∆x - f x ∆y = . ∆x ∆x 1 2 1 2 We can then express the derivative as y dy ∆y f = lim . dx ∆xS0 ∆x ' Actual value m = f (x) For values of ∆x close to 0, we have the approximation Value from the tangent line dy ∆y ∆y ≈ , or f′ x ≈ . dx ∆x ∆x Multiplying both sides by ∆1x, 2we have ∆y ≈ f′ x ∆x. x x We can see this in1 the2 graph at the left.

Copyright 2016 Pearson. All rights reserved.

M02_BITT9391_11_SE_C02.6.indd 262 29/10/14 4:59 PM Not for Sale

2.6 ● Marginals and Differentials 263

From this graph, it seems reasonable to assume that, for small values of ∆x, the y-values on the tangent line can be used to estimate function values on the curve.

For f, a continuous, , and small ∆x, ∆y f′ x ≈ and ∆y ≈ f′ x # ∆x. ∆x 1 2 1 2 Up to now, we have treated dy dx as one symbol. We now define dy and dx as separate entities. These symbols are called differentials. >

Definition For y = f x , we define dx,1 called2 the differential of x, by dx = ∆x and dy, called the differential of y, by dy = f′ x dx. 1 2 We can illustrate dx and dy as y shown at the right. Note that dx = ∆x, f but, in general, dy  ∆y, though dy ≈ ∆y, for small values of dx. m = f'(x)

dy

dx

x x

eXAMPLe 3 Consider the function given by y = f x = x. 2 a) Find dy when x = 9 and dx = 1. 1 2 b) Compare dy to ∆y. c) Use the results of parts (a) and (b) to estimate 10. Solution 2 a) We find dy dx, then dy:

dy > 1 = Differentiating y = x1 2 with respect to x dx 2 x > 21 dy = dx. 2 x To find dy when2 x = 9 and dx = 1, we substitute: 1 1 dy = 1 = = 0.1666c. 2 9 6 2 1 2 b) The value ∆y is the actual change in y between x = 9 and x = 10: ∆y = 10 - 9 = 3.162277672 2 c - 3 Using a calculator = 0.16227767. We see that ∆y = 0.16227767 and dy = 0.1666c do not differ by much. Copyright 2016 Pearson. All rights reserved.

M02_BITT9391_11_SE_C02.6.indd 263 29/10/14 4:59 PM Not for Sale

264 Chapter 2 ● Applications of Differentiation

Quick Check 3 c) Note that 10 ≈ 9 + dy, so 10 ≈ 3 + 0.1666c = 3.1666c is a close For y = x, find dy when approximation2 of 102. 2 3 x = 100 and2 dx = -2. Compare 2 dy to ∆y, and estimate the value We see that the approximation and the actual change are reasonably close. of 98. dy ∆y It is easier to calculate the approximation since that involves fewer steps, but the 2 trade-off is some loss in accuracy. As long as dx is small, this loss in accuracy is often acceptable, as the following example illustrates.

eXAMPLe 4 Business: Cost and Tolerance. In preparation for laying new tile, Michelle measures the floor of a banquet hall and finds it to be square, measuring 100 ft by 100 ft. Suppose these measurements are accurate to {6 in. (the tolerance). a) Use a differential to estimate the difference in area (dA) due to the tolerance. b) Compare the result from part (a) with the actual difference in area ∆A . c) If each tile covers 1 ft2 and a box of 12 tiles costs $24, how many extra boxes of 1 2 tiles should Michelle buy for covering the floor? What is the extra cost? Solution a) The floor is a square, with a presumed measurement of 100 ft per side and a 2 tolerance of {6 in. = {0.5 ft. The area A in square feet ft for a square of side length x ft is 1 2 A x = x2. The derivative1 2 is dA dx = 2x, and solving for dA gives the differential of A: dA = 2x dx. > To finddA , we substitute x = 100 and dx = {0.5: dA = 2 100 {0.5 = {100. The value of1 dA is21 the approximate2 difference in area due to the inexactness in measuring. Therefore, if Michelle’s measurements are off by half a foot, the total 2 area can differ by approximately{100 ft . A small “error” in measurement can lead to a significant difference in the resulting area.

b) The actual difference in area, ∆A, is calculated. We set x1 = 100 ft, and let x2 represent the length plus or minus the tolerance. If the true length is at the low end, we have x2 = 99.5 ft. The floor’s area is then 99.52 = 9900.25 ft2. The actual difference in area is

∆A = A x2 - A x1 A 99.5 A 100 = 1 2 - 1 2 99.52 1002 = 1 -2 1 2 = 9900.25 - 10,000 2 = -99.75 ft .

If the true length is at the high end, we have x2 = 100.5 ft. The floor’s area is then 100.52 = 10,100.25 ft2. The actual difference in area is

∆A = A x2 - A x1 A 100.5 A 100 = 1 2 -1 2 100.52 1002 = 1 -2 1 2 = 10,100.25 - 10,000 = 100.25 ft2.

Copyright 2016 Pearson. All rights reserved.

M02_BITT9391_11_SE_C02.6.indd 264 29/10/14 4:59 PM Not for Sale Exercise Set 2.6 265

2 Quick Check 4 We see that ∆A, the actual difference in area, can range from -99.75 ft to 2 2 The four walls of a room measure 100.25 ft . Both values compare well with the approximate value of dA = {100 ft . 10 ft by 10 ft each, with a tolerance c) The tiles (each measuring 1 ft2) come 12 to a box. Thus, if the room were exactly 100 ft 2 of {0.25 ft. by 100 ft (an area of 10,000 ft ), Michelle would need 10,000 12 = 833.33 c , a) Calculate the approximate dif- or 834 boxes to cover the floor. To take into account the possibility that the room 2 > ference in area, dA, for the four is larger by 100 ft , she needs a total of 10,100 12 = 841.67 c , or 842 boxes of walls. tiles. Therefore, she should buy 842 - 834, or 8 extra boxes of tiles, for an extra cost of 8 24 192. > 4 b) Workers will texture the four = + walls using “knockdown” spray. 1 21 2 Each bottle of knockdown spray Historically, differentials were quite valuable when used to make approximations. costs $9 and covers 12 ft2. How However, with the advent of computers and graphing calculators, such use has dimin- many extra bottles should the ished considerably. The use of marginals remains important in the study of business workers buy to allow for overage and economics. in wall area? What will the extra cost be?

Section Summary ● If C x represents the cost of producing x items, from the x + 1 st item can be approximated by then marginal cost C′ x is its derivative, and P x + 1 ≈ P x + P′ x . C′ x1 2≈ C x + 1 - C x . Thus, the cost to pro- ● In delta notation1 ,2 ∆x = x + h - x = h, and duce the x + 1 st item1 2 can be approximated by ∆1y = f 2x + h 1 -2 f x . 1For2 small values of ∆x, we have 1 2 1 2 1 2 1 2 C x + 1 ≈ C x + C′ x . ∆y ● If R x represents1 2 the revenue from selling x items, ≈ f1′ x , which2 1 is2 equivalent to ∆y ≈ f′ x ∆x. ∆x then1 marginal2 revenue1 2 R′1 x2 is its derivative, and 1 2 1 2 dy 1 2 R′ x ≈ R x + 1 - R x . Thus, the revenue ● The differential of x is dx = ∆x. Since = f′ x , we from the x + 1 st item can1 2 be approximated by dx R 1x 2+ 1 ≈1 R x 2+ R′1x2. have dy = f′ x dx. In general, dy ≈ ∆y, and 1the2 ● If P x represents1 2 profit from sellingx items, approximation is often very close for sufficiently then1 marginal2 profit1 2 P′ x1 2is its derivative, and small dx. 1 2 P′ x1 2≈ P x + 1 - P x . Thus, the profit 1 2 1 2 1 2 1 2 2.6 Exercise Set

APPLicAtions find each of the following. a) P x Business and Economics b) R 100 , C 100 , and P 100 1 2 1. Marginal revenue, cost, and profit. Let R x , C x , and c) R′ x , C′ x , and P′ x P x be, respectively, the revenue, cost, and profit, in d) R1′ 1002 , C1′ 1002 , and 1P′ 1002 dollars, from the production and sale of x1 items.2 1 If2 e) Describe1 2 1what2 each 1quantity2 in parts (b) and (d) 1 2 1 2 1 2 1 2 2 represents. R x = 50x - 0.5x and C x = 4x + 10, find each of the following. 3. Marginal cost. Suppose the daily cost, in hundreds of 1 2 1 2 dollars, of producing x security systems is a) P x 3 2 b) R 20 , C 20 , and P 20 C x = 0.002x + 0.1x + 42x + 300, c) R1′ x2 , C′ x , and P′ x and currently 40 security systems are produced daily. 1 2 d) R1′ 202 , C1′ 202 , and 1P′ 220 1 2 1 2 1 2 a) What is the current daily cost? 2. Marginal1 2revenue,1 2 cost, and1 profit.2 Let R x , C x , and b) What would be the additional daily cost of increasing P x be, respectively, the revenue, cost, and profit, in production to 41 security systems daily? dollars, from the production and sale of x1 items.2 1 If2 c) What is the marginal cost when x = 40? 1 2 2 d) Use marginal cost to estimate the daily cost of increasing R x = 5x and C x = 0.001x + 1.2x + 60, production to 42 security systems daily. 1 2 1 2 Copyright 2016 Pearson. All rights reserved.

M02_BITT9391_11_SE_C02.6.indd 265 29/10/14 4:59 PM Not for Sale

266 Chapter 2 ● Applications of Differentiation

4. Marginal cost. Suppose the monthly cost, in dollars, of c) What is the marginal profit when x = 9? producing x daypacks is d) Use the answers from parts a 9 c to estimate the 3 2 profit resulting from the production and sale of C x = 0.001x + 0.07x + 19x + 700, 10 laptops weekly. 1 2 1 2 and currently 25 daypacks are produced monthly. 1 2 a) What is the current monthly cost? 9. sales. Let N x be the number of computers sold b) What would be the additional cost of increasing annually when the price is x dollars per computer. 1 2 production to 26 daypacks monthly? Explain in words what occurs if N 1000 = 500,000 and c) What is the marginal cost when x 25? N′ 1000 = -100. = 1 2 d) Use marginal cost to estimate the difference in 10. sales.1 Estimate2 the number of computers sold in Exercise 9 cost between producing 25 and 27 daypacks per if the price is raised to $1025. month. e) Use the answer from part (d) to predict C 27 . For Exercises 11–16, assume that C x and R x are in dollars and x is the number of units produced and sold. 5. Marginal revenue. Pierce Manufacturing determines1 2 1 2 1 2 that the daily revenue, in dollars, from the sale of x lawn 11. For the total-cost function chairs is C x = 0.01x2 + 1.6x + 100, 3 2 R x = 0.005x + 0.01x + 0.5x. find C and C x when x 80 and x 1. 1∆2 ′ = ∆ = Currently, Pierce sells 70 lawn chairs daily. 1 2 12. For the total-cost1 2 function a) What is the current daily revenue? 2 b) How much would revenue increase if 73 lawn chairs C x = 0.01x + 0.6x + 30, were sold each day? find C and C x when x 70 and x 1. 1∆2 ′ = ∆ = c) What is the marginal revenue when 70 lawn chairs are sold daily? 13. For the total-revenue1 2 function d) Use the answer from part (c) to estimate R 71 , R 72 , R x = 2x, and R 73 . find R and R x when x 70 and x 1. 1 2 1 2 1 ∆2 ′ = ∆ = 6. Marginal profit. For Sunshine Motors, the weekly profit, 1 2 14. For the total-revenue1 2 function in dollars, from selling x cars is R x = 3x, P x = -0.006x3 - 0.2x2 + 900x - 1200, find ∆R and R′ x when x = 80 and ∆x = 1. and currently 60 cars are sold weekly. 1 2 1 2 a) What is the current weekly profit? 15. a) Using C x from1 2 Exercise 11 and R x from Exercise 14, b) How much profit would be lost if the dealership were find the total profit, P x . 1 2 1 2 able to sell only 59 cars weekly? b) Find ∆P and P′ x when x = 80 and ∆x = 1. 1 2 c) What is the marginal profit when x 60? = 16. a) Using C x from1 Exercise2 12 and R x from Exercise 13, d) Use marginal profit to estimate the weekly profit if find the total profit, P x . sales increase to 61 cars weekly. b) Find ∆P1 and2 P′ x when x = 70 1and2 ∆x = 1. 1 2 7. Marginal revenue. Solano Carriers finds that its 17. Marginal supply. The1 2 supply S, of a new rollerball pen is monthly revenue, in dollars, from the sale of x carry-on given by suitcases is 3 2 S = 0.007p - 0.5p + 150p, R x = 0.007x3 - 0.5x2 + 150x. where p is the price in dollars. Currently Solano is selling 26 carry-on suitcases monthly. 1 2 a) Find the rate of change of quantity with respect to a) What is the current monthly revenue? price, dS dp. b) How much would revenue increase if sales increased b) How many units will producers want to supply when from 26 to 28 suitcases? the price> is $25 per unit? c) What is the marginal revenue when 26 suitcases are c) Find the rate of change at p = 25, and interpret this sold? result. d) Use the answers from parts (a)–(c) to estimate the d) Would you expect dS dp to be positive or negative? revenue resulting from selling 27 suitcases per Why? month. > 18. Average cost. The average cost for Turtlehead, Inc., to 8. Marginal profit. Crawford Computing finds that its produce x units of its thermal outerwear is given by the weekly profit, in dollars, from the production and sale of function x laptop computers is 13x + 100 P x 0.004x3 0.3x2 600x 800. A x = . = - - + - x Currently Crawford builds and sells 9 laptops weekly. 1 2 Use1 A2′ x to estimate the change in average cost as pro- a) What is the current weekly profit? duction goes from 100 units to 101 units. b) How much profit would be lost if production and 1 2 sales dropped to 8 laptops weekly? Copyright 2016 Pearson. All rights reserved.

M02_BITT9391_11_SE_C02.6.indd 266 29/10/14 4:59 PM Not for Sale Exercise Set 2.6 267

19. Marginal productivity. An employee’s monthly produc- (406,751, 39.6) 40% tivity, M, in number of units produced, is found to be a (405,101, 35) function of the number of years of service, t. For a cer- (89,351, 28) tain product, the productivity function is given by 30 (186,351, 33) M t = -2t2 + 100t + 180. (36,901, 25) 20 a) Find the productivity of an employee after 5 yr, 10 yr, 1 2 25 yr, and 45 yr of service. (9,076, 15) 10 b) Find the marginal productivity. (9,075, 10) Tax rate (percentage) c) Find the marginal productivity at t = 5, t = 10, t = 25, t = 45; and interpret the results. 100 200 300 400 500 d) Explain how an employee’s marginal productivity Annual income (thousands of dollars) might be related to experience and to age. (Source: irs.gov, 2014.)

20. supply. A supply function for a certain product is given by 23. Was the taxation in 2014 progressive? Why or why not? S p 0.08p3 2p2 10p 11, = + + + 24. Marcy and Tyrone work for the same marketing agency. where S p is the number of items produced when the 1 2 Because she is not yet a partner, Marcy’s year-end income price is p dollars. Use S′ p to estimate how many more is approximately $95,000; Tyrone’s year-end income is units a producer1 2 will supply when the price changes from approximately $185,000. Suppose one of them is to re- $18.00 per unit to $18.201 per2 unit. ceive another $5000 in income for the year. Which one would keep more of that $5000 after taxes? Why? 21. gross domestic product. The U.S. gross domestic prod- uct, in billions of current dollars, may be modeled by the 25. Alan earns $85,000 per year and is considering a second function job that would earn him another $10,000 annually. At P x = 567 + x 36x0.6 - 104 , what rate will his tax liability (the amount he must pay in taxes) change if he takes the extra job? Express your where x is the number of years since 1960. (Source: U.S. 1 2 1 2 answer in tax dollars paid per dollar earned. Bureau for Economic Analysis.) Use P′ x to estimate how much the gross domestic product increased from 26. Iris earns $50,000 per year and is considering extra 2014 to 2015. 1 2 work that would earn her an extra $3000 annually. At what rate will her tax liability grow if she takes the extra 22. Advertising. Norris Inc. finds that it sells N units of a work (see Exercise 25)? product after spending x thousands of dollars on adver- tising, where Find ∆y and f x ∆x. Round to four and two decimal places, N x = x2 300x 6. respectively. - + + 1 2 Use N x to estimate how many more units Norris 27. For y f x x3, x 2, and x 0.01 1 2′ = = = ∆ = will sell by increasing its advertising expenditure from 2 $100,0001 2 to $101,000. 28. For y = f1x2 = x , x = 2, and ∆x = 0.01 2 Marginal tax rate. Businesses and individuals are frequently 29. For y = f1x2 = x + x , x = 3, and ∆x = 0.04 2 concerned about their marginal tax rate, or the rate at which 30. For y = f1x2 = x - x , x = 3, and ∆x = 0.02 the next dollar earned is taxed. In progressive taxation, the 80,001st dollar earned is taxed at a higher rate than the 31. For y = f1x2 = 1 x, x = 1, and ∆x = 0.2 25,001st dollar earned and at a lower rate than the 140,001st 32. For y f x 1 x2, x 1, and ∆x 0.5 dollar earned. Use the following graph, showing the marginal = 1 2 = > = = tax rate for 2014 for single filers, to answer Exercises 23–26. 33. For y = f1x2 = 3x> - 1, x = 4, and ∆x = 2 34. For y f x 2x 3, x 8, and ∆x 0.5 SinglE filErS = 1 2 = - = = inComE BraCkEt ratE Use ∆y ? f x1 2∆x to find a decimal approximation of each 0–9075 10% radical expression. Round to three decimal places. 1 2 9076–36900 15% 35. 26 36. 8 36901–89350 25% 2 2 37. 102 38. 103 89351–186350 28% 23 23 186351–405100 33% 39. 1005 40. 28 2 2 405101–406750 35% Find dy. 406751+ 39.6 41. y = 3x - 2 42. y = x + 1 2 2 43. y = 2x3 + 1 3 2 44. y = x3 2x + 5 2 > 1 2 1 2 Copyright 2016 Pearson. All rights reserved.

M02_BITT9391_11_SE_C02.6.indd 267 29/10/14 4:59 PM Not for Sale

268 Chapter 2 ● Applications of Differentiation

3 5 x + x + 2 46. y = x + 27 Business and Economics 45. y = 2 x + 3 2 59. Marginal average cost. In Section 1.6, we defined the aver- 4 3 2 age cost of producing x units of a product in terms of the 47. y = x - 2x + 5x + 3x - 4 total cost C x by A x = C x x. Find a general expres- 48. y = 7 - x 8 sion for marginal average cost, A′ x . 1 2 1 2 1 2> 49. In Exercise1 48,2 find dy when x = 1 and dx = 0.01. 60. Cost and tolerance. A firm contracts1 2 to paint the exte- rior of a large water tank in the shape of a half-dome (a 50. In Exercise 47, find dy when x = 2 and dx = 0.1. hemisphere). The radius of the tank is measured to be 51. For y = 3x - 10 5, find dy when x = 4 and dx = 0.03. 100 ft with a tolerance of {6 in. ({0.5 ft). (The formula for the surface area of a hemisphere is A = 2pr2; use 5 3 52. For y = x1 - 2x 2- 7x, find dy when x = 3 and 3.14 as an approximation for p.) Each can of paint costs dx = 0.02. $30 and covers 300 ft2. 53. For f x = x4 - x2 + 8, use a differential to approximate a) Calculate dA, the approximate difference in the f 5.1 . surface area due to the tolerance. 1 2 b) Assuming the painters cannot bring partial cans of paint 3 54. For1 f2x = x - 5x + 9, use a differential to approximate to the job, how many extra cans should they bring to f 3.2 . cover the extra area they may encounter? 1 2 c) How much extra should the painters plan to spend on synthesis1 2 paint to account for the possible extra area? life and Physical Sciences 61. strategic oil supply. The U.S. Strategic Petroleum Reserve (SPR) stores petroleum in large spherical caverns built 55. Body surface area. Certain chemotherapy dosages de- into salt deposits along the Gulf of Mexico. (Source: U.S. pend on a patient’s surface area. According to the Gehan Department of Energy.) These caverns can be enlarged by and George model, filling the void with water, which dissolves the surround- S = 0.02235h0.42246w0.51456, ing salt, and then pumping brine out. Suppose a cavern has a radius of 400 ft, which engineers want to enlarge where h is the patient’s height in centimeters, w is his or by 2 ft. Use a differential to estimate how much volume her weight in kilograms, and S is the approximation to his will be added to form the enlarged cavern. (The formula or her surface area in square meters. (Source: www.halls. 4 for the volume of a sphere is V pr3; use 3.14 as an md.) Joanne is 160 cm tall and weighs 60 kg. Use a dif- = 3 approximation for p.) ferential to estimate how much her surface area changes after her weight decreases by 1 kg. Marginal revenue. In each of Exercises 62–66, a demand function, p  D x , expresses price, in dollars, as a function 56. Medical dosage. The function of the number of items produced and sold. Find the marginal 0.8t + 1000 revenue. 1 2 N t = 5t + 4 62. p = 100 - x 63. p = 400 - x 1 2 2 gives the bodily concentration N t , in parts per million, 4000 64. p = 500 - x 65. p = + 3 of a dosage of medication after t hours. Use differentials x to determine whether the concentration1 2 changes more from 1.0 hr to 1.1 hr or from 2.8 hr to 2.9 hr. 3000 66. p = + 5 general interest x 57. Major league ticket prices. The average ticket price of 67. Look up “differential” in a book or Web site devoted a major league baseball game can be modeled by the to math history. In a short paragraph, describe your function findings. p x = 0.06x3 - 0.5x2 + 1.64x + 24.76, 68. Explain the uses of the differential. where x is the number of years after 2008. (Source: Major 1 2 League Baseball.) Use differentials to predict whether answers to Quick Checks ticket price will increase more between 2010 and 2012 or 1. (a) P x 0.5x2 40x 3; (b) C 40 403, between 2014 and 2016. = - + - = R 40 = 1200, P 40 = 797; (c) C′ 40 = 10, 1 2 1 2 58. Suppose a rope surrounds the earth at the equator. The R′ 40 = 10, P′ 40 = 0 2. ∆y = -3.1319875 1 2 1 2 1 2 rope is lengthened by 10 ft. By about how much is the 3. dy = -0.1, ∆y = -0.1005050634c. The two values 1 2 1 2 2 rope raised above the earth? are very close; 98 ≈ 9.9. 4. (a) {20 ft ; (b) $18 (for 2 extra bottles)2

Copyright 2016 Pearson. All rights reserved.

M02_BITT9391_11_SE_C02.6.indd 268 29/10/14 4:59 PM Not for Sale

2.7 ● Elasticity of Demand 269

Elasticity of Demand Retailers and manufacturers often need to know how a small change in price will affect 2.7 the demand for a product. If a small increase in price produces no change in demand, a price increase may make sense; if a small increase in price creates a large drop in demand, the increase is probably ill advised. To measure the sensitivity of demand to a ● Find the elasticity of a demand small percent increase in price, economists calculate the elasticity of demand. function. Suppose that Klix Video has found that demand for rentals of its DVDs is given by ● Find the maximum of a total-revenue function. q = D x = 120 - 2x, ● Characterize demand in terms where q is the number1 2 of DVDs rented per day at x dollars per rental. Klix Video wants of elasticity. to determine what effect a small increase in the price per rental will have on revenue. If the price per rental is currently $2, the demand is q = D 2 = 120 - 20 2 = 80 DVDs per day. If the price per rental is raised by a small amount, such as 10%, to $2.20, the demand will be q = D 2.2 = 120 - 20 2.2 = 76 DVDs1 2 per day, a 5%1 2decrease. The percent decrease in demand is smaller than the percent increase in price. In this case, the total revenue grows1 from2 $2 per rental1 2 80 rentals , or $160, to $2.20 per rental 76 rentals , or $167.20. If the price per rental is currently $4, the1 demand is21 q = D 4 2 = 120 - 2014 = 40 DVDs 21per day. If the2 price per rental is raised by 10% to $4.40, the de- mand will be q = D 4.4 = 120 - 20 4.4 = 32 DVDs per day, a 20%1 2 decrease. Here,1 2 the percent decrease in demand is larger thant the percent increase in price. In this case, the total1 revenue2 decreases1 from2 $4 per rental 40 rentals , or $160, to $4.40 rentals 32 rentals , or $140.80. Let’s develop a general formula for elasticity1 of demand.21 Suppose q2 represents a quantity1 of goods21 purchased2 and x is the price per unit of the goods. Recall that q and x are related by the demand function, D, where q = D x . For any change,1 ∆2x, in the price per unit, the percent change in price is ∆x ∆x 100 ∆x # 100 = # = %. x x 100 x A change in price produces a change, ∆q, in the quantity demanded, and the percent change in this quantity is ∆q ∆q # 100 = %. q q

The ratio of percent change in quantity sold to percent change in price is ∆q q , ∆x x > which can be> expressed as x ∆q # . q ∆x Since, for differentiable functions, ∆q dq lim = , ∆xS0 ∆x dx we have x ∆q x dq x x lim # = # = # D′ x = # D′ x . ∆xS0 q ∆x q dx q D x 1 2 1 2 1 2 Copyright 2016 Pearson. All rights reserved.

M02_BITT9391_11_SE_C02.7.indd 269 29/10/14 5:01 PM Not for Sale

270 Chapter 2 ● Applications of Differentiation

This result is the basis of the following definition.

Definition The elasticity of demand E is given as a function of price x by x # D′ x E x = - . D x 1 2 1 2 1 2 To understand the purpose of the negative sign in the preceding definition, note that the price, x, and the demand, D x , are both nonnegative. Since D x is normally decreasing, D′ x is usually negative. By inserting a negative sign in the definition, economists make E x nonnegative1 2and easier to work with. 1 2 1 2 eXAMPLe 1 Economics:1 2 Elasticity of Demand for DVD Rentals. The demand for DVD rentals at Klix Video is given by q = D x = 120 - 20x, where q is the number1 2 of DVDs rented per day at x dollars per rental. Find each of the following. a) The elasticity as a function of x b) The elasticity at x = 2 and at x = 4. Interpret the meaning of these values of the elasticity. c) The value of x for which E x = 1. Interpret the meaning of this price. d) The total-revenue function, R x x D x 1 2 = # e) The price x at which total revenue is a maximum 1 2 1 2 Solution a) To find the elasticity, we first find the derivative D′ x :

D′ x = -20. 1 2 Then we1 substitute2 -20 for D′ x and 120 - 20x for D x in the expression for elasticity: 1 2 1 2 x # D′ x x # -20 20x x E x = - = - = = . D x 120 20x 120 20x 6 x 1 2 1- 2 - - 1 2 2 1 b) E 2 = = 1 2 6 - 2 2 1 At1 x2 = 2, the elasticity is 2, which is less than 1. Thus, the ratio of percent change in quantity to percent change in price is less than 1. A small percentage increase in price will cause an even smaller percentage decrease in the quantity demanded. 4 E 4 = = 2 6 - 4 1 2 At x = 4, the elasticity is 2, which is greater than 1. Thus, the ratio of percent change in quantity to percent change in price is greater than 1. A small percentage increase in price will cause a larger percentage decrease in the quantity demanded. c) We set E x = 1 and solve for x: x1 2 = 1 6 - x x = 6 - x We multiply both sides by 6 - x, assuming that x  6. 2x = 6 x = 3. Copyright 2016 Pearson. All rights reserved.

M02_BITT9391_11_SE_C02.7.indd 270 29/10/14 5:01 PM Not for Sale

2.7 ● Elasticity of Demand 271

Quick Check 1 Thus, when the price is $3 per rental, the ratio of the percent change in quantity to Economics: Demand for DVD the percent change in price is 1. Rentals. Internet rentals affect d) Recall that total revenue R x is given by x # D x . Then Klix Video in such a way that the 2 R x = x # D x = x11202 - 20x = 1201x2- 20x . demand for rentals of its DVDs changes to e) To find1 the2 pricex 1that2 maximizes1 total2 revenue, we find R′ x : q = D x = 30 - 5x. R′ x = 120 - 40x. 1 2 a) Find the quantity1 2 demanded We see that1 2 R′ x exists for all x in the interval 0, ∞). Thus, we solve: when the price is $2 per R ′ x = 11202 - 40x = 0 3 rental, $3 per rental, and 40x 120 $5 per rental. 1 2 - = - b) Find the elasticity of demand x = 3. as a function of x. Since there is only one critical value, we use the second derivative to see if it yields c) Find the elasticity at x = 2, a maximum: x = 3, and x = 5. Interpret the meaning of these values. R″ x = -40 6 0. d) Find the value of x for which Thus, R″1 32 is negative, so R 3 is a maximum. That is, total revenue is a maximum E x = 1. Interpret the at $3 per rental. 1 meaning of this price. 1 2 1 2 e) Find1 2 the total-revenue Note in parts (c) and (e) of Example 1 that the value of x for which E x = 1 is function, R x = x # D x . the same as the value of x for which total revenue is a maximum. The following theo- f) Find the price x at which total rem states that this is always the case. 1 2 revenue is a1 maximum.2 1 2

theoreM 11 Total revenue is increasing at those x-values for which E x 6 1. Total revenue is decreasing at those x-values for which E x 1. 1 2 7 Total revenue is maximized at the value(s) of x for which E x 1. 1 2 = 1 2 Proof: We know that R x = x # D x , so R′ x x D′ x D x 1 Using the Product Rule 1 2 = # 1 2 + # x # D′ x 1 2 = D x 1 2 1 2+ 1 Factoring; check this by multiplying. D x 1 2 D1x2c E x 1 d = - 1 2 + D x 1 E x . = 1 23 -1 2 4 Since we can assume that1 23D x 7 10,24 it follows that R′ x is positive for E x 6 1, is negative for E x 7 1, and is 0 when E x = 1. Thus, total revenue is increasing for E x 6 1, is decreasing for E1 x2 7 1, and is maximized1 2when E x = 1. 1 2 1 2 1 2 1 2In summary, suppose Klix1 2 Video in Example 1 raises the 1price2 per rental and the total revenue increases. Then we say the demand is inelastic. If the total revenue decreases, we say the demand is elastic. In a typical economy, goods with inelastic demands tend to be items people purchase regularly, such as food, clothing, and fuel. These goods tend to be purchased in set amounts, regardless of small fluctuations in price. Goods with elastic de- mands tend to be larger items purchased occasionally, such as a vehicle or furniture. Purchases of such items can be delayed until the price falls.

Copyright 2016 Pearson. All rights reserved.

M02_BITT9391_11_SE_C02.7.indd 271 29/10/14 5:01 PM Not for Sale

272 Chapter 2 ● Applications of Differentiation

elasticity and revenue For a particular value of the price x: 1. The demand is inelastic if E x 6 1. An increase in price will bring an increase in revenue. If demand is inelastic, then revenue is increasing. 1 2 2. The demand has unit elasticity if E x = 1. The demand has unit elasticity when revenue is at a maximum. 1 2 3. The demand is elastic if E x 7 1. An increase in price will bring a decrease in revenue. If demand is elastic, then revenue is decreasing. 1 2

q Inelastic Unit elasticity Elastic

Demand function, E(x) < 1 q = D(x)

E(x) = 1 E(x) > 1

x1 x Price

R(x) Inelastic demand Elastic demand E = 1 Total revenue Total Increasing revenue Decreasing revenue

x1 x Price

Section Summary ● For a demand function q = D x , where x is in dollars, ● Demand is elastic when E x 7 1. If demand is elastic, the elasticity of demand is then revenue is decreasing. 1 2 ● Demand has unit elasticity1 when2 E x = 1. If xD′ x E x = - . demand has unit elasticity at price x, then revenue is D x 1 2 1 2 maximized. 1 2 ● Demand is inelastic1 2 when E x 6 1. If demand is inelastic, then revenue is increasing. 1 2

Copyright 2016 Pearson. All rights reserved.

M02_BITT9391_11_SE_C02.7.indd 272 29/10/14 5:01 PM Not for Sale Exercise Set 2.7 273

2.7 Exercise Set

For the demand function given in each of Exercises 1–10, find 12. Demand for chocolate chip cookies. Good Times Bakers the following. determines that the demand function for its chocolate a) The elasticity chip cookies is b) The elasticity at the given price, stating whether the q = D x = 967 - 25x, demand is elastic or inelastic where q is the quantity of cookies sold when the price c) The value(s) of x for which total revenue is a maximum 1 2 is x cents per cookie. (assume that x is in dollars) a) Find the elasticity. 1. q = D x = 400 - x; x = 125 b) At what price is elasticity of demand equal to 1? c) At what prices is elasticity of demand elastic? 2. q D x 500 x; x 38 = 1 2 = - = d) At what prices is elasticity of demand inelastic? 3. q = D1x2 = 200 - 4x; x = 46 e) At what price is the revenue a maximum? f) At a price of 20¢ per cookie, will a small increase 4. q = D1x2 = 500 - 2x; x = 57 in price cause total revenue to increase or decrease? 1 2 400 5. q = D x = ; x = 50 x 13. Demand for computer games. High Wire Electronics determines the following demand function for a new game: 1 2 3000 6. q = D x = ; x = 60 3 x q = D x = 200 - x , where q is the number2 of games sold per day when the 1 2 1 2 7. q = D x = 600 - x; x = 100 price is x dollars per game. 2 8. q D x 300 x; x 250 = 1 2 = - = y 2 15 1 2 100 9. q = D x = 2 ; x = 1 x + 3 1 2 1 5002 10. q = D x = 2 ; x = 8 2x 12 sold per day + 3

Number of games D(x) = √200 − x 1 2 1 2 1 APPLicAtions 16x Business and Economics Price (in dollars) a) Find the elasticity. 11. Demand for oil. Suppose you have been hired as an b) Find the elasticity when x 3. economic consultant concerning the world demand for = c) At x 3, will a small increase in price cause total oil. The demand function is = revenue to increase or decrease? 2 q = D x = 50,000 + 300x - 3x , for 0 … x … 180, 14. Demand for tomato plants. Sunshine Gardens where q is measured in millions of barrels of oil per day 1 2 determines the following demand function during early at a price of x dollars per barrel. summer for tomato plants: a) Find the elasticity. 2x + 300 b) Find the elasticity at a price of $75 per barrel, stating q = D x = , whether demand is elastic or inelastic at that price. 10x + 11 c) Find the elasticity at a price of $100 per barrel, stating where q 1is 2the number of plants sold per day when the whether demand is elastic or inelastic at that price. price is x dollars per plant. d) Find the elasticity at a price of $25 per barrel, stating whether demand is elastic or inelastic at that q price. 30 e) At what price is revenue a maximum? f) What quantity of oil will be sold at the price that maximizes revenue? Compare the current world price 2x + 300 D(x) = Ð ÐÐÐ to your answer. sold per day 10x + 11 g) At a price of $110 per barrel, will a small increase Number of plants 5 in price cause total revenue to increase or 120 x decrease? Price (in dollars)

Copyright 2016 Pearson. All rights reserved.

M02_BITT9391_11_SE_C02.7.indd 273 29/10/14 5:01 PM Not for Sale

274 Chapter 2 ● Applications of Differentiation

a) Find the elasticity. b) Is the value of the elasticity dependent on the price b) Find the elasticity when x = 3. per unit? c) At $3 per plant, will a small increase in price cause c) For what value of n does total revenue have a total revenue to increase or decrease? maximum?

synthesis 17. Explain in your own words the concept of elasticity and its usefulness to economists. Do research or consult an 15. Business. Tipton Industries determines that the demand economist to determine when and how this concept was function for its sunglass cases is first developed. q = D x = 180 - 10x, 18. Explain how the elasticity of demand for a product where q is the quantity of sunglass cases sold when the can be affected by the availability of substitutes for the 1 2 price is x dollars per case. product. a) Find the elasticity of demand. b) Find the elasticity when x = 8. c) At what price is revenue a maximum? Answers to Quick Check d) If the price is $8 and Tipton Industries raises the x 1. (a) 20, 15, 5; (b) E x = ; (c) 0.5, 1, 5 price by 20%, will revenue increase or decrease? Does 6 - x this contradict the answers to parts (b) and (c)? Why (see Example 1 for the interpretations);1 2 or why not? (d) $3; (e) R x = 30x - 5x2; (f) $3 16. Economics: constant elasticity curve. 1 2 a) Find the elasticity of the demand function k q = D x = , xn where k 1is 2a positive constant and n is an integer greater than 0.

Implicit Differentiation and Related Rates* 2.8 We often write a function with the output variable (such as y) isolated on one side of the equation. For example, if we write y = x3, we have expressed y as an explicit ● Differentiate implicitly. function of x. With an equation like y3 + x2y5 - x4 = -11, it may be difficult to ● Solve related-rate problems. isolate the output variable. In such a case, we have an implicit relationship between the variables x and y and can find the derivative ofy with respect to x using implicit differentiation.

Implicit Differentiation Consider the equation 3 y y = x. This equation implies that y is a function of x, for if we solve for y, we get 2 y3 = x 3 1 y = x1/3 y = x = x21 3. −10 −5 510 x > −1 We know from our earlier work that

−2 dy 1 = x-2 3. (1) dx 3 >

*This section can be omitted without loss of continuity.

Copyright 2016 Pearson. All rights reserved.

M02_BITT9391_11_SE_C02.7.indd 274 29/10/14 5:01 PM Not for Sale

2.8 ● Implicit Differentiation and Related Rates 275

A method known as implicit differentiation allows us to find dy dx without solving for y. To do so, we use the Chain Rule, treating y as a function of x, and differentiate both sides of > y3 = x with respect to x: d d y3 = x. dx dx The derivative on the left side is found using the Extended Power Rule: dy Remembering that the derivative of y with respect 3y2 = 1. dx to x is written dy dx Finally, we solve for dy dx by dividing both> sides by 3y2:

dy 1 > 1 = , or y-2. dx 3y2 3 We can show that this indeed gives us the same answer as equation (1) by replacing y with x1 3: dy> 1 1 1 = y-2 = x1 3 -2 = x-2 3. dx 3 3 3 > > Equations like 1 2 y3 + x2y5 - x4 = -11 determine y as a function of x, but are difficult to solve for y. We can nevertheless find a formula for the derivative of y without solving for y. To do so usually involves the Extended Power Rule in the form d dy yn = nyn - 1 # . dx dx

3 2 5 4 TEchnology connEcTion EXAMPLE 1 For y + x y - x = -11: a) Find dy dx using implicit differentiation. Exploratory b) Find the slope of the tangent line to the curve at the point 2, 1 . > Graphicus can be used to Solution graph equations that relate 1 2 2 5 x and y implicity. Press + a) We differentiate the term x y using the Product Rule. Because y is a function of x, and t hen g(x, y)=0, and enter it is critical that dy dx is included as a factor in the result any time a term involving 3 2 3 4 y is differentiated. When an expression involving just x is differentiated, there is no y + x y - x = 27 as factor dy dx. > y^3+x^2(y^3)-x^4= -11. > d 3 2 5 4 d Differentiating both y + x y - x = -11 ExErcisEs dx dx sides with respect to x Graph each equation. d 3 1 d 2 5 d 42 1 2 3 2 3 4 y + x y - x = 0 1. y + x y - x = -11 dx dx dx 2 3 3 Using the Extended Power 2. y x + 2x y = y + 1 2 dy 2 4 dy 5 3 3y # + x # 5y # + y # 2x - 4x = 0. dx dx Rule and the Product Rule

Copyright 2016 Pearson. All rights reserved.

M02_BITT9391_11_SE_C02.8.indd 275 29/10/14 5:05 PM Not for Sale

276 Chapter 2 ● Applications of Differentiation

We next isolate those terms with dy dx as a factor on one side:

2 dy 2 4 dy 3 > 5 3 5 3y # + 5x y # = 4x - 2xy Adding 4x - 2xy to both sides dx dx

2 2 4 dy 3 5 3y + 5x y = 4x - 2xy Factoring dy dx dx 3 5 1 2dy 4x - 2xy Solving for dy> dx and leaving the

= 2 2 4. answer in terms of x and y dx 3y + 5x y > b) To find the slope of the tangent line to the curve at 2, 1 , we replace x with 2 and y with 1: 1 2 3 5 y dy 4 # 2 - 2 # 2 # 1 28 . = 2 2 4 = 2 dx 3 # 1 + 5 # 2 # 1 23 28 Slope = ⎯ 23 1 (2, 1)

−2 −1 12x

−1 Quick Check 1 2 3 3 For y x + 2x y = y + 1, −2 y3 + x2y5 − x4 = −11 find dy dx using implicit differentiation. 1 > It is common for the expression for dy dx to contain both variables x and y. When this occurs, we calculate a slope by evaluating the derivative at both the x-value and the y-value of the point of tangency. > The steps in Example 1 are typical of those used when differentiating implicitly.

To differentiate implicitly: a) Differentiate both sides of the equation with respect to x (or whatever variable you are differentiating with respect to). b) Apply the rules for differentiation (the Power, Product, Quotient, and Chain Rules) as necessary. Any time an expression involving y is differentiated, dy dx will be a factor in the result. c) Isolate all terms with dy dx as a factor on one side of the equation. d) If necessary,> factor out dy dx. e) If necessary, divide both> sides of the equation to isolate dy dx. > > The demand function for a product (see Section R.5) is often given implicitly.

EXAMPLE 2 Differentiate the demand equation x = 200 - p3, implicitly to find dp dx. 2

> d d 3 Solution x = 200 - p dx dx 2 1 dp Using the Extended Power 1 = 200 - p3 -1 2 # -3p2 # Quick Check 2 2 dx Rule twice > Differentiate the equation 1 -3p2 2 dp 1 2 1 2 = # y = 2 1 + p 2 200 - p3 dx 2 dp 2 200 - p3 dp2 implicitly to find . 2 = dy 2 -3p dx 2

Copyright 2016 Pearson. All rights reserved.

M02_BITT9391_11_SE_C02.8.indd 276 29/10/14 5:05 PM Not for Sale

2.8 ● Implicit Differentiation and Related Rates 277

related rates Suppose y is a function of x, y = f x , and x is a function1 2 of time, t. Since y depends on x and x depends on t, it follows that y depends on t. The Chain Rule gives the following: dy dy dx = # . dt dx dt Thus, the rate of change of y is related to the rate of change of x. When this comes up in problems, it helps to keep in mind that any variable can be thought of as a func- tion of time t. If no expression in terms of t is given, the variable’s rate of change with respect to t will be 0.

EXAMPLE 3 Business: service Area. A restaurant supplier services the restau- rants in a circular area in such a way that the radius r is increasing at the rate of 2 mi per year at the moment when r = 5 mi. At that moment, how fast is the area increasing?

31 Rocky 37 465 Ripple 36

65 74 Lawrence

Clermont Indianapolis 70 465 5 mi Cumberland

40 Beech 52 70 Grove 31 65 74 465

Solution The area A and radius r are related by A = pr2. We take the derivative of both sides with respect to t, and then evaluate, using dr dt = 2 mi yr and r = 5 mi: > dA > dr The factor dr dt results from the Chain Rule and the = 2pr # . dt dt fact that r is assumed to be a function of t. > Quick Check 3 dA mi = 2p 5 mi 2 Substituting A spherical balloon is leaking, dt yr 3 losing 20 cm of air per minute. 1 mi22a b At the moment when the radius = 20p ≈ 63 mi2 yr. yr of the balloon is 8 cm, how fast is the radius decreasing? The supplier’s service area is growing> at a rate of about 63 square miles per year when 4 3 Hint: V = 3pr . r = 5 mi and dr dt = 2 mi yr. 3 1 2 > >

Copyright 2016 Pearson. All rights reserved.

M02_BITT9391_11_SE_C02.8.indd 277 29/10/14 5:05 PM Not for Sale

278 Chapter 2 ● Applications of Differentiation

EXAMPLE 4 Business: rates of change of revenue, cost, and Profit. For Luce Landscaping, the total revenue and the total cost for yard maintenance of x homes are given by 2 R x = 1000x - x , and C x = 3000 + 20x. Suppose that1 2 Luce is adding 10 homes1 2per day at the moment when the 400th customer is signed. At that moment, what is the rate of change of (a) total revenue, (b) total cost, and (c) total profit? Solution a) We regard R x = 1000x - x2 as R = 1000x - x2. Implicitly differentiating with respect to time t, we have 1 2 dR dx dx = 1000 # - 2x # Differentiating both sides with respect to time dt dt dt = 1000 # 10 - 2 400 10 Substituting 10 for dx dt and 400 for x $2000 per day. = 1 2 > b) We regard C x = 3000 + 20x as C = 3000 + 20x. Implicitly differentiating with respect to time t, we have 1 2 dC dx = 20 # Differentiating both sides with respect to time dt dt = 20 10 $200 per day. = 1 2 c) Since P x = R x - C x , we have dP1 2 dR1 2 dC 1 2 = - dt dt dt = $2000 per day - $200 per day = $1800 per day.

Section Summary ● If variables x and y are related to one another by an ● To determine the slope of a tangent line at a point on equation but neither variable is isolated on one side of the graph of an implicit relationship, we may need to the equation, we say that x and y have an implicit rela- evaluate the derivative by inserting both the x-value tionship. To find dy dx without solving such an equa- and the y-value of the point of tangency. tion for y, we use implicit differentiation. ● If two or more variables are related to one another by ● Whenever we implicitly> differentiate y with respect to x, an equation, then we may implicity differentiate with the factor dy dx will appear as a result of the Chain Rule. respect to time t. The result is a related rate.

>

Copyright 2016 Pearson. All rights reserved.

M02_BITT9391_11_SE_C02.8.indd 278 29/10/14 5:05 PM Not for Sale Exercise Set 2.8 279

2.8 Exercise Set

Differentiate implicitly to find dy,dx. Then find the slope of 31. Nonnegative variable quantities G and H are related by the curve at the given point. the equation 3 2 G2 H2 25. 1. 3x - y = 8; 2, 4 + = What is the rate of change dH dt when dG dt = 3 and 2. x3 2y3 6; 2, 1 + = 1 -2 G = 0? G = 1? G = 3? 3 2 > > 3. 2x + 4y = -12;1 -22, -1 32. Variable quantities A and B are related by the equation 2 3 3 3 4. 2x - 3y = 5; -2,1 1 2 A + B = 9. 11 3 2 What is the rate of change dA dt at the moment when 5. x2 + y2 = 1; , 2 A = 2 and dB dt = 3? 2 2 > a b 6. x2 - y2 = 1; 3, 2 APPLicATions> 3 2 2 2 7. 2x y = -18; 1 -1, 3 2 Business and Economics 2 4 8. 3x y = 12; 2,1 -1 2 rates of change of total revenue, cost, and profit. In 4 2 3 Exercises 33–36, find the rates of change of total revenue, 9. x - x y = 12;1 -22, 1 cost, and profit with respect to time. Assume that R x and 3 2 2 10. x - x y = -9; 1 3, -22 C x are in dollars. 1 2 2 2 11. xy + y - 2x = 0; 1 1, -22 33.1 2R x = 50x - 0.5x , C x 10x 3, 1 2 2 1 2 = + 12. 2 3; 5, xy - x + y = - when x 10 and dx dt 5 units per day 3 1 2 = = a b 2 13. 4x3 - y4 - 3y + 5x + 1 = 0; 1, -2 34. R x = 50x - 0.5x ,> 2 3 3 C x = 4x + 10, 14. x y - 2x - y + 1 = 0; 2, -13 2 1 2 when x 30 and dx dt 20 units per day 1 2 = = Differentiate implicitly to find dy 1dx. 2 , 2 2 2 35. R x = 280x - 0.4x>, 15. x + 2xy = 3y 16. 2xy + 3 = 0 2 C x = 5000 + 0.6x , 2 2 2 2 1 2 17. x - y = 16 18. x + y = 25 when x 200 and dx dt 300 units per day 1 2 = = 5 3 19. y3 = x5 20. y = x 36. R x = 2x, > 21. x2y3 x3y4 = 11 22. x3y2 x5y3 = 19 C x 0.01x2 0.6x 30, + + - 1 2 = + + when x 20 and dx dt 8 units per day For each demand equation in Exercises 23–30, differentiate 1 2 = = implicitly to find dp dx. , 37. change of sales. Suppose> that the price p, in dollars, and 23. p2 + p + 2x = 40 24. p3 + p - 3x = 50 number of sales, x, of a mechanical pencil are related by 3 3 2 5p 4x 2px = 60. 25. xp = 24 26. x p = 108 + + 2 If p and x are both functions of time, measured in days, x p + xp + 1 find the rate at which x is changing when 27. = 1 (Hint: Clear the fraction first.) 2x + p dp x = 3, p = 5, and = 1.5. xp dt 28. = 2 (Hint: Clear the fraction first.) x + p 38. change of revenue. For x and p as described in Exercise 37, 29. p + 4 x + 3 = 48 find the rate at which total revenue R = xp is changing when 2 dp 30. 10001 -21300p +2 25p = x x = 3, p = 5, and = 1.5. dt

Copyright 2016 Pearson. All rights reserved.

M02_BITT9391_11_SE_C02.8.indd 279 29/10/14 5:05 PM Not for Sale

280 Chapter 2 ● Applications of Differentiation

Life and Natural Sciences blood vessel, and viscosity of the blood, respectively. Assume that dV,dt is measured in millimeters per second squared 39. rate of change of the Arctic ice cap. In a trend that mm sec2 . Use this formula for Exercises 42 and 43. scientists attribute, at least in part, to global warming, , the floating cap of sea ice on the Arctic Ocean has been 1 2 shrinking since 1980. The ice cap always shrinks in summer and grows in winter. Average minimum size of the ice cap, in square miles, can be approximated by A pr2. = r In 2013, the radius of the ice cap was approximately R 792 mi and was shrinking at a rate of approximately 4.7 mi yr. (Source: Based on data from nsidc.org.) How L fast was the area changing at that time? >

42. Assume that r is a constant as well as p, L, and v. a) Find the rate of change dV dt in terms of R and dR dt when L = 80 mm, p = 500, and v = 0.003. b) A person goes out into the >cold to shovel snow. Cold> air has the effect of contracting blood vessels far from the heart. Suppose a blood vessel contracts at a rate of dR = -0.0002 mm sec dt at a place where the radius> of the vessel is R = 0.075 mm. Find the rate of change, dV dt, at that location. > 40. rate of change of a healing wound. The area of a heal- ing wound is given by A = pr2. The radius is decreasing at the rate of 1 millimeter per day -1 mm day at the moment when r = 25 mm. How fast is the area decreasing at that moment? 1 > 2 41. Body surface area. Certain chemotherapy dosages depend on a patient’s surface area. According to the Mosteller model, hw S = , 260 where h is the patient’s height in centimeters, w is the patient’s weight in kilograms, and S is the approximation The flow of blood in a blood vessel can be modeled by Poiseuille’s Law. of the patient’s surface area in square meters. (Source: 43. Assume that r is a constant as well as p, L, and v. www.halls.md.) Assume that Kim’s height is a constant a) Find the rate of change dV dt in terms of R and dR dt 165 cm, but she is losing weight. If she loses 2 kg per when L 70 mm, p 400, and v 0.003. month, how fast is her surface area decreasing at the in- = = = b) When shoveling snow in cold> air, a person with a > stant she weighs 70 kg? history of heart trouble can develop angina (chest Poiseuille’s law. The flow of blood in a blood vessel is pains) due to contracting blood vessels. To counteract faster toward the center of the vessel and slower toward the this, he or she may take a nitroglycerin tablet, outside. The speed of the blood, V, in millimeters per second which dilates the blood vessels. Suppose, after a mm sec , is given by nitroglycerin tablet is taken, a blood vessel dilates at , a rate of p 2 2 1 V 2 R - r , 4Lv dR = 0.00015 mm sec where R is the radius1 of the2 blood vessel, r is the distance of dt the blood from the center of the vessel, and p, L, and v are at a place where the radius> of the vessel is R = 0.1 mm. physical constants related to blood pressure, length of the Find the rate of change, dV dt.

> Copyright 2016 Pearson. All rights reserved.

M02_BITT9391_11_SE_C02.8.indd 280 29/10/14 5:05 PM Not for Sale Exercise Set 2.8 281

General Interest synThEsis 44. Two cars start from the same point at the same time. Differentiate implicitly to find dy,dx. One travels north at 25 mph, and the other travels east 1 1 at 60 mph. How fast is the distance between them 48. x y 1 49. 5 2 2 2 + = 2 + 2 = increasing at the end of 1 hr? (Hint: D = x + y . To x y 2 2 2 2 2 find D after 1 hr, solve D = 25 + 60 .) 2 3 x - 1 2 x - 1 50. y = 51. y = 2 x + 1 x + 1 3 2 2 3 52. x + y = 1 53. x > y 3 > x y 3 x5 y5 D - + + = + x 2 2 Differentiate1 2 implicitly1 to2 find d y,dx . 2 2 54. xy + x - 2y = 4 55. y - xy + x = 5 2 2 3 3 56. x - y = 5 57. x - y = 8

y 58. Explain the usefulness of implicit differentiation. 59. Look up the word “implicit” in a dictionary. Explain 45. A ladder 26 ft long leans against a vertical wall. If the how that definition can be related to the concept of a lower end is being moved away from the wall at the rate function that is defined “implicitly.” of 5 ft sec, how fast is the height of the top changing (this will be a negative rate) when the lower end is 10 ft from the> wall? TEchnology connEcTion

Graph each of the following equations. Equations must be solved for y before they can be entered into most calculators. Graphicus does not require that equations be solved for y. 2 2 60. x + y = 4 26 ft Note: You will probably need to sketch the graph in two 2 2 parts: y = 4 - x and y = - 4 - x . Then graph 26 ft y the tangent 2line to the graph at the2 point -1, 3 . 4 2 6 2 61. x = y + x 1 2 Then graph the tangent line to the graph at the point -0.8, 0.384 . x 4 2 2 62. 1y = y - x2 46. An inner city revitalization zone is a rectangle that is 3 2 63. x = y 2 - x twice as long as it is wide. A diagonal through the region 2 3 is growing at a rate of 90 m per year at a time when the 64. y = x 1 2 region is 440 m wide. How fast is the area changing at that point in time?

47. The volume of a cantaloupe is approximated by Answers to Quick Checks 4 3 2 2 3 2 V = 3pr . dy y + 6x y dp 1 + p 1. 2. = 3 2 = The radius is growing at the rate of 0.7 cm week, at a dx 1 - 2xy - 6x y dy 2 2p time when the radius is 7.5 cm. How fast is the volume 3. Approximately -0.025 cm min changing at that moment? > >

Copyright 2016 Pearson. All rights reserved.

M02_BITT9391_11_SE_C02.8.indd 281 29/10/14 5:05 PM Not for Sale

ChapTer 2 Summary

Key Terms and ConCepTs examples seCTion 2.1

A function is increasing over an open y interval I if, for all x in I, f′ x 7 0. f ′(x2) > 0 f ′(x4) < 0 A function is decreasing over an open 1 2 f ′(x ) < 0 interval I if, for all x in I, f′ x 6 0. 5 f ′(x1) > 0 1 2

x0 x1 x2 x3 x4 x5 x6 x

f is increasing over the interval x0, x3 and decreasing over x3, x6 . 1 2 1 2 If f is a continuous function, then a y critical value is any number c in the domain of f for which f′ c = 0 or f f′ c does not exist. 1 2 If 1f′2c does not exist, then the graph of f may have a corner or a vertical tangent1 2 at c, f c . c1 c2 c3 c4 c5 x In both cases,1 1the22 ordered pair c, f c is called a critical point. The values c1, c2, c3, c4, and c5 are critical values of f.

1 1 22 ● f′ c1 does not exist (corner). ● f′ c = 0. 1 22 ● f′ c does not exist (vertical tangent). 1 32 ● f′ c = 0. 1 42 ● f′ c = 0. 1 52 1 2 If f is a continuous function, then y any relative extremum (maximum or minimum) occurs at a critical value. f The converse is not true: a critical value may not correspond to an extremum.

c1 c2 c3 c4 c5 x

● The critical point c1, f c1 is a relative maximum. ● The critical point c , f c is a relative minimum. 1 2 1 222 ● The critical point c3, f c3 is neither a relative maximum nor a relative minimum. 1 1 22 1 1 22 ● The critical point c4, f c4 is a relative maximum. ● The critical point c5, f c5 is neither a relative maximum nor a relative minimum. 1 1 22 1 1 22

282 Copyright 2016 Pearson. All rights reserved.

M02_BITT9391_11_SE_C02_EOC.indd 282 29/10/14 5:08 PM Not for Sale Chapter 2 summary 283

Key Terms and ConCepTs examples seCTion 2.1 (continued) The First-Derivative Test allows us Find the relative extrema of the function given by to classify a critical value as a relative 1 1 f x = x3 x2 20x 7. maximum, a relative minimum, or 3 - 2 - + neither. Critical values occur where f′ x = 0 or f′ x does not exist. The 1 2 2 derivative f′ x = x - x - 20 exists for all real numbers, so the only critical 2 values occur when f′ x = 0. Setting1 2 x - x -1202 = 0 and solving, we have x = -4 and 1x 2= 5 as the critical values. To apply the First-Derivative1 2 Test, we check the sign of f′ x to the left and right of each critical value, using test values: 1 2

Test Value x = -5 x = 0 x = 6

Sign of f′ x f′ -5 7 0 f′ 0 6 0 f′ 6 7 0 Result 1 2 f increasing1 2 f decreasing1 2 f increasing1 2 on - ∞, -4 on -4, 5 on 5, ∞ 1 2 1 2 1 2 2 Therefore, there is a relative maximum at x = -4: f -4 = 57 3.There is a 5 relative minimum at x = 5: f 5 = -63 6. 1 2 seCTion 2.2 1 2 The second derivative, f″ x , can be The function given by used to determine the concavity of a f x 1 x3 1 x2 20x 7 graph. 1 2 = 3 - 2 - + has the second derivative f ″ x 2x 1. Setting the second derivative equal If f″ x 0 for all x in an open interval 1 2 = - 7 to 0, we have x 1. Using test values, we can check the concavity on either I, then the graph of f is concave up over 0 = 2 side of x 1: 1 2 I. 1 2 0 = 2 If f″ x 6 0 for all x in an open interval I, then the graph of f is concave down Test Value x = 0 x = 1 over1 I.2 Sign of f ″ x f ″ 0 6 0 f ″ 1 7 0 A point of inflection occurs at x0, f x0 if f″ x0 = 0 and there is a Result 1 2 f is concave1 2 down f is concave1 2 up 1 1 change in concavity on either side of x0. on - ∞, 2 on 2, ∞ 1 1 22 1 2 1 2 1 2 1 Therefore, the graph of f is concave down over the interval - ∞, 2 and 1 concave up over the interval 2, ∞ . Since there is a change in concavity on 1 1 1 either side of x0 = 2, we also conclude that the point 2, -3112 is a2 point of 1 1 inflection, where f 2 = -3112. 2 1 2 The Second-Derivative Test can also For the function 1 2 be used to classify relative extrema: 1 3 1 2 f x = 3 x - 2 x - 20x + 7, If f′ c = 0 and f ″ c 7 0, then f c evaluating the second derivative, f ″ x = 2x 1, at the critical values yields is a relative minimum. 1 2 - 1 2 1 2 1 2 the following conclusions: If f′ c 0 and f ″ c 0, then f c 1 2 = 6 ● At x = -4, we have f ″ -4 6 0. Since f′ -4 = 0 and the graph is is a relative maximum. concave down, we conclude that there is a relative maximum at x = -4. 1 2 1 2 1 2 1 2 1 2 If f′ c = 0 and f ″ c = 0, then the ● At x = 5, we have f ″ 5 7 0. Since f′ 5 = 0 and the graph is concave up, First-Derivative Test must be used to we conclude that there is a relative minimum at x = 5. classify1 2 f c . 1 2 1 2 1 2

1 2 (continued)

Copyright 2016 Pearson. All rights reserved.

M02_BITT9391_11_SE_C02_EOC.indd 283 29/10/14 5:08 PM Not for Sale

284 Chapter 2 ● Applications of Differentiation

Key Terms and ConCepTs examples seCTion 2.3

A line x = a is a vertical asymptote if Consider the function given by 2 x - 1 lim- f x = ∞, xSa f x = 2 . x + x - 6 lim- f1x2 = - ∞, xSa Factoring,1 we2 have lim f1x2 = ∞, xSa+ x + 1 x - 1 f x = . 1 2 x 3 x 2 or 1 + 21 - 2 This expression1 2 is simplified. Therefore, x 3 and x 2 are vertical lim+ f x = - ∞. 1 21 2 = - = xSa asymptotes since 1 2 lim f x = ∞ and lim f x = - ∞ xS - 3- xS - 3+ and lim f x1 2= - ∞ and lim f x1 2= ∞. xS2- xS2+ 1 2 1 2 An asymptote is usually represented For the function given by y as a dashed line; it is not part of the x2 1 graph itself. - 5 f x = 2 , f x + x - 6 A line y = b is a horizontal asymptote if y = 1 we have 1 2 lim f x = b −5 5 x xS - ∞ lim f x = 1 and lim f x = 1. xS -∞ xS ∞ x = ±3 x = 2 or 1 2 Thus, the line y1 =2 1 is a horizontal asymptote.1 2 −5 lim f x = b. xS ∞ 1 2 2 For a rational function of the form x + 1 y Let f x = . Division yields f f x = P x Q x , a slant asymptote x + 3 5 occurs if the degree of the numerator 1 2 10 is1 12 greater1 2>than1 the2 degree of the f x = x - 3 + . x 3 −10 −5 510 x denominator. + As x S ∞ or x S - ∞, Slant asymptote: 1 2 10 y = x − 3 the remainder S 0. −10 x + 3 Therefore, the slant asymptote is y = x - 3. −15

Asymptotes, extrema, x- and Consider the function given by y 1 3 1 2 y-intercepts, points of inflection, f x = 3x - 2x - 20x + 7. (±4, 57.66...) 100 concavity, and intervals over 2 f ● f has a relative maximum point at -4, 573 which the function is increasing or 1 2 5 and a relative minimum point at 5, -636 . decreasing are all used in the strategy 1 11 2 ● f has a point of inflection at , -3 . −10 −5 510 x for accurate graph sketching. 2 1 12 2 ● f is increasing over the intervals - ∞, -4 and 5, ∞ , decreasing over the1 interval2 −100 -4, 5 , concave down over the 1interval 2 (5, ±63.83...) 1 1 2 1 - ∞, 2 , and concave up over the interval 2, ∞ . 1 2 ● f has a y-intercept at 0, 7 . 1 2 1 2 1 2

Copyright 2016 Pearson. All rights reserved.

M02_BITT9391_11_SE_C02_EOC.indd 284 29/10/14 5:08 PM Not for Sale Chapter 2 summary 285

Key Terms and ConCepTs examples seCTion 2.3 (continued) Consider the function given by y 2 x - 1 5 f x = 2 . f x + x - 6 y = 1 ● f has vertical1 2 asymptotes x = -3 and x = 2. −5 5 x ● f has a horizontal asymptote given x = ±3 x = 2 by y = 1. −5 1 ● f has a y-intercept at 0, 6 . ● f has x-intercepts at -1, 0 and 1, 0 . 1 2 ● f has a relative maximum at 0.101, 0.168 and a relative minimum1 at 2 9.899,1 0.9522 . 1 2 ● f is concave up on - ∞, -3 and 2, 14.929 , 1 2 and concave down on -3, 2 and 14.929, ∞ . 1 2 1 2 seCTion 2.4 1 2 1 2

If f is continuous over a, b , then the y Extreme-Value Theorem tells us that f Absolute maximum at b will have both an absolute3 4maximum value and an absolute minimum Relative value over this interval. One or both Maxima maximum absolute extrema may occur at an at c3 endpoint of this interval. Maximum–Minimum Principle 1 can Minima Relative be used to determine these absolute minima at c and c extrema: we find all critical values 1 4 Absolute c1, c2, c3,c, cn, in a, b , then evaluate minimum at c1 f a , f c1 , f c2 , f c33 , c4 , f cn , f b . abc1 c2 c3 c4 x The1 2 largest1 2 of1 these2 1 is2 the absolute1 2 1 2 maximum, and the smallest is the absolute minimum.

If f is differentiable for all x in 2 y Let f x = x + , for x 7 0. The derivative an interval I, and there is exactly x 4 one critical value c in I such that 1 2 2 f f′ c 0, then, according to is f′ x = 1 - . We solve for the critical value: = x2 Maximum–Minimum Principle 2, 1 2 f 1c 2 is an absolute extremum over I. 2 2 (√2, f(√2)) 1 - = 0 x2 1 2 x2 = 2 24x x = { 2. The only critical value 2over the interval where x 7 0 is x = 2. The second 24 derivative is f ″ x = . We see that x3 14 2 f ″ 2 0. Therefore, = 3 7 2 2 1 2, f 2 2 2 is an absolute minimum. 1 2 2 2 1 1 22 (continued)

Copyright 2016 Pearson. All rights reserved.

M02_BITT9391_11_SE_C02_EOC.indd 285 29/10/14 5:08 PM Not for Sale

286 Chapter 2 ● Applications of Differentiation

Key Terms and ConCepTs examples seCTion 2.5 Many real-world applications involve See Examples 1–6 in Section 2.5 and the problem-solving strategy on p. 245. maximum–minimum problems.

seCTion 2.6

Marginal cost, marginal revenue, y and marginal profit are estimates of the cost, revenue, and profit for the x + 1 st item produced: y = C(x)

● C′ x ≈ C x + 1 - C x ,

1 2 Cost so C x + 1 ≈ C x + C′ x . 1 2 1 2 1 2 ● R′ x ≈ R x + 1 - R x , C′(x) ≈ C(x + 1) − C(x) 1 2 1 2 1 2 so R x + 1 ≈ R x + R′ x . 1 2 1 2 1 2 ● P′ x ≈ P x + 1 - P x , 1 2 1 2 1 2 x x + 1 x so P x + 1 ≈ P x + P′ x . Quantity produced 1 2 1 2 1 2 1 2 1 2 1 2 2 Delta notation represents the change For f x = x , let x1 = 2 and x2 = 2.1. Then, ∆x = 2.1 - 2 = 0.1. in a variable: Since f x1 = f 2 = 4 and f x2 = f 2.1 = 4.41, we have 1 2 ∆x = x2 - x1 1 ∆2y = 1f x22 - f x1 1= 4.412 -1 4 2= 0.41. and Since ∆x = 10.12 is small,1 2 we can approximate ∆y by ∆y = f x2 - f x1 . ∆y ≈ f′ x ∆x. If x2 = x1 + h, then1 2 ∆x 1= h2. The derivative is f1′ 2x = 2x. Therefore, If ∆x is small, then the derivative can ∆y ≈ f′ 21 2# 0.1 = 2 2 # 0.1 = 0.4. be used to approximate ∆y: This approximation,1 2 0.4,1 is2 very close1 2 1 to the2 actual difference, 0.41. ∆y ≈ f′ x # ∆x. 3 Differentials allow1 us2 to approximate For y = x, find dy when x = 27 and dx = 2. changes in the output variable y given dy 1 1 Note 2that . Thus, dy dx. Evaluating, we have a change in the input variable x: = 3 = 3 dx 3 x2 3 x2 dx x = ∆ 1 2 2 2 dy = 2 = . and 3 3 27 2 27 1 2 3 dy = f′ x dx. This result can be2 used to approximate the value of 29, using the fact that 3 3 1 2 If ∆x is small, then1 2 dy ≈ ∆y. 29 ≈ 27 + dy: 2 2 2 In practice, it is often simpler to 3 3 2 29 ≈ 27 + dy = 3 + ≈ 3.074. calculate dy, and it provides a close 27 approximation for ∆y. 2 2 Thus, the approximation 3 29 ≈ 3.074 is very close to the actual value, 3 29 = 3.07231 c, found2 with a calculator. 2 seCTion 2.7

The elasticity of demand E is a Business. The Leslie Davis Band finds that demand for its CD at perfor- function of x, the price: mances is given by x # D′ x q = D x = 50 - 2x, E x = - . D x 1 2 where x is the price,1 2 in dollars, of each CD sold and q is the number of CDs 1 2 sold at a performance. Find the elasticity when the price is $10 per CD, and 1 2 interpret the result. Then, find the price at which revenue is maximized.

Copyright 2016 Pearson. All rights reserved.

M02_BITT9391_11_SE_C02_EOC.indd 286 29/10/14 5:08 PM Not for Sale Chapter 2 summary 287

Key Terms and ConCepTs examples seCTion 2.7 (continued)

When E x 7 1, total revenue is Elasticity at x is given by decreasing; x # D′ x when E 1x 2 6 1, total revenue is E x = - D x increasing; 1 2 and 1 2 1 2 x -2 = - 1 2 when E x = 1, total revenue is 50 - 2x maximized. 1 2 -2x x 1 2 = - = . 50 - 2x 25 - x 10 2 Thus, E 10 = = . 25 - 10 3 1 2 Since E 10 is less than 1, the demand for the CD is inelastic, and a modest increase in price will increase revenue. Revenue1 2 is maximized when E x = 1: x = 1 1 2 25 - x x = 25 - x 2x = 25 x = 12.5. At a price of $12.50 per CD, revenue will be maximized.

seCTion 2.8

If an equation has variables x and y dy 5 3 Find if y = x + 7. and y is not isolated on one side of dx the equation, the derivative dy dx can dy We differentiate both sides with respect to x, and then solve for . be found without solving for y using dx implicit differentiation. > d 5 d 3 d y = x + 7 dx dx dx dy 5y4 = 3x2 dx dy 3x2 = . dx 5y4

A related rate occurs when the rate of A large cube of ice is melting, losing 30 cm3 of its volume, V, per minute. change of one variable (with respect When the length of a side is 20 cm, how fast is that length decreasing? to time) can be calculated in terms Since V = x3 and both V and x are changing with time, we differentiate of the rate of change (with respect to each variable with respect to time: time) of another variable of which it is a function. dV dx = 3x2 . dt dt dV We have x = 20 and = -30. Evaluating gives dt

2 dx -30 = 3 20 dt dx 1 302 or = - = -0.025 cm min. dt 3 20 2 > 1 2

Copyright 2016 Pearson. All rights reserved.

M02_BITT9391_11_SE_C02_EOC.indd 287 29/10/14 5:08 PM Not for Sale

ChapteR 2 These review exercises are for test preparation. They can also be used as a practice test. Answers are at the back of the book. The red bracketed section references tell you what Review Exercises part(s) of the chapter to restudy if your answer is incorrect.

ConCept ReinfoRCement 7. A function with a first g) y derivative that is always Match each description in column A with the most appropriate positive graph in column B. [2.1–2.4]

Column A Column B x

1. A function with a relative a) y maximum but no absolute extrema In Exercises 8–16, classify each statement as either true or false. x 8. Every continuous function has at least one critical value. [2.1]

9. If a continuous function y = f x has extrema, they will 2. A function with both a vertical b) y always occur where f′ x = 0. [2.1] asymptote and a horizontal 1 2 asymptote 10. If f′ c = 0 and f ″ c 172 0, then f c is a relative minimum. [2.2] 1 2 1 2 1 2 x 11. If f′ c = 0 and f ″ c = 0, then f c cannot be a rela- tive minimum. [2.2] 1 2 1 2 1 2 12. If the graph of f x = P x Q x has a horizon- tal asymptote, then the degree of the polynomial 3. A function that is concave c) y P x is less than1 or2 equal1 to2> that1 2of the polynomial up and decreasing Q x . [2.3] 1 2 13. Absolute1 2 extrema of a continuous function f always oc- x cur at the endpoints of a closed interval. [2.4] 14. In general, if f is a continuous function, then ∆x = dx and ∆y Ú dy. [2.6] 15. If the elasticity of demand at a certain price x is 1, then 4. A function that is d) y revenue is maximized at this price. [2.7] concave up and increasing 16. In a related-rate problem, the variables are commonly differentiated implicitly with respect to time, t. x [2.8]

Review exeRCises For each function given, find any extrema, along with the 5. A function with e) y x-value at which they occur. Then sketch a graph of the three critical function. [2.1] values 2 4 2 17. f x = 4 - 3x - x 18. f x = x - 2x + 3

x 8x 1 2 - 20. f1x2 4 x 1 3 19. f x = 2 = + - x + 1 1 2 3 2 1 2 2 31 2 21. f x = x + x - x + 3 22. f x = 3x 3 2 > 6. A function with one f) y 23. f1x2 = 2x - 3x - 12x + 10 1 2 critical value and a 3 second derivative that 24. f1x2 = x - 3x + 2 is always positive 1 2 x

288 Copyright 2016 Pearson. All rights reserved.

M02_BITT9391_11_SE_C02_EOC.indd 288 29/10/14 7:04 PM Not for Sale Chapter 2 review exerCises 289

Sketch the graph of each function. List any minimum or maxi- one bicycle for a year. To reorder, there is a fixed cost of mum values, where they occur, and any points of inflection. $10, plus $2 for each bicycle. How many times per year State where the function is increasing or decreasing and where should the shop order bicycles, and in what lot size, in it is concave up or concave down. [2.2] order to minimize inventory costs? [2.5]

1 3 2 46. Business: marginal revenue. Crane Foods determines 25. f x = x + 3x + 9x + 2 3 that its daily revenue, R x , in dollars, from the sale of x frozen dinners is 1 2 2 26. f x = x - 10x + 8 1 2 R x = 4x3 4. [2.6] 3 2 27. f1x2 = 4x - 6x - 24x + 5 a) What is Crane’s> daily revenue when 81 frozen dinners 1 2 4 2 are sold? 28. f1x2 = x - 2x b) What is Crane’s marginal revenue when 81 frozen 4 3 2 29. f1x2 = 3x + 2x - 3x + 1 (Round to three decimal dinners are sold? places where appropriate.) c) Use the answers from parts (a) and (b) to estimate 1 2 1 5 3 4 4 3 R 82 . 30. f x = 5 x + 4 x - 3 x + 8 (Round to three decimal 3 places where appropriate.) For Exercises1 2 47 and 48, y  f x  2x  x. [2.6] 1 2 Sketch the graph of each function. Indicate where each func- 47. Find ∆y and dy, given that1 x2= 1 and ∆x = -0.05. tion is increasing or decreasing, the coordinates at which any relative extrema occur, where any asymptotes occur, where the 48. a) Find dy. graph is concave up or concave down, and where any inter- b) Find dy when x = -2 and dx = 0.01. cepts occur. [2.3] 49. Approximate 83 using ∆y ≈ f′ x ∆x. [2.6] 2x + 5 x 1 31. f x = 32. f x = 50. Physical science: waste storage. 1The2 Waste Isolation x + 1 x - 2 Pilot Plant (WIPP) in New Mexico consists of large 1 2 1 2 rooms carved into a salt deposit and is used for long- 5 x + 1 33. f x = 2 34. f x = - 2 term storage of radioactive waste. (Source: www.wipp x - 16 x - x - 2 .energy.gov.) A new storage room in the shape of a cube 1 2 2 1 2 2 x - 2x + 2 x + 3 with an edge length of 200 ft is to be carved into the salt. 35. f x = 36. f x = Use a differential to estimate the potential difference in x 1 x - the volume of this room if the edge measurements have 1 2 1 2 For each function, find any absolute extrema over the indicated a tolerance of {2 ft. [2.6] interval. Indicate the x-value at which each extremum occurs. 51. Economics: elasticity of demand. Consider the demand Where no interval is specified, use the real line. [2.4] function given by 4 2 37. f x = x - 2x + 3; 0, 3 q = D x = 400 - 5x. [2.7] 2 3 a) Find the elasticity. 38. f1x2 = 8x - x ; -1, 38 4 1 2 b) Find the elasticity at x = 10, and state whether the 1 2 50 3 4 39. f x = x + ; 0, ∞ demand is elastic or inelastic. x c) Find the elasticity at x = 50, and state whether the 1 2 1 2 demand is elastic or inelastic. 40. f x x4 2x2 1 = - + d) At a price of $25, will a small increase in price cause 41. Maximize1 2 Q = xy, where 3x + 4y = 24. [2.5] total revenue to increase or decrease? e) Find the price at which total revenue is a maximum. 2 2 42. Find the minimum value of Q = x - 2y , where x - 2y = 1. [2.5] 52. Differentiate the following implicitly to find dy dx. Then find the slope of the curve at the given point. 2 43. Business: maximizing profit. If R x = 52x - 0.5x > 2x3 2y3 9xy; 1, 2 [2.8] and C x = 22x - 1, find the maximum profit and the + = - - - 1 2 number of units that must be produced and sold in order 53. A ladder 25 ft long leans 1against a2 vertical wall. If the to yield1 this2 maximum profit. Assume that R x and bottom moves away from the wall at the rate of 6 ft sec, C x are in dollars. [2.5] how fast is the height of the top changing when the 1 2 lower end is 7 ft from the wall? [2.8] > 44. Business:1 2 minimizing cost. A rectangular box with a 3 square base and a cover is to have a volume of 2500 ft . If 54. Business: total revenue, cost, and profit. Find the rates 2 2 the cost of material is $2 ft for the bottom, $3 ft for the of change, with respect to time, of total revenue, cost, 2 top, and $1 ft for the sides, what should the dimensions and profit for > > of the box be in order to minimize the cost? [2.5] 2 > R x = 120x - 0.5x and C x = 15x + 6, 45. Business: minimizing inventory cost. Venice Cyclery when x 100 and dx dt 30 units per day. Assume 1 2 = = 1 2 sells 360 hybrid bicycles per year. It costs $8 to store that R x and C x are in dollars. [2.8] > Copyright 2016 Pearson. All rights reserved.1 2 1 2

M02_BITT9391_11_SE_C02_EOC.indd 289 29/10/14 5:08 PM Not for Sale

290 Chapter 2 ● Applications of Differentiation

synThesis 62. life and physical sciences: incidence of breast cancer. The following table provides data relating the incidence of 55. Find the absolute maximum and minimum values, if breast cancer per 100,000 women of various ages. they exist, over the indicated interval. a) Use regression to fit linear, quadratic, cubic, and 2 5 f x = x - 3 ; - ∞, ∞ [2.4] quartic functions to the data. (The function used in > Exercise 28 of Section R.1 was found in this manner.) 56. Find1 2the absolute1 2 maximum1 and2 minimum values of the piecewise-defined function given by AgE IncIdEncE pER 100,000 2 x2, for 2 x 1, - - … … 0 0 f x = 3x - 2, for 1 6 x 6 2, 27 10 2 x - 4 , for 2 … x … 6. [2.4] 1 2 32 25 57. Differentiate1 implicitly2 to find dy dx: 37 60 x y 4 x y 4 x6 y6. [2.8] 42 125 - c + + = + > 47 187 58. Find any relative maxima and minima of 1 2 1 2 52 224 4 3 2 y = x - 8x - 270x . [2.1, 2.2] 57 270 59. Determine a rational function f whose graph has a verti- 62 340 cal asymptote at x = -2 and a horizontal asymptote at 67 408 y = 3 and includes the point 1, 2 . [2.3] 72 437 1 2 77 475 82 460 TEchnology connEcTion 87 420

Use a calculator to estimate the relative extrema of each (Source: National Cancer Institute.) function. [2.1, 2.2] b) Which function best fits the data? 5 3 60. f x = 3.8x - 18.6x c) Determine the domain of the function on the basis of 3 2 the function and the problem situation. 61. f1x2 = ͉ 9 - x ͉ - 1 2 d) Determine the maximum value of the function over 1 2 the domain. At what age is the incidence of breast cancer the greatest? [2.1, 2.2]

ChapTer 2 Test

Find all relative minimum or maximum values as well as the Find the absolute maximum and minimum values, if they exist, x-values at which they occur. State where each function is increas- of each function over the indicated interval. Where no interval ing or decreasing. Then sketch a graph of the function. is specified, use the real line. 2 3 1. f x = x - 4x - 5 2. f x = 4 + 3x - x 13. f x = x 6 - x 3 2 1 1 2 2 3 1 2 16 14. f1x2 = x1 + x 2- x + 1; -2, 2 3. f x = x - 2 - 4 4. f x = x2 4 2 > + 15. f1x2 = -x + 8.6x + 10 1 2 1 2 1 2 3 4 Sketch a graph of each function. List any extrema, and indicate 16. f1x2 = -2x + 5; -1, 1 any asymptotes or points of inflection. 3 2 4 2 17. f1x2 = -2x + 5 3 4 5. f x = x + x - x + 1 6. f x = 2x - 4x + 1 2 3 2 18. f1x2 = 3x - x - 1 7. f1x2 = x - 2 + 3 8. f1x2 = x 9 - x 2 1 2 2 128 2 8 19. f x = x + ; 0, ∞ 1 2 1 2 1 2 - x 9. f x = 10. f x = 2 x - 1 x - 4 1 2 Q 2 1 2 1 2 2 1 2 20. Maximize = xy , where x + 2y = 50. x - 1 x - 3 11. f x = 12. f x = 21. Minimize Q x2 y2, where x y 10. x x + 2 = + - = 1 2 1 2 Copyright 2016 Pearson. All rights reserved.

M02_BITT9391_11_SE_C02_EOC.indd 290 29/10/14 5:08 PM Not for Sale Chapter 2 test 291

22. Business: maximum profit. Find the maximum profit synThesis and the number of units, x, of a leather briefcase that must be produced and sold in order to yield the maxi- 32. Find the absolute maximum and minimum values of the mum profit. Assume that R x and C x are the revenue following function, if they exist, over [0, ∞): and cost, in dollars, when x units are produced: x2 2 1 2 1 2 f x = 3. R x = x + 110x + 60, 1 + x 2 C x = 1.1x + 10x + 80. 1 2 1 2 33. Business: minimizing average cost. The total cost in 23. Business:1 2 minimizing cost. From a thin piece of card- dollars of producing x units of a visor is given by board 60 in. by 60 in., square corners are cut out so that x3 the sides can be folded up to make an open box. What C x = 100x + 100 x + . 2100 dimensions will yield a box of maximum volume? What 2 is the maximum volume? How1 many2 units should be produced to minimize aver- age cost? 24. Business: minimizing inventory costs. Ironside Sports sells 1225 tennis rackets per year. It costs $2 to store one tennis racket for a year. To reorder, there is a fixed cost of $1, plus $0.50 for each tennis racket. How many times TEchnology connEcTion per year should Ironside order tennis rackets, and in 34. Estimate any extrema of the function given by what lot size, in order to minimize inventory costs? 3 2 f x = 5x - 30x + 45x + 5 x. 25. For y f x x2 3, x 5, and ∆x 0.1, find ∆y 2 = = - = = 35. Estimate any extrema of the function given by and f′ x ∆x. 1 2 5 3 1 2 g x = x - x . 26. Approximate1 2 50 using ∆y ≈ f′ x ∆x. 36. Maximize Q x2y2, where x2 y 20 and x and y 22 1 2 = + = 27. For y = x + 3: 1 2 are positive. a) Find dy2. 37. Business: advertising. The business of manufacturing b) Find dy when x = 2 and dx = 0.01. and selling bowling balls is one of frequent changes. 28. Economics: elasticity of demand. Consider the demand Companies introduce new models to the market about function given by every 3 to 4 months. Typically, a new model is created because of advances in technology such as new surface 600 stock or a new way to place weight blocks in a ball. To q = D x = 2. x + 4 decide how to best use advertising dollars, companies 1 2 track sales in relation to amount spent on advertising. a) Find the elasticity.1 2 Suppose a company has the following data from past b) Find the elasticity at x = 1, stating whether demand sales. is elastic or inelastic. c) Find the elasticity at x 12, stating whether demand AmounT SpEnT numBER of BowlIng = on AdvERTISIng BAllS Sold, N is elastic or inelastic. (in thousands) d) At a price of $12, will a small increase in price cause total revenue to increase or decrease? $ 0 8 e) Find the price at which total revenue is a 50 13,115 maximum. 100 19,780 150 22,612 29. Differentiate the following implicitly to find dy dx. Then 200 20,083 find the slope of the curve at 1, 2 : > 250 12,430 x3 y3 9. + = 1 2 300 4 30. A spherical balloon has a radius of 15 cm. Use a differen- tial to find the approximate change in the volume of the a) Use regression to fit linear, quadratic, cubic, and balloon if the radius is increased or decreased by 0.5 cm. quartic functions to the data. (The volume of a sphere is V 4 pr3. Use 3.14 for p.) = 3 b) Determine the domain of the function in part (a) that 31. A pole 13 ft long leans against a vertical wall. If the best fits the data and the problem situation. How did lower end moves away from the wall at the rate of you arrive at your answer? 0.4 ft sec, how fast is the upper end sliding down when c) Determine the maximum value of the function over the lower end is 12 ft from the wall? the domain. How much should the company spend > on advertising a new model in order to maximize the number of bowling balls sold?

Copyright 2016 Pearson. All rights reserved.

M02_BITT9391_11_SE_C02_EOC.indd 291 29/10/14 5:08 PM Not for Sale

292 Chapter 2 ● Applications of Differentiation r.5 ● Nonlinear Functions and Models 292 ExTEndEd TEchnology APPlicATion

Maximum Sustainable Harvest In certain situations, biologists are able to determine what is called a reproduction curve. This is a function y = f P such that if1 P 2is the population at a given point in time, then f P is the population a year later. Such a curve is shown below. 1 2

y Population Reproduction function, 1 year y = f(P) later ( P, f(P)) y = P

f(P)

Population P f(P)

The line y = P is significant because if it coincides with the curve y = f P , then we know that the popula- Reproduction function, tion stays the same from year to year. Here the graph of f y m = 1 y = f(P) lies mostly above the1 line,2 indicating that the population is increasing. Too many deer in a forest can deplete the food supply and cause the population to decrease. In such cases, Maximum sustainable hunters are often allowed to “harvest” some of the deer. harvest, f(P ) − P Then with a greater food supply, the remaining deer pop- 0 0 ulation may prosper and increase. We know that a population P will grow to a popula- P0 P tion f P in a year. If the population were increasing, then hunters could “harvest” the amount 1 2 f P - P Let the harvest function H be given by each year 1without2 shrinking the initial population P. If H P = f P - P. the population were remaining the same or decreasing, Then H ′ P = f′ P 1. then such a harvest would deplete the population. 1 2 1 2 - Suppose we want to know the value of P0 that will Now, if we1 assume2 1that2 H′ P exists for all values of P and allow the harvest to be the largest. If we could determine that there is only one critical value, it follows that the max- 1 2 that P0, we could let the population grow until it reached imum sustainable harvest occurs at that value P0 such that that level and then begin harvesting year after year the H ′ P0 = f′ P0 - 1 = 0 amount f P0 - P0. and H ″1P02 = f ″1P02 6 0. 1 2 1 2 1 2

292 Copyright 2016 Pearson. All rights reserved.

M02_BITT9391_11_SE_C02_EOC.indd 292 29/10/14 5:09 PM Not for Sale Extended Technology Application 293

Or, equivalently, we have the following. For Exercises 4 and 5, do the following. a) Graph the reproduction curve, the line y P, and the harvest function using the same viewing window. Theorem b) Graphically determine the population at which the The maximum sustainable harvest occurs at P0 maximum sustainable harvest occurs. such that c) Find the maximum sustainable harvest.

f′ P0 = 1 and f ″ P0 6 0, 4. f P = 40 P, where P is measured in thousands. Assume that1 this is the reproduction curve for the and is given1 2by 1 2 brown1 2 trout population in a large lake. H P0 = f P0 - P0. 1 2 1 2 Exercises For Exercises 1–3, do the following. a) Graph the reproduction curve, the line y P, and the harvest function using the same viewing window. b) Find the population at which the maximum sustainable harvest occurs. Use both a graphical solution and a calculus solution. c) Find the maximum sustainable harvest. 1. f P = P 10 - P , where P is measured in thousands. 2 2. f1P2 = -10.025P 2+ 4P, where P is measured in thousands. This is the reproduction curve in the 2 Hudson1 2 Bay area for the snowshoe hare. 5. f P = 0.237P 2000 - P , where P is measured in 2 thousands. 2 3. f P = -0.01P + 2P, where P is measured in thou- 1 2 sands. This is the reproduction curve in the Hudson 6. The table below lists data regarding the reproduction Bay1 2 area for the lynx. of a certain animal. a) Use regression to fit a cubic polynomial to these data. b) Graph the reproduction curve, the line y = P, and the harvest function using the same viewing window. c) Graphically determine the population at which the maximum sustainable harvest occurs.

PoPulation, P PoPulation, f(P), (in thousands) 1 Year later

10 9.7 20 23.1 30 37.4 40 46.2 50 42.6

Copyright 2016 Pearson. All rights reserved.

M02_BITT9391_11_SE_C02_EOC.indd 293 12/11/14 11:02 PM